Sie sind auf Seite 1von 160

14

A. Moral Philosophy, Right & Wrong, & the Law Governing Lawyers 14

B. The Role of the Lawyer 14


i. Standard Conceptions of the Lawyer’s Role 14
ii. Roles of the Lawyer 14
iii. Practice Settings 15

15

A. Organization of the Bar 15


i. The American Bar Association 15
ii. Alternative National Bar Associations 15
iii. State Bar Associations 16

B. Sources of Law Governing Lawyers: Ethics Codes 16


i. ABA models and their organization 16
ii. 1969 Model Code of Professional Responsibility 16
iii. 1983 Model Rules of Professional Conduct 16
iv. State-Adopted Codes 16

C. Sources of Law Governing Lawyers: Case Authority, Ethics Opinions, Restatement,


Constitutional Constraints, “Other Law” 16
i. Case Authority 16
ii. Ethics Opinions 17
iii. Restatement of Law Governing Lawyers 17
iv. Constitutional Constraints 17
v. “Other Law” 17

D. Admission to the Bar 17


i. Model Rule 8.1 Bar Admission and Disciplinary Matters 17
ii. Bar Admission Criteria at Issue and Citation 18
iii. Mental Health of Applicants 18
iv. Law School Discipline: A Preliminary Screening Process 18

19

A. Professional Discipline 19

B. Discipline Process 19
i. How does a disciplinary proceeding work? 19
ii. Do the lawyer disciplinary agencies investigate most of the complaints? 19
iii. Other Questions 19

C. Grounds for Discipline 20


i. What kinds of professional conduct can result in discipline? 20
ii. Can a lawyer be disciplined for advising a client about proposed conduct that may be
criminal or fraudulent? 20
iii. Model Rule 1.2(d) 20
iv. Model Rule 1.0(d) “Fraud” or “Fraudulent” 20
v. Model Rule 1.0(f) “Knowingly,” “Known,” or “Knows 21
vi. Model Rule 1.0(i) “Reasonable Belief” or “Reasonably Believes” 21
vii. Under what circumstances might a lawyer commit or assist a fraud by failing to state a fact
(omission) or by telling a half-truth? 21
viii. Can lawyers be disciplined for conduct that has nothing to do with the practice of law? 21
ix. Can a lawyer be disciplined for committing a crime? 21
x. What if a lawyer has not been convicted or charged with the crime? 21
xi. Other Questions 21
xii. Model Rule 8.4(c) Misconduct 22

D. Reporting Misconduct of Other Lawyers 22


i. Model Rule 8.3 Reporting Professional Misconduct 22
ii. Standards and Application of Model Rule 8.3 23

E. Responsibility for Ethical Misconduct by Colleagues and Superiors 23


i. Model Rule 5.1: Responsibilities Of Partners, Managers, & Supervisory Lawyers 23
ii. Model Rule 5.2: Responsibilities of A Subordinate Lawyer 24
iii. Model Rule 5.3: Responsibilities Regarding Nonlawyer Assistance 24
iv. Can discipline be imposed on a whole law firm? 25
v. Can a law student get in trouble for violating an ethics rule? 25
vi. Does the law provide any protection for lawyers who are fired because they refuse to
participate in unethical conduct or because they report misconduct of other lawyers to the
disciplinary authorities? 25

25

A. What Clients Can Sue For 25

B. Legal Malpractice 25
i. General 25
ii. Elements of a tort claim of Legal Malpractice 26
iii. Most common mistakes made by lawyers that lead to malpractice liability 26
iv. Available Remedies 26
v. Fiduciary Duties Owed by a Lawyer to a Client 26
vi. Elements of a Claim of Breach of Fiduciary Duty 26
vii. Implications of one act of misconduct 26
viii. Liability to a non-client 26
ix. Malpractice Insurance 27
x. Other Civil Liability of Lawyers 27

27

A. General 27

B. Rule 3.8 Special Responsibilities of a Prosecutor 27


i. The prosecutor in a criminal case shall: 27

C. Client Protection Funds 28

2
28

A. Basic Principle 28

B. Rule 1.6 Confidentiality of Information 28


i. Rule 1.6 28
ii. Comment [4] of Model Rule 1.6: Paragraph (a) 29
iii. Comment [18] of Model Rule 1.6: Paragraph (c) 29

C. Implications of Confidentiality Duties 29


i. Information that must be protected as confidential 29
ii. Lawyer’s duty to take precautions 30
iii. Consequences of an attorney’s failure to protect confidences 30

D. Protection of Info if there is a Reasonable Prospect of Harm to Client Interests 30


i. Adverse Effects 30
ii. How to know if there is a “reasonable prospect” of harm to client interests? 30

E. Revelation of Past Criminal Conduct General Rule 30

F. Exceptions to the Duty to Protect Confidences 30


i. Extent to which Exceptions Permit Disclosure (Comment 16) 30
ii. When is revelation allowed in relation to stage of proceedings? 30
iii. How much can a lawyer reveal if authorized? 31
iv. Should the lawyer inform the client before revealing confidential information? 31
v. Exception: Client Consent 1.6(a) 31
vi. Exception: Risk of Future Injury or Death  1.6 Comment 6: Paragraph (b)(1) and
Spaulding v. Zimmerman 31
vii. Exception: Client Fraud and Crimes Causing Financial Harm 1.6(b)(2), (b)(3) 31
viii. Exception: Revealing Confidences to Obtain Advice about Legal Ethics 1.6(b)(4) 32
ix. Exception: Using a Client’s Confidential Information to Protect Lawyer 1.6(b)(5) 32
x. Exception: Rule 3.3 Candor Toward Tribunal 32
xi. Exception: Rule 1.16 Declining or Terminating Representation 33
xii. Exception: Rule 4.1 Truthfulness in Statements to Others 34
xiii. Exception: Rule 8.4(c) Misconduct 34
xiv. Conditions in which a lawyer may reveal confidential information in self-defense 34
xv. Revealing Confidence to Comply w/a Court Order or Other Law 34
xvi. Revealing Confidences to Prevent Certain Conflicts of Interests 34
xvii. Summary of Exceptions in Rule 1.6 34

35

A. Confidentiality v. Attorney-Client Privilege 35


i. General 35
ii. Difference in Scope 35
iii. Different Modes of Enforcement 35
iv. Chart of Differences 35

B. Attorney-Client Privilege 36
i. How does A-C privilege protect the right to be represented by counsel? 36
ii. Can showing a need for the information overcome a privilege claim? 36
iii. Chart w/Occasions Where Privilege May Be Claimed 36
iv. Source of Attorney-Client Privilege 37
3
C. Elements of Attorney-Client Privilege 37
i. Communication 37
ii. Privileged Persons 37
iii. Communications in Confidence 37
iv. Communication for Purpose of Seeking Legal Assistance 37
v. Chart of Requirements 37
vi. Chart w/Consequences if a Communication is Privileged 38

D. Client Identity under Attorney Client Privilege 38

E. Waiver 38
i. Waiver by Client 38
ii. Waiver by Attorney 38
iii. Waiver by Putting Privileged Information Into Issue 39
iv. Waiver as a Conversation by Disclosure of Part of It 39
v. Compliance w/Court Order 39

F. Crime Fraud Exception 39


i. General 39
ii. A conversation where a client asks a lawyer for advice in planning or help in committing a
crime or fraud 39
iii. A client asks his lawyer for advice about a past criminal or fraudulent act 39
iv. If a client asks a lawyer for advice, learns the planned conduct is criminal, and doesn’t
commit the crime, is the convo privileged? 39
v. A client consults a lawyer about a planned crime but doesn’t know the planned conduct is
criminal 39

G. Procedure for Challenging a Claim of Privilege 39

H. Revelations Permitted or Required by the Ethics Code 39

I. Death of a Client 40
i. General 40
ii. Swidler & Berlin v. United States 40

J. Work-Product Doctrine 40
i. Origins 40
ii. What does it protect and apply to? 40
iii. What does it not protect and apply to? 40
iv. Limitations In Practice 41
v. Protection of a Lawyer’s Mental Impressions 41
vi. Protection of Word Product, Not Underlying Information 41
vii. Expert Witnesses 41

K. Privilege for a Corporation Client 41


i. Upjohn Case 41

L. Privilege for Government Officials 42

42

A. Formation of the Relationship 42


i. General Questions 42

4
ii. Rule 6.2 Accepting Appointments 43
(c) the client or the cause is so repugnant to the lawyer as to be likely to impair the client-lawyer relationship
or the lawyer's ability to represent the client 43
iii. Existence of an attorney-client relationship (Togstad v. Vesely, Otto, Miller & Keefe) 43

B. Lawyer’s Responsibilities as Agents 43


i. General 43
ii. Express and Implied Authority 43
iii. Apparent Authority 43
iv. Authority to Settle 44

C. Lawyers’ Duties of Competence, Honesty, Communication, and Diligence 44


i. Rule 1.1 Competence 44
ii. Comment [2] of Rule 1.1 44
iii. Competence in Criminal Cases 44
iv. 6th Amendment Requirements for Ineffective Counsel (Strickland v. Washington) 44
v. Examples of 6th Amendment Ineffective Assistance of Counsel 45

D. Lawyers’ Duty of Diligence 45


i. Rule 1.3: Diligence 45
ii. Comments [1],[2],[3],[4] for Rule 1.3 45

E. Lawyers’ Duties of Candor and Communication 46


i. Rule 1.4 Communication 46
ii. Rule 2.1 Advisor 46
iii. Candor in Counseling 47

F. Who Calls the Shots 47


i. Rule 1.2 Scope of Representation and Allocation of Authority Between Client and
Lawyer 47
ii. Restatement in re 1.2 48
iii. Jones v. Barnes 48
iv. 1.16(b) When a Lawyer May Withdraw Representation 48
v. The Competent Adult Client 49
vi. Rule 1.14 Clients with Diminished Capacity 49
vii. Clients who May Have Mental Impairments 50
viii. Can a lawyer avoid making decisions for a possibly impaired client by getting someone else
to make those decisions? 50
ix. Delinquency Cases 50

G. Terminating a Lawyer-Client Relationship 50


i. Duties to the Client at the Conclusion of the Relationship 50
ii. If a client has not paid the bill at the end of the representation, may the lawyer keep the
client’s documents until the client pays? 51
iii. Grounds for Termination Before Work is Completed 51
iv. Rule 1.16 Declining or Terminating Representation 51
v. Fees after withdrawing from representation 52

H. Rule 1.8(b) Lawyer’s Duty to Not Use Information to Disadvantage of Client 52

52

A. Introduction 52

5
i. General 52
ii. Possible Consequences of Representing a Client in the Face of Conflict 52
iii. Difficulties of Study of Conflict of Interest 53

B. Conflict Rule Chart 53

C. General Rules 53
i. Rule 1.7 Conflict of Interest: Concurrent Conflicts 53
ii. Comment [2] of Rule 1.7: Requirements for Resolution of Conflict of Interest Problem 54
iii. Rule 1.10 Imputation of Conflict of Interest and Firms 54
iv. Rule 1.0 (k) “Screened” 55

D. Evaluating Concurrent Conflicts 55


i. Overview of Rule 1.7 55
ii. Direct Adversary and Material Limitation (Rule 1.7) 55
iii. Personal Interest Conflicts 56
iv. How to Evaluate Conflicts 57
v. Non-Consentable Conflicts 57
vi. Informed Consent 58
vii. Comment 20 Consent Confirmed in Writing 58
viii. Comments 23-25 Conflicts in Litigation 59
ix. Withdrawal and Disqualification 60
x. Imputation of Concurrent Conflicts 60

60

A. Conflicts Between Current Clients in Civil Litigation 60


i. Suing a Current Client 60
ii. Cross-Examining a Current Client 60
iii. Representation of Co-Plaintiffs or Co-Defendants in Civil Litigation 60

B. Positional Conflicts 61
i. Representing Economic Competitors in Unrelated Matters (Rule 1.7, Comment [6]) 61

C. Conflicts Involving Prospective Clients 62


i. Rule 1.18 Duties to Prospective Client 62
ii. Comment [2] of Rule 1.18 62

D. Representing Both Parties to a Transaction 63


i. Standards Applied if Both Buyer and Seller Want You to Represent Them Both 63
ii. How does a lawyer know whether she needs to obtain informed consent? 63

E. Representing Organizations 63
i. Rule 1.13 Organization as Client 63
ii. Factors Affecting Whether a Related Entity is a Client 64
iii. Representing the Entity and Employees 64
iv. Duty to Protect Confidence and Employees 64
v. Responding to Unlawful Conduct by Corporate Officers and Other Employees 65
vi. Entity Lawyers on Boards of Directors 65

F. Representing Co-Defendants in Criminal Cases 65


i. Costs and Benefits of Joint Representation 65
ii. Ethics Rules and the Sixth Amendment 65

6
iii. Relevant Rules 65

G. Representing Family Members 65


i. Representing Both Sides in a Divorce 65
ii. Representing Family Members in Estate Planning 65

H. Representing Insurance Companies and Insured Persons 66


i. Fundamental Dilemma 66
ii. Who is the client? 66
iii. When is there a conflict of interest between the insurer and the insured? Rule 1.8(f) 66
iv. What should the lawyer do if a conflict arises? 66

I. Rule 1.8 (g) Aggregate Settlements/Pleas 66

J. Relevant Rules 66

67

A. Conflicts Between Present and Former Clients 67


i. Timing and Possible Implications 67
ii. Applicable Ethics Rules 67
iii. Is a conflict between a present and former client both a successive and current conflict? 67
iv. Chart: Scenarios w/Present and Former Clients 67
v. Are the ethical rules less restrictive as to successive conflicts than they are as to concurrent
conflicts? 67

B. Duties to Former Clients 67


i. What duties does a lawyer owe to a former client? 68
ii. Rule 1.9 Duties to Former Clients 68
iii. Comment [3] “Substantially Related” 68
iv. Distinguishing Present and Former Clients 69

C. Evaluating Successive Conflicts 69


i. Critical question 69
ii. Applying Rule 1.9 (General) 69
iii. “The Same Matter” 69
iv. “Substantial Relationship” 70
v. Material Adversity 71

D. Courses of Action Available to a Lawyer Who Discovers a Conflict Between a Former and
Present Client 71

E. Representing the Competitor of a Former Client 71


i. A comment to rule 1.7 states that ordinarily representation of economic competitors poses no serious
conflict of interest 71
ii. If it is not a conflict to represent economic competitors concurrently, one would think it would be
less problematic to represent the economic interests of a former client 71

F. Conflicts between the interests of a present client and a client who was represented by a
lawyer’s former firm 71
i. Analyzing Former Firm Conflicts  Rule 1.9 (b) 71
ii. Using or Revealing a Former Client’s Confidences  Rule 1.9 (c) 71

G. Imputation of Former Client Conflicts to Affiliated Lawyers 71


7
i. General Rule 71
ii. Other situations besides those listed in 1.10(a)(2) in which the Model Rules permit
screening? 72

72

A. Legal Fees 72
i. Rule 1.5 Fees 72
ii. Matter of Fordham  Fees When You Need to Become Competant 73
iii. 1.5 (b)  See Rule 7.1 Communications Concerning A Lawyer’s Services 73
iv. Other Rules That May Come Up As a Result 74
v. Contingent Fees 74

B. Forbidden and Restricted Fee and Expense Arrangements 74


i. Rule 1.8 (i) Buying Legal Claims 74
ii. When is a lien okay?  Comment [16] Comment [2] 74
iii. Rule 1.8 (e) Financial Assistance to a Client 74
iv. [Comment 10] Financial Assistance 74
v. Rule 1.8 (d) Publication Rights 75
vi. Advance Payment of Fees and Nonrefundable Retainer Fees 75

C. Fee Disputes 75
i. Rule 1.8 (h) Prospective Limitations of Lawyers’ Liability and Settlement of Claims
Against Lawyers 76
ii. Comment [14] of Rule 1.8(h) Fee Arbitration 76
iii. Comment [14] of Rule 1.8(h) Limited Liability Entity 76
iv. Collection of Fees 76

D. Dividing Fees with Other Firms 76


i. Rule 1.5 (e) Dividing Fees w/Lawyers Not in Same Firm 76
ii. Referral Fees 76
iii. What does it mean for a lawyer to take “financial and ethical responsibility for the
representation? 77

E. Sharing Fees w/Non Lawyers 77


i. Rule 5.4(a) 77
ii. District of Columbia and Other Countries (i.e. UK) 77
iii. “Runners” who find clients that are accident victims 77

F. Rule 1.8 (f) Payment of Fees by Third Parties 77


i. Rule 77
ii. Comment [13] 78

G. Lawyers as Custodians of Client Property and Documents 78


i. Client Trust Accounts 78
ii. Rule 1.15 Safekeeping Property 78
iii. May a lawyer act as executor of a client’s estate? 79

H. Conflicts with Lawyers’ Personal or Business Interests 79


i. In General  1.7 (a)(2) 79
ii. Rule 1.8 (a) Business Transactions between Lawyers and Clients 79
iii. Before you make a deal w/a client ask these questions: 79
iv. Do restrictions on lawyer client business deals apply to every lawyer-client contract? 80

8
v. Can a lawyer accept stock or stock options instead of cash as payment for legal services? 80
vi. Rule 1.8 (c) Gifts from Clients 80
vii. Unsolicited Gifts 80

I. Sexual Relations w/Clients and Adverse Attorneys 80


i. Rule 1.8 (j) Sexual Relations w/Clients 80
ii. With Adverse Attorneys 80

J. Imputed Conflicts to Other Lawyers 80


i. Rule 1.8 (k) Extension of 1.8(a) through (i) to other attorneys in a firm 80
ii. Rule 1.10 (a) 80

K. Other Rules That May Come Up As a Result 80

81

A. Regulation of Government Lawyers 81


i. The law governing lobbying 81
ii. Conflict of Interest and “Revolving Door” Statutes 81

B. Successive Conflicts of Former and Present Government Lawyers 81


i. Background 81
ii. Present and former are subject to 1.9(c) 81
iii. Rule 1.11 (e) Defining “matter” in the context of 1.11 81

C. Conflicts of Former Government Lawyers in Private Practice 82


i. General 82
ii. Exceptions 82
iii. 1.19 (a) doesn’t apply, but 1.11(a)(2) does 82
iv. Rule 1.11(a) Evaluating Conflicts for Former Government Lawyers 82
v. Rule 1.11(b) Screening a Former Government Lawyer 83
vi. Rule 1.11(c) Confidential Government Information and Former Government Attorney
83
vii. Bases for Disqualification of Former Government Lawyers 83

D. Conflicts of Government Workers Who Formerly Worked in Private Practice 84


i. Rule 1.11 (d) Special Conflicts for Current Government Lawyers 84

84

A. History of Judicial Ethics Codes in the United States 84

B. Overview of the Model Code of Judicial Conduct (Canons 1-4) 84

C. Impartiality and fairness; avoidance of bias, prejudice, and harassment 85


i. The canons 85
ii. Model Code of Judicial Fairness 3.6 85
iii. CC: 2.4 External Influences on Judicial Conduct 85

D. Ex Parte Communications 85
i. Circumstances where ex parte conduct is permissible 85
ii. CC: 2.9 Ex Parte Communications 85

9
E. Disqualification of Judges 86
i. General 86
ii. CC: 2.11 Disqualification 86

F. Conflicts Rules for Former Judges, Law Clerks, Arbitrators, and Mediators 88
i. Model Rule 1.12 Former Judge, Arbitrator, Mediator or Other Third-Party Neutral 88

88

A. Model Rule 3.2 Expediting Litigation 88

B. Investigation Before Filing a Complaint 89


i. Rule 3.1 Meritorious Claims and Contentions 89
ii. FRCP 11 89

C. Truth and Falsity in Litigation 89


i. Rule 3.3 Candor Toward the Tribunal 89
ii. Rule 8.4 (c) Misconduct 90
iii. Rule 3.4 (e): Lawyer’s Duties in Trial 90
iv. Chart for Which Rules Apply When…. 90
v. A Lawyer’s Duty if a Client Intends to Give Testimony the Lawyer Knows is False Nix v.
Whiteside 91
vi. “Knowledge” of a Client’s Intent to Give False Testimony 91
vii. Can lawyers protect themselves and their clients by deliberately not knowing all the facts? 91
viii. A lawyer’s duties if a client intends to mislead the court without lying  Bronston v. United
States 91
ix. False Impressions Created by Lawyers During Litigation 91
x. Preparing Witnesses Rule 3.4(b), (f) 91

D. Concealment of Physical Evidence and Documents 92


i. Duties of criminal defense lawyers with respect to evidence of crimes 92
ii. Rule 3.4(a) A Lawyer Shall Not Unlawfully Obstruct 92
iii. Sarbanes-Oxley law  Lawyers w/Posession of Documentary Evidence 93
iv. Civil Cases Lmtd Obligation to Reveal 3.4 (a) and 3.4 (c) 93
v. Rule 3.4 (d) A lawyer’s duties in responding to discovery requests 93

E. The Duty to Disclose Adverse Legal Authority 93


i. Rule 3.3(a)(2) 93

F. Disclosures in Ex Parte Proceedings 94


i. Rule 3.3(d) 94

G. Improper Influences on Judges and Juries 94


i. Rule 3.5 Impartiality and Decorum of the Tribunal 94
ii. Ex-Parte Communications w/Judges 94
iii. Lawyer’s Comments to the Press Rule 3.6 Trial Publicity 94
iv. Impeachment of Truthful Witnesses 96
v. Statements by Lawyers During Jury Trials  Rule 3.7 Advocate Witness 96
vi. Statements by Lawyers During Jury Trials  Rule 3.4 (e) Allusion to Irrelevant or
Unsupported Matter 97

H. Lawyers’ Duties in Non-Adjudicative Proceedings 97


i. Rule 3.9 Advocate in Nonadjudicative Proceedings 97

10
ii. Application of Rule 3.9 97

98

A. Deception of Third Persons 98


i. Rule 4.1 Truthfulness In Statements To Others 98
In the course of representing a client a lawyer shall not knowingly: 98
ii. Comment [1] of Rule 4.1 Misrepresentation 98
iii. Comment [2] of Rule 4.1  Statements of Fact; duties of truthfulness in negotiation 99
iv. Comment [3] of Rule 4.1  Crime or Fraud by Client 99
v. Lawyers’ duties of truthfulness in fact investigation 99
vi. Receipt of inadvertently transmitted information, including metadata 99
vii. Rule 4.4 Respect For Rights Of Third Persons 99

B. Restrictions on Contact w/Represented Persons 100


i. Rule 4.2 Communication With Person Represented By Counsel 101
ii. Comments [2] and [3] of Rule 4.2  Applicability of the Rule 101
iii. Comment [4] of Rule 4.2  Rule Doesn’t Apply to… 101
iv. Comment [7] of Rule 4.2  Representing Businesses 101

C. Restrictions on Contact with Unrepresented Persons 101


i. Rule 4.3 Dealing With Unrepresented Person 101
ii. Respect for the Rights of Third Persons 102

D. Duties of Prosecutors 102


i. Undercover Investigations and Rules 4.1 and 4.2 102
ii. Rule 3.8 Special Responsibilities Of A Prosecutor (emphasis on (a)(d)(f)(g)(h)) 102

104

A. Unmet Need for Legal Services 104


i. General 104

B. Right to Counsel for Indigent Litigants 104


i. Criminal defendants (753) 104
ii. Rule 6.2 Accepting Appointments 104
iii. Parties in Civil and Administrative Proceedings 104

C. Civil Legal Aid 105


i. General (Legal Services Corporation) 105
ii. Conflicts of Interest for Legal Services Corporations 105
iii. Other Civil Legal Services (775) 105
iv. The IOLTA Controversy 105

D. Fee Shifting Statutes 105


i. General 105
ii. 42 USC 1983 105
iii. Fee Waiver as Term of Settlement 105
iv. Who is a “prevailing party” entitled to attorneys’ fees? 105

E. Pro-Bono Representation 105


i. Rule 6.1 Voluntary Pro Bono Public Service 105

11
ii. Background to Pro Bono Representation 106
iii. Loan Forgiveness and Scholarship for Public Service 106

F. Restrictions on Participation by Nonlawyers in Providing Legal Services 106


i. Unauthorized Practice of Law Statutes 106
ii. Forms of Unauthorized Practice 107
iii. The Prohibition of Multidisciplinary Practice 107
iv. The Prohibition of Nonlawyer Investment in Law Firms 107
v. Rule 5.4 Professional Independence of a Lawyer 107

108

A. Advertising Legal Services 108


i. Bates v. State Bar of Arizona (822) 108
ii. Rule 7.2 Advertising 109
iii. Rule 7.1 Communications Concerning A Lawyer's Services 109
iv. Rule 7.4 Communication of Fields of Practice and Specialization 109
v. Rule 7.5 Firm Names And Letterheads 109
vi. Rule 5.4 Professional Independence Of A Lawyer 110

B. Solicitation of Clients 110


i. General 110
ii. Ohralik v. Ohio 110
iii. Shapero v. Kentucky 110
iv. Rule 7.3 Solicitation of Clients 110

C. Legal Culture in Certain Practice Settings 111


i. What’s covered in the book 111
2. Small firms (863 111

D. The business of law practice in the twenty-first century (875) 111


i. Minor Things Covered in the Book 111
ii. Multistate practice: A challenge to state-based licensing (General) 112
iii. Rule 5.5 Unauthorized Practice Of Law; Multijurisdictional Practice Of Law 112

19. IN CLASS HYPOS 113


The Midnight Phone Call 113
1-1 The New Country 113
1-2 Weed 113
1-3 Doctored Resume 114
2-1 Dying Mother 114
2-2 “I’m Not Driving” 115
2-3 Exculpatory Evidence 115
2-4 The Little Hearing 116
3-1 Your Dinner w/Anna (145) 117
3-2 The Missing Persons, Scene 1*158 118
3-3 The Missing Persons Scene 2 (*163) 119
3-4 The Missing Persons Scene 3 (*169) 120
3-5 Rat Poison 120
3-6 Reese’s Leases 121
3-7 An Investment Project 122
4-1 Murder for Hire 122
4-2 A Secret Confession 123

12
4-3 Worldwide Bribery 124
5-1 The Washing Machine 125
5-2 A Desire to Investigate (273) 125
5-3 Torture *283 126
5-4 The Package Bomber (303) 127
5-5 Vinyl Windows *310 128
6-1 The Injured Passengers, Scene 1 130
6-2 Food Poisoning 130
6-3 I Thought You Were My Lawyer 130
6-4 The Injured Passengers, Scene 2 132
6-6 Top Gun (Legal Positions) 133
6-7 The Secret Affair 134
7-2 My Client’s Subsidiary 136
7-3 Police Brutality, Scene 1 137
7-4 Police Brutality, Scene 2 138
7-5 Police Brutality, Scene 3 139
7a The Leased Car 139
Florida Bar Op. 95-4 139
7-6 Representing the McCarthys 139
8-1 Keeping in Touch 142
8-2 Toxic Waste 143
8-3 A Brief Consultation 144
The Fordham Case 144
9-1 An Unreasonable Fee? 144
Scenes from a Law Firm, Questions 1, 2, and 5 145
9-3 An Impoverished Client 145
10-1 The District Attorney 146
10-2 A Trip to Monte Carlo 146
10-3 The Judge’s Former Professor 147
Professor Giller’s Hypo 147
11-2 Flight From Sudan, Scene 1 147
11-3 Flight From Sudan, Scene 2 149
OJ Simpson 150
11-4 The Drug Test 150
11-5 The Body Double 152
11-6 Refreshing Recollection 152
11-7 Child Pornography 152
11-8 A Letter to the Editor 153
Harry Subin’s Case 154
12-1 Emergency Food Stamps 154
12-3 The Break-In 155
12-4 The Prosecutor’s Masquerade 156
Legal Services for the Poor (pp 773-774) 156
6-5 The Prisoner’s Dilemma 156
13-2 Mandatory Pro Bono Services 158
13-3 Special Education 159
14-1 Do You Need a Lawyer? 160

13
A. Moral Philosophy, Right & Wrong, & the Law Governing Lawyers
1. Moral Philosophy
a. The study of professional responsibility law; Does not replace legal
analysis as tool for determining application of professional
responsibility law
2. Right and Wrong
a. More to practicing law than right & wrong; many rules governing
lawyers are counterintuitive & based on role-based morality that
attends the lawyer’s role
3. Law Governing Lawyers
a. Complicated mix of substantive law
b. I.e. Bar’s self-regulations; agency; contract; tort; procedure; evidence
laws have specific applications to lawyers
4. Role Morality
a. A choice is moral if it is consistent w/the effective execution of a role;
A lawyer who effectively carries out the lawyer’s role is moral; Role
morality supplants the generally applicable moral standards where the
two conflict; A lawyer’s continued zealous representation of the Client
is moral if it comports w/the lawyer’s role in the dispute resolution
system
5. Balance of Duties
a. There is hierarchy of duties, but q’s involve attempted balancing of
duties
b. Legal rules that govern lawyers are nearly all an attempted balance
among competing duties
c. Different standards are not really higher or lower, they merely stri ke
the balance between the competing duties in a different place
B. The Role of the Lawyer
i. Standard Conceptions of the Lawyer’s Role
1. Neutrality
a. Lawyer is separated from and not responsible for the client’s goals.
2. Procedural Justice or Role Morality
a. Lawyer must effectively carry out his role within a moral system of justice, even if
that roles requires actions that may produce consequences of questionable moral
value
3. Partisanship for Client
4. Professional, moral decision making
a. Lawyer must follow the profession’s collective judgments about the resolution of
ethical questions, regardless of whether lawyer might personally have different
judgment about appropriate resolution of the q
ii. Roles of the Lawyer
1. The Lawyer as a Businessperson
a. Sees law as public profession only to the extent the law affects the public welfare to
a greater degree than many other businesses; in a money-making trade 1st &
foremost; Competence & efficiency among highest priorities

14
2. The Moral Activist Lawyer
a. Shares a responsibility w/client for the goals and means of representation; Attempts
to persuade client to do what is morally right, not merely what is legally required or
permitted, and refrains from representation that the lawyer finds morally
objectionable
3. Differences Between Lawyers’ Litigation and Planning Roles
a. Litigation Context Backward-looking work
b. Planning Context Forward looking work to predict consequences of proposed
conduct
c. Responsibility for Client’s Acts Lawyer bears more responsibility for the client’s
acts in the planning context rather than the litigation context
4. Other Roles
a. I.e. judges and third party neutrals
iii. Practice Settings
1. Prosecutors
a. Have no specific client with whom to consult; Expected to seek justice and have
fairness obligations to opposing party that exceed those of other lawyers
2. Other government lawyer
a. Represent government or people of their jurisdiction; Obligated to reflect clients’
public-abiding obligations; Expected to be more open to public inspection than
affairs of a private client; A-C privilege much narrower
3. Criminal Defense
a. Creates unique overlaps; Must allow client to testify even if believe but do not know
will be false. MR 3.3(a)(3)
4. Corporate or Other Organization Counsel
a. Represent an entity and not the corporate officers
5. Legal Aid
a. Set up to receive government funding to represent low income individuals; Must
account for third-party interference conflicts; Operate under legislative restrictions
on their funding

A. Organization of the Bar


i. The American Bar Association
1. No direct regulatory authority
2. Views on issues affecting the law of lawyering and its model ethics codes
have tremendous influence
ii. Alternative National Bar Associations
1. National Lawyers Guild
a. Established as more liberal and progressive organization in 1936
2. National Bar Association
a. Organization for lawyers denied admission to the ABA (i.e. for
minorities)
3. Organizations of Women Lawyers
a. Promotes legal and political positions of members
4. Others

15
a. Associations organized by race, ethnicity, and sexual orientation, etc.
iii. State Bar Associations
1. Some states require membership in state bar association
2. State courts and legislatures have the power to create integrated or
mandatory bars in the states
3. A mandatory bar may not expend members’ dues to advance political or
ideological positions
B. Sources of Law Governing Lawyers: Ethics Codes
i. ABA models and their organization
1. ABA adopted series of three model ethics codes that serve as models for
state adoption, which have been chief source of model ethics codes
a. They have no legal force unless they are adopted by the relevant gov’t
authority, usually a state’s highest court
2. 1908 Cannons of Ethics
ii. 1969 Model Code of Professional Responsibility
1. 1 st effort to influence mandatory national standards for lawyer conduct
2. Basis of state codesBy 1974, 49 states adopted codes based on MRPC
3. Three types of provisions:
a. Canons Nine Cannons are broad statements of basic norms
b. Disciplinary Rules Each Cannon has disciplinary rules, which are mandatory
prescriptions “that state a minimum level of conduct below which no lawyer can fall
without being subject to disciplinary action.”
c. Ethical Considerations Meant to be aspirational but courts have used as if
mandatory @ times
iii. 1983 Model Rules of Professional Conduct
1. Drafted in response to Watergate
2. Rule and Comment Organization
a. Largely abandoned aspirational tone of Model Code in favor of “rule
and comment”
3. Rule of law treatment
a. Result has been more rule of law treatment of law of lawyering than
possible under former
4. Basis of State codes
a. 48 states have implemented model-based ethics code
5. February 2002 Revision
a. Significant amendments to MR 1.6
iv. State-Adopted Codes
1. It is the state adopted code, not the ABA rules that controls particular
jurisdiction
2. States adopt either by legislation or rule-making action by the state’s court
of last resort
C. Sources of Law Governing Lawyers: Case Authority, Ethics
Opinions, Restatement, Constitutional Constraints, “Other Law”
i. Case Authority
1. Interpretation of Codes
16
a. Courts interpret ethic codes similar to how they do with statute
interpretation
2. Inherent Power to Regulate Lawyers
a. Common law of lawyer regulation also exists
ii. Ethics Opinions
1. Both ABA and bar associations issue nonbinding ethics opinions, which
are often relied on by courts in law-of-lawyering cases
2. Normally given in hypothetical scenarios
iii. Restatement of Law Governing Lawyers
iv. Constitutional Constraints
1. Commercial Speech
a. Lawyer’s advertising and solicitation of clients is afforded considerable
First Amendment protection
2. Speech Rights of Lawyers
a. Sometimes called on to speak publicly for clients and First
Amendment provides some protection from regulation
3. Entry to the Bar
a. States efforts to impose residency requirements for entry to practice is
unconstitutional
v. “Other Law”
1. Contracts
2. Torts
a. lawyer liability for malpractice and third parties for intentional
wrongful acts, and in some cases, incompetence
3. Fiduciary Law
a. Governs the special relationship and duties of lawyers to clients and
certain other beneficiaries of lawyer and client acts
4. Agency
a. Agency law governs and informs much of the relationship between
lawyer and client with the lawyer acting as the client’s agent
5. Criminal Law
a. Governs lawyer criminal liability for possible liability as an accomplice
or a conspirator with a client engaged in criminal conduct
6. Procedural Law
a. Criminal and civil procedural law governs lawyer conduct, particularly
in litigation contexts
7. Antitrust
8. Administrative Regulations
a. Includes tax, banking, securities, environmental, and occupational
safety regulations
9. Employment Law
D. Admission to the Bar
i. Model Rule 8.1 Bar Admission and Disciplinary Matters

17
1. An applicant for admission to the bar, or a lawyer in connection with a bar
admission application or in connection with a disciplinary matter, shall
not:
a. (a) knowingly make a false statement of material fact; or
b. (b) fail to disclose a fact necessary to correct a misapprehension
known by the person to have arisen in the matter, or knowingly fail to
respond to a lawful demand for information from an admissions or
disciplinary authority, except that this rule does not require disclosure
of information otherwise protected by Rule 1.6.
ii. Bar Admission Criteria at Issue and Citation
1. Manslaughter
a. Defendant used chloroform to subdue someone and that killed them.
The defendant pled guilty to manslaughter
b. The court granted admission after finding the defendant adequately
rehabilitated themselves.
2. Sexual Relations w/Minors
a. Admission denied to teacher who slept w/14 year old
3. Shoplifting and misrepresentation of debt
a. Admission granted for proving sound moral character after the fact
4. Declaration of bankruptcy
a. Admission granted
5. Pattern of hostile and disruptive conduct
a. Harassed dean w/nude photos
b. Admission denied
6. Cheating on law school exam
a. Admission denied because student lacked sufficient moral character
7. Racist opinions
a. Admission denied
8. Criticism of the bar
a. Admission denied because the applicant had “already written off a
huge component of the justice system”
9. Posing naked in a playboy
a. Admission granted
iii. Mental Health of Applicants
1. Should bar admissions authorities ask questions about the mental health
of applicants?
a. National Conference of Bar Examiners conducts the character and
fitness evaluation for a majority of states
b. Some states have created systems through which applicants with
histories of emotional trouble and substance abuse can be admitted to
the bar conditionally for a probationary period, during which they
must comply with specified conditions such as participating in mental
health care
iv. Law School Discipline: A Preliminary Screening Process

18
1. The bar examiners often ask applicants to disclose any sanctions posed by
a law school, whether they were considered to be “confidential” or not
2. The law school disciplinary system tends to be structured like the
microcosms of the lawyer disciplinary system
3. The law schools perform a prescreening process for the bar examiners
with respect to students who engage in misconduct while in law school

A. Professional Discipline
1. Violation of the rules may lead to disbarment, suspension, a public or
private reprimand, or another sanction
2. It is a central element in the state supreme courts’ efforts to require high
standards of professional conduct
B. Discipline Process
i. How does a disciplinary proceeding work?
1. In most states, the highest court runs the disciplinary system
2. Some are run by the state bars but most are independent from the bar
association
3. A complaint is issued against a lawyer, if there is merit to the complaint
then it is brought before an adjudicator (depending on the disciplinary
agency this can be a third party volunteer, lawyer, judge, etc.)
4. The adjudicator collects and hears evidence, makes findings of fact, and
recommends sanctions
5. The recommendations are then review by an admin board, the boards
decisions can be appealed to the highest court
ii. Do the lawyer disciplinary agencies investigate most of the complaints?
1. Probably not
2. Most cases are dismissed
3. Disciplinary systems are still problematic
4. Imposition of sanctions is inconsistent
5. Most of the people disciplined are solo practitioners
6. Formal discipline is disproportionately imposed on minorities
7. Most complaints come from clients
8. The disciplinary systems aren’t well funded
iii. Other Questions
1. Are some states making significant improvements in their disciplinary
systems?
a. Yes – many states are engaged in an ongoing process of review and
implementation of improvements in their disciplinary systems
2. If the disciplinary agencies tend to under-enforce the rules, does that
mean that lawyers need not worry about compliance with ethics rules?
19
a. No
b. Even being accused of an ethical violation can be damaging
c. Noncompliance can still be the basis for legal malpractice liability if
the violation causes harm to a client or third party
d. Noncompliance could have employment consequences
C. Grounds for Discipline
i. What kinds of professional conduct can result in discipline?
1. The most common conduct is the misappropriation of client funds,
commingling law firm and client funds, missing court filing deadlines,
failing to respond to client communications, committing mail fraud and
tax evasion, and neglecting client cases
2. Can also be for non-criminal activity like publicly criticizing the court
ii. Can a lawyer be disciplined for advising a client about proposed conduct
that may be criminal or fraudulent?
1. It depends
2. A lawyer may advise a client who wants to know whether a possible
course of action is lawful, but a lawyer may be disciplined if the lawyer
guides a client as to how to violate the law or helps the client to engage in
conduct that is criminal or fraudulent – see 1.2(d)
iii. Model Rule 1.2(d)
1. (d) A lawyer shall not counsel a client to engage, or assist a client, in
conduct that the lawyer knows is criminal or fraudulent, but a lawyer
may discuss the legal consequences of any proposed course of conduct
with a client and may counsel or assist a client to make a good faith effort
to determine the validity, scope, meaning or application of the law.
a. [Comment 10] When the client's course of action has already begun
and is continuing, the lawyer's responsibility is especially delicate. The
lawyer is required to avoid assisting the client, for example, by drafting
or delivering documents that the lawyer knows are fraudulent or by
suggesting how the wrongdoing might be concealed. A lawyer may not
continue assisting a client in conduct that the lawyer originally
supposed was legally proper but then discovers is criminal or
fraudulent. The lawyer must, therefore, withdraw from the
representation of the client in the matter. See Rule 1.16(a). In some
cases, withdrawal alone might be insufficient. It may be necessary for
the lawyer to give notice of the fact of withdrawal and to disaffirm any
opinion, document, affirmation or the like. See Rule 4.1.
b. The restatement states that except for decisions reserved for clients
and in the absence of an agreement on these matters, a lawyer may
take “any lawful measure within the scope of representation that is
reasonably calculated to advance a client’s objective.”
c.
iv. Model Rule 1.0(d) “Fraud” or “Fraudulent”
1. (d) "Fraud" or "fraudulent" denotes conduct that is fraudulent under
the substantive or procedural law of the applicable jurisdiction and
has a purpose to deceive
20
v. Model Rule 1.0(f) “Knowingly,” “Known,” or “Knows
1. (f) "Knowingly," "known," or "knows" denotes actual knowledge of the
fact in question. A person's knowledge may be inferred from
circumstances.
vi. Model Rule 1.0(i) “Reasonable Belief” or “Reasonably Believes”
1. (i) "Reasonable belief" or "reasonably believes" when used in reference to
a lawyer denotes that the lawyer believes the matter in question and that
the circumstances are such that the belief is reasonable.
vii. Under what circumstances might a lawyer commit or assist a fraud by
failing to state a fact (omission) or by telling a half-truth?
1. Under the Model Rules, a lawyer’s omission may be fraudulent if the
lawyer intended to deceive another person
a. Rule 4.1(b) bars a lawyer from knowingly failing to disclose a non -
confidential material fact when disclosure is necessary to avoid
assisting a client’s fraudulent act
b. Comment 1 after 4.1 explains that misrepresentations can also occur
by partially true but misleading statements or omissions that are the
equivalent affirmative false statements
viii. Can lawyers be disciplined for conduct that has nothing to do with the
practice of law?
1. Yes – a lawyer may be disciplined for violation of the applicable ethics
codes whether or not the violation occurs in the course of law practice
2. Most ethical rules impose requirements of conduct of law practice
a. Ex: domestic violence, failure to pay child support, drunk driving
ix. Can a lawyer be disciplined for committing a crime?
1. Yes, A lawyer may be disciplined for the commission of any criminal act
that violates an ethical rule or that reflect dishonesty, untrustworthiness,
or lack of fitness to practice
x. What if a lawyer has not been convicted or charged with the crime?
1. A lawyer may be disciplined for committing a criminal act even if no
criminal charge is filed or the lawyer is acquitted of a charge in a criminal
proceeding
2. However, if a disciplinary action is filed based on conduct that is the
subject of a pending criminal charge, the disciplinary action usually is
stayed until the criminal proceeding is concluded
xi. Other Questions
1. Can a lawyer be disciplined based on the actions of an employee ?
a. Yes - A lawyer may be disciplined for inducing or assisting another
person to do something that would violate the rules if done by a
lawyer
2. Can a lawyer be disciplined for something they do outside the state in
which they are licensed to practice?
a. Yes – the lawyer may be disciplined regardless or whether the violation
occurs outside of the state

21
3. Can a lawyer be disciplined by the authorities of a state in which they are
not licensed?
a. It depends
b. Most say no, but California and DC will discipline any lawyer who
violates a rule of the jurisdiction
4. What if a lawyer is admitted to practice in several states but is suspended
or disbarred in one of those states? Can the lawyer continue their practice
in the other states?
a. They could until the other states impose reciprocal or other discipline
5. Can a lawyer be disciplined for engaging in discriminatory behavior?
a. It is not explicitly prohibited in the Model Rules
b. Other states say it’s a violation of 8.
xii. Model Rule 8.4(c) Misconduct
1. It is professional misconduct for a lawyer to:
(a) violate or attempt to violate the Rules of Professional Conduct,
knowingly assist or induce another to do so, or do so through the acts of
another;
(b) commit a criminal act that reflects adversely on the lawyer's honesty,
trustworthiness or fitness as a lawyer in other respects;
(c) engage in conduct involving dishonesty, fraud, deceit or
misrepresentation;
(d) engage in conduct that is prejudicial to the administration of
justice;
(e) state or imply an ability to influence improperly a government agency
or official or to achieve results by means that violate the Rules of
Professional Conduct or other law;
(f) knowingly assist a judge or judicial officer in conduct that is a
violation of applicable rules of judicial conduct or other law; or
(g) engage in conduct that the lawyer knows or reasonably should know
is harassment or discrimination on the basis of race, sex, religion,
national origin, ethnicity, disability, age, sexual orientation, gender
identity, marital status or socioeconomic status in conduct related to the
practice of law. This paragraph does not limit the ability of a lawyer to
accept, decline or withdraw from a representation in accordance with
Rule 1.16. This paragraph does not preclude legitimate advice or
advocacy consistent with these Rules.
2. See In re Riehlmann in Hypo 2-3
D. Reporting Misconduct of Other Lawyers
i. Model Rule 8.3 Reporting Professional Misconduct
1. (a) A lawyer who knows that another lawyer has committed a violation of
the Rules of Professional Conduct that raises a substantial question as
to that lawyer's honesty, trustworthiness or fitness as a lawyer in other
respects, shall inform the appropriate professional authority.
2. (b) A lawyer who knows that a judge has committed a violation of
applicable rules of judicial conduct that raises a substantial question as to
the judge's fitness for office shall inform the appropriate authority.

22
3. (c) This Rule does not require disclosure of information otherwise
protected by Rule 1.6 or information gained by a lawyer or judge while
participating in an approved lawyers assistance program.
ii. Standards and Application of Model Rule 8.3
1. See In re Riehlmann in Hypo 2-3
2. The standard for assessing knowledge is objective
a. Knowledge must be more than mere suspicion that misconduct
occurred
b. The question is whether a “reasonable lawyer in the circumstances
would have a firm opinion that the conduct in question is more
likely than not to have occurred”
3. Substantial question
a. is ambiguous
b. counter-argument: you’re working in an adversarial system, so don’t
want to open the door to reports on every little violation
4. Must blow the whistle on a boss if they do something unethical
a. A lawyer cannot get off the hook by informing senior lawyers in a firm
about the misconduct of another lawyer
5. Not required to report:
a. Information protected by the confidentiality rules
b. Information learned while participating in a lawyer’s assistance
program
6. No Approval Required
a. Simply shields lawyers from reporting confidential client info
7. Failure to Report May = Discipline
E. Responsibility for Ethical Misconduct by Colleagues and Superiors
i. Model Rule 5.1: Responsibilities Of Partners, Managers, & Supervisory
Lawyers
1. Summary: establishes the responsibility of a partner or supervising lawyer
for ensuring compliance with the ethical rules by subordinate lawyers, and
explains when a senior lawyer may be subject to discipline for the conduct
of a subordinate lawyer
2. Rule:
(a) A partner in a law firm, and a lawyer who individually or together with
other lawyers possesses comparable managerial authority in a law firm,
shall make reasonable efforts to ensure that the firm has in effective
measures giving reasonable assurance that all lawyers in the firm conform
to the Rules of Professional Conduct.
(b) A lawyer having direct supervisory authority over another lawyer shall
make reasonable efforts to ensure that the other lawyer conforms to the
Rules of Professional Conduct.
(c) A lawyer shall be responsible for another lawyer's violation of the
Rules of Professional Conduct if:
(1) the lawyer orders or, with knowledge of the specific conduct, ratifies
the conduct involved; or

23
(2) the lawyer is a partner or has comparable managerial authority in the
law firm in which the other lawyer practices, or has direct supervisory
authority over the other lawyer, and knows of the conduct at a time
when its consequences can be avoided or mitigated but fails to take
reasonable remedial action.
ii. Model Rule 5.2: Responsibilities of A Subordinate Lawyer
1. Summary: states when a subordinate lawyer is responsible for her own
conduct, and under what circumstances she may follow orders without
fear of discipline
2. Rule:
(a) A lawyer is bound by the Rules of Professional Conduct
notwithstanding that the lawyer acted at the direction of another person.
(b) A subordinate lawyer does not violate the Rules of Professional
Conduct if that lawyer acts in accordance with a supervisory lawyer's
reasonable resolution of an arguable question of professional duty.
3. Implications:
Under rule 5.2(b), subordinate lawyers may be held accountable for
unethical action they were ordered to undertake if the supervisor’s
instruction was not based on a “reasonable resolution of an arguable
question of professional duty”
4. Options for a lawyer told to do something they think is unethical
Accept the directs of the superior
Argue with the superior
Discuss the problems with another superior
Do more research or investigation to try to clarify the problem
Ask to be relieved from work on the matter
Resign (or be fired) from employment nder rule 5.2(b), subordinate
lawyers may be held accountable for unethical action they were ordered to
undertake if the supervisor’s instruction was not based on a “reasonable
resolution of an arguable question of professional duty”
iii. Model Rule 5.3: Responsibilities Regarding Nonlawyer Assistance
1. Summary: explains the responsibility of lawyers who supervise nonlawyer
employees for ensuring that the employees comply with the rules of
professional conduct and explains when a lawyer may be subject to
discipline based on the conduct of a nonlawyer employee
2. Rule:
With respect to a nonlawyer employed or retained by or associated with a
lawyer:
(a) a partner, and a lawyer who individually or together with other lawyers
possesses comparable managerial authority in a law firm shall make
reasonable efforts to ensure that the firm has in effect measures giving
reasonable assurance that the person's conduct is compatible with the
professional obligations of the lawyer;
(b) a lawyer having direct supervisory authority over the nonlawyer shall
make reasonable efforts to ensure that the person's conduct is compatible
with the professional obligations of the lawyer; and

24
(c) a lawyer shall be responsible for conduct of such a person that would
be a violation of the Rules of Professional Conduct if engaged in by a
lawyer if:
(1) the lawyer orders or, with the knowledge of the specific conduct,
ratifies the conduct involved; or
(2) the lawyer is a partner or has comparable managerial authority in the
law firm in which the person is employed, or has direct supervisory
authority over the person, and knows of the conduct at a time when its
consequences can be avoided or mitigated but fails to take reasonable
remedial action.
iv. Can discipline be imposed on a whole law firm?
1. In most states, only an individual lawyer may be disciplined for violation
of ethical rules
2. NY and NJ have rules that allow imposition of discipline on law firms for
certain misconduct, such as failure to supervise employees or failure to
maintain a system for checking new matters to identify conflicts of
interest
v. Can a law student get in trouble for violating an ethics rule?
1. Law students are subject to professional discipline for violating the ethics
rules, except for 8.1, which governs disclosure by applicants for admission
to the bar.
vi. Does the law provide any protection for lawyers who are fired because they
refuse to participate in unethical conduct or because they report
misconduct of other lawyers to the disciplinary authorities?
1. The NY Court of Appeals held that the right to fire employees at will did
not include the right to fire lawyers who complained of unethical behavior
by another lawyer, because lawyers have professional duties not only to
their employers, but to a broader public

A. What Clients Can Sue For


1. Negligence
2. Intentional Misconduct
3. Breach of Contract
4. Breach of Fiduciary Duty
B. Legal Malpractice
i. General
1. Refers to a claim brought against a lawyer for professional misconduct
that is alleged to have caused harm to another person
2. A single claim may assert more than one theory of liability
3. Legal malpractice is an umbrella term that covers all of the
aforementioned actions
25
ii. Elements of a tort claim of Legal Malpractice
1. The client must assert:
a. The lawyer owed a duty to the plaintiff
b. The lawyer failed to exercise exercise “the competence and diligence
normally exercised by lawyer in in similar circumstances”
c. The breach of duty caused harm to the plaintiff
iii. Most common mistakes made by lawyers that lead to malpractice liability
1. Ignore conflicts of interest
2. Sue your former client for an unpaid fee
3. Accept any client and any matter that comes along
4. Do business with your client
5. Practice outside your area of expertise
6. Go overboard in opening branch offices and making lateral hires
7. Leave partner peer review to the other firms
8. Ignore a potential claim and represent yourself in a professional liability
dispute
9. Settle a matter without written authorization from your client
10. Fail to communicate with your client
iv. Available Remedies
1. Money damages
2. Ordered compliance w/an injunction
3. Return of property
4. Alteration or cancellation of a legal document
v. Fiduciary Duties Owed by a Lawyer to a Client
1. Safeguarding client’s confidences and property
2. Avoiding impermissible conflicting interests
3. Adequately informing the client
4. Following instructions of the client
5. Not employing powers arising from the client-lawyer relationship
adversely to the client
vi. Elements of a Claim of Breach of Fiduciary Duty
1. Plaintiff must prove:
a. But for the lawyer’s conduct, the ∏ would have obtained a favorable
judgment or settlement in the case in which the lawyer originally
misrepresented the client or that the ∏ suffered some other
compensable harm
vii. Implications of one act of misconduct
1. A lawyer could be disciplined, sued for malpractice, and criminally
prosecuted, all at once
viii. Liability to a non-client
1. A lawyer could be liable to a non-client including: prospective clients and
people who are the intended beneficiaries of the lawyer’s work for a client,
such as those who will inherit assets under a will drafted by a lawyer

26
ix. Malpractice Insurance
1. Lawyers aren’t required to carry
2. Some states require lawyers to disclose if they have malpractice insurance
(to incentivize lawyers to carry insurance)
3. Malpractice insurance policy is unlikely to cover:
a. Intentional acts, including fraud or other dishonest conduct
b. Orders of restitution of legal fees, fines, or penalties
c. Orders to pay punitive damages
d. Conduct of lawyers in other roles, such as notary public, title agent, or
trustee
e. Conduct of lawyers who represent businesses of which they are also
part-owners
f. Intra-firm disputes, such as liability to former partners or former
employees
x. Other Civil Liability of Lawyers
1. Advising or assisting unlawful client conduct
2. Stealing
3. Lying
4. Intentional infliction of emotional distress
5. Violation of regulatory statutes or
6. Breach of contract

A. General
1. A lawyer who knowingly provides services for criminal activity can be held
criminally liable
2. Any criminal action by the lawyer is criminally liable
B. Rule 3.8 Special Responsibilities of a Prosecutor
i. The prosecutor in a criminal case shall:
(a) refrain from prosecuting a charge that the prosecutor knows is not supported by
probable cause;
(b) make reasonable efforts to assure that the accused has been advised of the right to, and
the procedure for obtaining, counsel and has been given reasonable opportunity to obtain
counsel;
(c) not seek to obtain from an unrepresented accused a waiver of important pretrial rights,
such as the right to a preliminary hearing;
(d) make timely disclosure to the defense of all evidence or information known to the
prosecutor that tends to negate the guilt of the accused or mitigates the offense, and, in
connection with sentencing, disclose to the defense and to the tribunal all unprivileged
mitigating information known to the prosecutor, except when the prosecutor is relieved of
this responsibility by a protective order of the tribunal;

27
(e) not subpoena a lawyer in a grand jury or other criminal proceeding to present evidence
about a past or present client unless the prosecutor reasonably believes:
(1) the information sought is not protected from disclosure by any applicable
privilege;
(2) the evidence sought is essential to the successful completion of an ongoing
investigation or prosecution; and
(3) there is no other feasible alternative to obtain the information;
(f) except for statements that are necessary to inform the public of the nature and extent of
the prosecutor's action and that serve a legitimate law enforcement purpose, refrain from
making extrajudicial comments that have a substantial likelihood of heightening public
condemnation of the accused and exercise reasonable care to prevent investigators, law
enforcement personnel, employees or other persons assisting or associated with the
prosecutor in a criminal case from making an extrajudicial statement that the prosecutor
would be prohibited from making under Rule 3.6 or this Rule.
(g) When a prosecutor knows of new, credible and material evidence creating a reasonable
likelihood that a convicted defendant did not commit an offense of which the defendant was
convicted, the prosecutor shall:
(1) promptly disclose that evidence to an appropriate court or authority, and
(2) if the conviction was obtained in the prosecutor’s jurisdiction,
(i) promptly disclose that evidence to the defendant unless a court
authorizes delay, and
(ii) undertake further investigation, or make reasonable efforts to cause an
investigation, to determine whether the defendant was convicted of an
offense that the defendant did not commit.
(h) When a prosecutor knows of clear and convincing evidence establishing that a defendant
in the prosecutor’s jurisdiction was convicted of an offense that the defendant did not
commit, the prosecutor shall seek to remedy the conviction
C. Client Protection Funds
1. See book if question comes up

A. Basic Principle
1. One basic rules of client representation is that lawyers are obliged to keep
clients’ secrets
2. Protection of “information relating to the representation of a client”
B. Rule 1.6 Confidentiality of Information
i. Rule 1.6
1. (a) A lawyer shall not reveal information relating to the representation of a
client unless the client gives informed consent, the disclosure is impliedly
authorized in order to carry out the representation or the disclosure is
permitted by paragraph (b).
2. (b) A lawyer may reveal information relating to the representation of a
client to the extent the lawyer reasonably believes necessary:
a. (1) to prevent reasonably certain death or substantial bodily harm;
b. (2) to prevent the client from committing a crime or fraud that is
reasonably certain to result in substantial injury to the financial

28
interests or property of another and in furtherance of which the client
has used or is using the lawyer's services;
c. (3) to prevent, mitigate or rectify substantial injury to the financial
interests or property of another that is reasonably certain to result or
has resulted from the client's commission of a crime or fraud and in
furtherance of which the client has used the lawyer's services;
d. (4) to secure legal advice about the lawyer's compliance with these
Rules;
e. (5) to establish a claim or defense on behalf of the lawyer in a
controversy between the lawyer and the client, to establish a defense
to a criminal charge or civil claim against the lawyer based upon
conduct in which the client was involved, or to respond to allegations
in any proceeding concerning the lawyer's representation of the client;
f. (6) to comply with other law or a court order; or
g. (7) to detect and resolve conflicts of interest arising from the lawyer’s
change of employment or from changes in the composition or
ownership of a firm, but only if the revealed information would not
compromise the attorney-client privilege or otherwise prejudice the
client.
3. (c) A lawyer shall make reasonable efforts to prevent the inadvertent or
unauthorized disclosure of, or unauthorized access to, information
relating to the representation of a client.
ii. Comment [4] of Model Rule 1.6: Paragraph (a)
1. [4] Paragraph (a) prohibits a lawyer from revealing information relating to
the representation of a client. This prohibition also applies to disclosures
by a lawyer that do not in themselves reveal protected information but
could reasonably lead to the discovery of such information by a third
person. A lawyer's use of a hypothetical to discuss issues relating to the
representation is permissible so long as there is no reasonable
likelihood that the listener will be able to ascertain the identity of the
client or the situation involved.
a. ALSO (not in comment): “To honor the rule of confidentiality,
however, and to maintain its strength, lawyers should exercise self-
restrained and resolve marginal cases in favor of nondisclosure”
iii. Comment [18] of Model Rule 1.6: Paragraph (c)
1. There is no violation if the lawyer makes reasonable prevention efforts
2. Factors that should be considered include:
a. The sensitivity of the information
b. The likelihood of disclosure in the absence of additional safeguards
c. The cost of additional safeguards
d. The difficulty of implementing such safeguards
e. The extent to which the safeguards adversely affect the lawyer’s ability
to represent clients
C. Implications of Confidentiality Duties
i. Information that must be protected as confidential

29
1. All information relating to the matter on which the lawyer is representing
the client, except information that is “generally known”
2. Personal information relating to the client that the client would not want
disclosed
3. Information learned from the client and information learned from
interviews. Documents, photographs, observation, or other sources
4. Information relating to the representation acquired before representation
begins and after representation terminates
5. Notes or memoranda that the lawyer creates relating to the matter
ii. Lawyer’s duty to take precautions
1. Lawyers must also take care to protect such information from inadvertent
or unauthorized disclosure
iii. Consequences of an attorney’s failure to protect confidences
1. May be subject to professional discipline;
2. Liable in tort or contract for negligent or intentional breach of duty;
3. Disqualified from representation of one or more clients;
4. OR Enjoined by the court from further revelation
D. Protection of Info if there is a Reasonable Prospect of Harm to Client Interests
i. Adverse Effects
1. Includes frustration of the client’s objectives in the representation,
material misfortune, disadvantage, or other prejudice to the client,
financial or physical harm to the client
ii. How to know if there is a “reasonable prospect” of harm to client interests?
1. Use best judgement
E. Revelation of Past Criminal Conduct General Rule
1. A client who has committed a crime needs legal advice, and the best legal
advice comes from being told the whole truth, which means clients need
to feel confident their lawyer won’t spill the beans
2. But see Client Fraud and Crimes Causing Financial Harm
F. Exceptions to the Duty to Protect Confidences
i. Extent to which Exceptions Permit Disclosure (Comment 16)
1. Comment [16] states that paragraph (b) permits disclosure only to the
extent the lawyer reasonably believes the disclosure is necessary to
accomplish one of the purposes specified. Where practicable, the lawyer
should first seek to persuade the client to take suitable action to obviate
the need for disclosure. In any case, a disclosure adverse to the client's
interest should be no greater than the lawyer reasonably believes necessary
to accomplish the purpose. If the disclosure will be made in connection
with a judicial proceeding, the disclosure should be made in a manner that
limits access to the information to the tribunal or other persons having a
need to know it and appropriate protective orders or other arrangements
should be sought by the lawyer to the fullest extent practicable.
ii. When is revelation allowed in relation to stage of proceedings?

30
1. The lawyer need not wait for formal proceedings to be instituted but may
reveal information to prevent such action
iii. How much can a lawyer reveal if authorized?
1. No more than necessary to vindicate the lawyer. The lawyer should
minimize the number of people who learn the confidential information
revealed, should seek a prospective order, and should take other available
steps to avoid the dissemination of the information
iv. Should the lawyer inform the client before revealing confidential
information?
1. Yes, the lawyer should notify the client before using confidential
information in self-defense and should seek solutions that do not require
the lawyer to make the revelation, but the lawyer may use the information
even if the client does not consent
v. Exception: Client Consent 1.6(a)
1. A client may consent to a lawyer’s revelation of confidence but only to the
extent that the lawyer has given the client full information about the
potential risks
vi. Exception: Risk of Future Injury or Death  1.6 Comment 6: Paragraph
(b)(1) and Spaulding v. Zimmerman
1. Although the public interest is usually best served by a strict rule requiring
lawyers to preserve the confidentiality of information relating to the
representation of their clients, the confidentiality rule is subject to limited
exceptions. Paragraph (b)(1) recognizes the overriding value of life and
physical integrity and permits disclosure reasonably necessary to
prevent reasonably certain death or substantial bodily harm. Such
harm is reasonably certain to occur if it will be suffered imminently or if
there is a present and substantial threat that a person will suffer such
harm at a later date if the lawyer fails to take action necessary to
eliminate the threat. Thus, a lawyer who knows that a client has
accidentally discharged toxic waste into a town's water supply may reveal
this information to the authorities if there is a present and substantial risk
that a person who drinks the water will contract a life-threatening or
debilitating disease and the lawyer's disclosure is necessary to eliminate the
threat or reduce the number of victims.
2. Predecessor to this rule was Spaulding v. Zimmerman
a. See notes for case details
vii. Exception: Client Fraud and Crimes Causing Financial Harm 1.6(b)(2),
(b)(3)
1. A lawyer may reveal a client’s frauds and financial crimes to prevent,
mitigate, or remedy harm to others
2. Example of Scenario
a. A lawyer helps a client to prepare documents that include false
information, not knowing that the information is false
b. The recipients of the documents rely on the statements in the
documents and on the fact that the documents were prepared in part
by a reputable lawyer or law firm

31
c. The lawyer later discovers that the documents include false statements
or that the client plans to use or has used the documents in a
misleading way
3. Applicability of subsection (b)(2)
a. Applies if the client plans to commit or is committing the crime or
fraud
4. Applicability of subsection (b)(3)
a. Refers to a past crime of fraud
5. A lawyer may reveal client criminal or fraudulent conduct whether it is
past, ongoing, or future IF:
a. There is a reasonable certainty that the client’s conduct will result
in substantial financial injury or substantial injury to the property of
another person
b. The client is using or has used the lawyer’s services in committing
the acts
c. The purpose of revealing confidence is to prevent the criminal or
fraudulent act or to prevent, mitigate, or rectify the harm resulting
from the acts
6. If the criminal or fraudulent conduct is past, the client did not use the
lawyer’s services to assist in that conduct, and the client has hired the
lawyer for representation relating to the conduct, the lawyer may not
reveal information under Rule 1.6(b)(3)
viii. Exception: Revealing Confidences to Obtain Advice about Legal Ethics
1.6(b)(4)
1. 1.6(b)(4) permits a layer to reveal confidences to the extent necessary for
the lawyer to obtain advice about complying with the rules of professional
conduct
ix. Exception: Using a Client’s Confidential Information to Protect Lawyer
1.6(b)(5)
1. 1.6(b)(5) allows lawyers to reveal confidential information to the extent
necessary to authenticate claim for legal fees, to defend herself against any
civil claim or any criminal charge that involves the lawyer’s work on behalf
of a client, or to respond to any allegations that the lawyer has engaged in
professional misconduct
x. Exception: Rule 3.3 Candor Toward Tribunal
(a) A lawyer shall not knowingly:
(1) make a false statement of fact or law to a tribunal or fail to correct
a false statement of material fact or law previously made to the
tribunal by the lawyer;
(2) fail to disclose to the tribunal legal authority in the controlling
jurisdiction known to the lawyer to be directly adverse to the position
of the client and not disclosed by opposing counsel; or
(3) offer evidence that the lawyer knows to be false. If a lawyer, the
lawyer’s client, or a witness called by the lawyer, has offered material
evidence and the lawyer comes to know of its falsity, the lawyer shall
take reasonable remedial measures, including, if necessary, disclosure
32
to the tribunal. A lawyer may refuse to offer evidence, other than the
testimony of a defendant in a criminal matter, that the lawyer
reasonably believes is false.
(b) A lawyer who represents a client in an adjudicative proceeding and
who knows that a person intends to engage, is engaging or has engaged in
criminal or fraudulent conduct related to the proceeding shall take
reasonable remedial measures, including, if necessary, disclosure to the
tribunal.
(c) The duties stated in paragraphs (a) and (b) continue to the conclusion
of the proceeding, and apply even if compliance requires disclosure of
information otherwise protected by Rule 1.6.
(d) In an ex parte proceeding, a lawyer shall inform the tribunal of all
material facts known to the lawyer that will enable the tribunal to make an
informed decision, whether or not the facts are adverse.
xi. Exception: Rule 1.16 Declining or Terminating Representation
1. (a) Except as stated in paragraph (c), a lawyer shall not represent a client
or, where representation has commenced, shall withdraw from the
representation of a client if:
(1) the representation will result in violation of the rules of
professional conduct or other law;
(2) the lawyer's physical or mental condition materially impairs the
lawyer's ability to represent the client; or
(3) the lawyer is discharged.
(b) Except as stated in paragraph (c), a lawyer may withdraw from
representing a client if:
(1) withdrawal can be accomplished without material adverse effect
on the interests of the client;
(2) the client persists in a course of action involving the lawyer's
services that the lawyer reasonably believes is criminal or fraudulent;
(3) the client has used the lawyer's services to perpetrate a crime or
fraud;
(4) the client insists upon taking action that the lawyer considers
repugnant or with which the lawyer has a fundamental disagreement;
(5) the client fails substantially to fulfill an obligation to the lawyer
regarding the lawyer's services and has been given reasonable warning
that the lawyer will withdraw unless the obligation is fulfilled;
(6) the representation will result in an unreasonable financial burden
on the lawyer or has been rendered unreasonably difficult by the
client; or
(7) other good cause for withdrawal exists.
(c) A lawyer must comply with applicable law requiring notice to or
permission of a tribunal when terminating a representation. When
ordered to do so by a tribunal, a lawyer shall continue representation
notwithstanding good cause for terminating the representation.
(d) Upon termination of representation, a lawyer shall take steps to the
extent reasonably practicable to protect a client's interests, such as

33
giving reasonable notice to the client, allowing time for employment of
other counsel, surrendering papers and property to which the client is
entitled and refunding any advance payment of fee or expense that has not
been earned or incurred. The lawyer may retain papers relating to the
client to the extent permitted by other law.
xii. Exception: Rule 4.1 Truthfulness in Statements to Others
1. In the course of representing a client a lawyer shall not knowingly:
(a) make a false statement of material fact or law to a third person; or
(b) fail to disclose a material fact to a third person when disclosure is
necessary to avoid assisting a criminal or fraudulent act by a client,
unless disclosure is prohibited by Rule 1.6.
xiii. Exception: Rule 8.4(c) Misconduct
It is professional misconduct for a lawyer to:
(c) engage in conduct involving dishonesty, fraud, deceit or
misrepresentation
xiv. Conditions in which a lawyer may reveal confidential information in self-
defense
1. To establish a claim against a client for unpaid fees
2. To defend against a malpractice or other claim of civil liability against the
lawyer
3. To defend against a disciplinary proceeding
4. To defend against a criminal charge
xv. Revealing Confidence to Comply w/a Court Order or Other Law
1. Rule 1.6(b)(6) permits a lawyer to disclose confidential information to
comply with a court order or with other law
xvi. Revealing Confidences to Prevent Certain Conflicts of Interests
1. When a lawyer moves to another firm or when a lawyer moves to another
firm or when firms merge, it becomes necessary for the firm to check for
conflicts
xvii. Summary of Exceptions in Rule 1.6
1. Client waives confidentiality, in which case disclosure is permitted if client
gives informed consent
2. Disclosure necessary for representation, such that it is “impliedly
authorized” to carry out the representation
3. Past physical harms to people are not permissible
4. Threatened physical harms to people, in which case disclosure is permitted
ot prevent “reasonably certain” death or substantial bodily harm
5. Threatened or Continuing Client Fraud or Other Economic Crime, in
which case 1.6(b)(2) permits and 4.1(b) requires if conditions are met and
revelation is necessary to avoid assisting a criminal or fraudulent act
6. Past client Fraud or Other Economic Crime may be disclosed if
conditions of 1.6(b)(3) are met
7. Ethics Advice from Another Lawyer, if conditions of 1.6(b)(5) are met
8. Self-defense for fee collection or misconduct charge if conditions of
1.6(b)(6) are met

34
9. Court ordered disclosure, if 1.6(b)(6) conditions are met
10. Avoidance of conflict of interest, if 1.6(b)(7) conditions are met
11. Client provides materially false testimony to a court
12. Client is a biz organization that has committed certain frauds, in which
refer to 1.6(b)(2), 1.6(b)(3), 1.13, and Sarbanes-Oxley Act

A. Confidentiality v. Attorney-Client Privilege


i. General
1. Both confidentiality and privilege are based on the idea that a legal system
in which advocates speak for clients will work best if clients feel free to
speak openly to their attorneys
2. Duty to protect confidences is imposed by ethics rules, violation of which
can result in disciplinary action
3. Privilege is part of the law of evidence that which governs what kinds of
evidence can be admitted in court
ii. Difference in Scope
1. The duty of confidentiality is very broad
2. Privilege covers only a relatively small part of that information: the part
that involves communications between lawyer and client in which the
client is seeking legal advice or other legal services
3. In other words, the duty to protect confidences requires protection of
nearly all information that is privileged and a great deal of additional
information
4. Information covered by the privilege is only a subset of the confidential
information
iii. Different Modes of Enforcement
1. The confidentiality and privilege rules are implemented or enforced in
different ways
2. If a lawyer seeks information that an opposing lawyer claims is privileged,
that lawyer claiming privilege might move to quash or subpoena or object
to compliance with a discovery request
iv. Chart of Differences
Ethical Duty to Protect Attorney-Client Privilege
Confidences
Source Ethical duty, 1.6 Common-law evidence rule

35
Scope Information relating to the Narrower Scope: confidential
representation of a client communication between a lawyer and
(obtained from any source) a client for the purpose of obtaining
legal advice

Method of Professional discipline Quash subpoena or otherwise exclude


Enforcement the revelation from evidence

B. Attorney-Client Privilege
i. How does A-C privilege protect the right to be represented by counsel?
1. Both confidentiality and privilege are based on the idea that a legal system in which
advocates speak for clients will work best if clients feel free to speak openly to their
attorneys
2. The privilege helps to protect lawyers and clients from the prospect that
an adversary might call a lawyer as a witness against the lawyer’s own
client
ii. Can showing a need for the information overcome a privilege claim?
1. The privilege is pretty close to absolute
2. However,
In Vela v. Super. Ct., 208 Cal. App. 3d 141 (1989), the privilege was
overrode because it would have deprived the criminal defendant of their
constitutional right to confrontation and cross examination. (see p. 200).
In Henderson v. State (Tex. Crim. App. 1997), the attorney-client privilege
had to yield to the strong public policy interest of protecting a child from
death or serious bodily harm. (see p. 200).
iii. Chart w/Occasions Where Privilege May Be Claimed
Type of Case Privilege might be invoked as to arguably privileged material
if:
Criminal Cases  A lawyer or a client is subpoenaed to testify before a grand
jury
 A lawyer is subpoenaed to testify before a trial jury
 A client is cross-examined during a trial
 A client’s documents are seized from his lawyer pursuant
to a search warrant
Civil cases and administrative  Discovery is sought from a lawyer or a client through
adjudicators depositions, interrogatories, or a request for production of
documents
 A lawyer is called to testify before a judge or jury at trial
Legislative and administrative  A lawyer or a client is subpoenaed to testify before a
investigations legislative or an administrative agency
A reporting statute appears to  A lawyer seeks to avoid disclosure by preemptive
compel a lawyer to disclose proceeding

36
information even without an
official request

iv. Source of Attorney-Client Privilege


1. Privilege is a common law doctrine, so the exact scope of the privilege is
somewhat different in each state
2. When federal courts apply state law, they also apply state privilege rules
3. When federal courts apply federal law, they apply a federal common law
of privilege, and there are slight variations
C. Elements of Attorney-Client Privilege
i. Communication
1. The privilege may be claimed for a face-to-face conversation or other
communicative acts
2. The privilege only protects against disclosure of the communication
itself
a. This means the communication with the lawyer is privileged, but the
underlying facts are not
ii. Privileged Persons
1. Include both the actual lawyer and lawyer’s staff
2. A lawyer should not casually allow a third person to be present during a
confidential communication because the person’s presence could later be
found to constitute a waiver of privilege
3. If a client brings a third person to a meeting with a lawyer, the lawyer
should clarify the role of the third person
4. Communications with a prospective client are also privileged
5. The privilege also generally applies to communications about a client
matter between two lawyers, rather than between a lawyer and a client
iii. Communications in Confidence
1. The client must reasonably believe that the communication is confidential
iv. Communication for Purpose of Seeking Legal Assistance
1. Communication is privileged only if the purpose was obtaining legal
assistance
2. The conversation is still privileged even if the legal advice is given to a
friend (not billed)
v. Chart of Requirements
Requirements for attorney- Comment
client privilege
A communication between The communication may be oral, written, electronic, etc.
privileged persons Privileged persons include:
 Lawyers, clients, prospective clients, interpreters
 Others who participate in the lawyer-client
conversation to facilitate the communication
 Secretaries, paralegals, and other agents of the
lawyer or the client

37
Which the client reasonably The communication must be private, and if the lawyer or
believes is confidential the client reports on the communication to non-
privileged persons, the privilege is waived
And whose purpose is to seek Business advice and personal advice are not covered by
or to provide legal advice or the privilege.
legal services

vi. Chart w/Consequences if a Communication is Privileged


Consequence Comment
The lawyer can’t be forced to The lawyer may not testify over the client’s objection,
testify even if he is willing to do so. The privilege belongs to the
client. A lawyer may not waive privilege over his client’s
objection
The client can’t be forced to The client may waive the privilege, even if the lawyer
testify objects
Paper and electronic records of Only the communication, not the underlying
a privileged communication information, is protected
are also protected

D. Client Identity under Attorney Client Privilege


1. In general, the identity of a client is not privileged
2. Some courts have protected a client’s identity if disclosure would reveal
a client confidence or incriminate the client in the very criminal activity
for which the client sought advice in the first place
E. Waiver
i. Waiver by Client
1. The attorney-client privilege can be waived by the client:
a. expressly by a voluntary act
b. unintentionally in a casual conversation
c. if the client reveals the privileged information to a non-privileged
person
ii. Waiver by Attorney
1. What if a lawyer reveals privileged communication to a non-privileged
person?
a. Privilege can be waived by the client’s lawyer if the client has
authorized the waiver
b. The client could authorize the lawyer to waive privilege by telling
the lawyer that he may do so (express authority) by giving the lawyer
authority that impliedly includes the authority to waive (implied
authority) or by making a statement to a third party that the lawyer
has the authority to waive privilege (apparent authority)
c. If the lawyer deliberately reveals the information without having
express, implied, or apparent authority, the revelation does not
effect a waiver of privilege
2. A lawyer can waive privilege by failure to invoke it during a trial.

38
iii. Waiver by Putting Privileged Information Into Issue
1. The privilege is also waived if the client puts the privileged
communication into issue in a case
iv. Waiver as a Conversation by Disclosure of Part of It
1. If a lawyer or client discloses part of an otherwise privileged lawyer -client
communication, a judge might find that the partial disclosure was a waiver
of the privilege as to the part of the conversation that relates to the
subject matter on which the client volunteered testimony
v. Compliance w/Court Order
1. Might be required to disclose information or be held in contempt of court
F. Crime Fraud Exception
i. General
1. No Privilege if a Client Seeks Assistance w/a Crime or Fraud
ii. A conversation where a client asks a lawyer for advice in planning or help
in committing a crime or fraud
1. Even if a client-conversation satisfies all the criteria above for privilege,
no privilege attaches if the client consults a lawyer for assistance in
committing a crime or fraud
2. Likewise there is no privilege for a conversation if the client later uses the
advice he received from the lawyer during the conversation to commit a
crime or a fraud
iii. A client asks his lawyer for advice about a past criminal or fraudulent act
1. Such communication is privileged so long as the past act is really past
2. The crime-fraud exception does not apply to past crimes or frauds
3. If there is a continuing crime or fraud that results from a past act, there is
no privilege
iv. If a client asks a lawyer for advice, learns the planned conduct is criminal,
and doesn’t commit the crime, is the convo privileged?
1. Yes, one purpose of the privilege is to enable clients to get sound advice
from lawyers and avoid committing criminal acts
v. A client consults a lawyer about a planned crime but doesn’t know the
planned conduct is criminal
1. The client’s intention to perform a criminal or fraudulent act triggers the
crime-fraud exception
2. It doesn’t matter whether the client knows that the act is wrongful
G. Procedure for Challenging a Claim of Privilege
1. A judge who is persuaded by the party seeking review that there is a
sufficient factual basis for challenging the application of the privilege can
order its examination
H. Revelations Permitted or Required by the Ethics Code
1. If a lawyer reveals information in the context of litigation, pursuant to an
exception to the rules and the privilege claim is asserted in a different
proceeding, any waiver of privilege might be held to be limited to the
original proceeding

39
2. In some instances, an explicit exception in the rules has been incorporated
into the law of privilege, which permits a lawyer to reveal otherwise
privileged information if allowed by an ethics rule
I. Death of a Client
i. General
1. Traditionally, the attorney-client privilege remains in force even if
revelation would prevent the wrongful incarceration or execution of an
innocent person, and it remains in effect after the client dies
ii. Swidler & Berlin v. United States
1. When a client makes certain communications that are protected by the
attorney-client privilege and then subsequent to making the statements
dies, the privilege survives the death of the client and, except for in
litigation between the declarant’s heirs, the communications are not
admissible as evidence.
2. NOTE: Recall the 60 minutes segment where the attorneys exonerated a
guy who was convicted of a crime their dead client committed.
Man, Logan, imprisoned for murder. Attorneys knew he was innocent,
because their client, Wilson, confessed to the murder. They couldn’t
speak out. They were torn morally, but they felt they couldn’t say
anything if it wasn’t death. They wrote a sealed notarized affidavit
saying they knew Logan was innocent and locked it under a bed.
They were more concerned about protecting their client than getting
disbarred. And they think their testimony wouldn’t have been allowed
in court because it violated attorney-client privilege.
They convinced Wilson to let them tell after his death. He agreed.
When he died, they took the affidavit out of the lockbox & contacted
Logan’s lawyer.
J. Work-Product Doctrine
i. Origins
1. The work product doctrine was first recognized by the Supreme Court in
Hickman v. Taylor
ii. What does it protect and apply to?
1. It protects notes and other material that a lawyer prepares in anticipation
of litigation from discovery in pretrial civil proceedings
2. It applies to documents that a lawyer prepares or collects while working
on pending on litigation or on a matter in which the lawyer knows that a
lawsuit is about to be filed
iii. What does it not protect and apply to?
1. Materials not created or collected in anticipation of litigation
a. The work product doctrine does not protect materials that a lawyer
creates or collects for reasons other than to prepare for litigation
b. It is only the lawyer’s need to use the information in litigation that
creates a degree of protection

40
c. If certain information would have been collected routinely but also
was collected because litigation was anticipated, most court will deny
protection to the information
2. Does not generally apply to documents relevant to impending litigation a
client gives to a lawyer
a. However, if the lawyers can demonstrate that their “selection and
compilation” of the documents reflects their litigation strategy, the
documents may be protected
i. Absent such a showing, the work product doctrine does not
apply
iv. Limitations In Practice
1. Courts are more likely to enforce it in the case of a witness statement to a
lawyer if the lawyer asks questions and takes notes that reflect her
strategic thinking in asking the questions rather than merely asking the
witness to mail her a statement
2. A judge can order disclosure of written or oral information otherwise
protected by this doctrine if the opposing party can show “substantial
need” for the material and that the opposing party is “unable without
undue hardship to obtain the substantial equivalent” of the material by
other means
v. Protection of a Lawyer’s Mental Impressions
1. A judge can order disclosure of written or oral information otherwise
protected by this doctrine if the opposing party can show “substantial
need” for the material and that the opposing party is “unable without
undue hardship to obtain the substantial equivalent” of the material by
other means
2. A lawyer’s own notes of his own opinions, theories, observations, or
feelings are immune from discovery
vi. Protection of Word Product, Not Underlying Information
1. The protection offered by the work product is not as powerful as it might
seem
2. A lawyer often can get the information contained in protected documents
from the original witnesses or sources
3. The doctrine prevents freeloading on an opponent’s work, but it does not
enable the opponent to close off a lawyer’s sources of information by
getting there first
vii. Expert Witnesses
1. Work product protection is given to an expert witness’s working drafts
and their communications with the attorneys who hired them
K. Privilege for a Corporation Client
i. Upjohn Case
1. In the corporate context, attorney-client privilege extends to lower level
employees, not just to those in control of the corporation. The work -
product doctrine protects oral statements made to attorneys, which
necessitates a showing of undue hardship on the part of the party-
opponent who seeks that information.
41
a. The Petitioner, Upjohn Co. (Petitioner), conducted an internal audit
and investigation that revealed alleged illegal payments made to foreign
officials in exchange for business. Petitioner volunteered notice of
such actions to the Internal Revenue Service (IRS), who issued a
summons for information collected by Petitioner, including internal
questionnaires sent to managerial employees. Petitioner maintained
those documents were protected by the attorney-client privilege and
attorney work product.
L. Privilege for Government Officials
1. Government officials being advised by lawyers do not enjoy the same
attorney-client privilege and work production protection as do employees
of private corporations

A. Formation of the Relationship


i. General Questions
1. Should lawyers investigate prospective clients before agreeing to represent
them?
a. Yes, if they feel uncertain about whether the prospective clients are
being dishonest with them
2. May a lawyer accept legal work that requires knowledge of an area of law
in which the lawyer has no experience?
a. Yes, if the lawyer compensates for inexperience through study or
affiliation with another lawyer
b. A lawyer may take on work in a new field only if he does the necessary
study
c. A lawyer may be subject to discipline if the lawyer bills the client for
spending an unreasonable amount of time on research, especially if the
research does not lead to worthwhile progress on the matter
3. Is a lawyer obligated to represent a client who wants to hire him?
a. In general, lawyers are allowed the discretion to decide whom to
represent
b. They don’t have to accept any particular clients and may craft their
practices according to their interests and aspiration
c. Likewise, a lawyer may accept representation of a client but limit the
scope of the work according to the lawyer’s wishes and abilities
d. Pro Bono exception – Rule 6.1
e. No lawyer is obligated to accept representation of a client whose
character or cause the lawyer regards as repugnant, even if the judge
asks the lawyer to accept a particular matter
4. To form a client-lawyer relationship, must the client sign an agreement or
pay a fee?

42
a. An agreement to pay a fee is unnecessary
ii. Rule 6.2 Accepting Appointments
A lawyer shall not seek to avoid appointment by a tribunal to represent a
person except for good cause, such as:
(a) representing the client is likely to result in violation of the Rules of
Professional Conduct or other law;
(b) representing the client is likely to result in an unreasonable financial
burden on the lawyer; or
(c) the client or the cause is so repugnant to the lawyer as to be likely to
impair the client-lawyer relationship or the lawyer's ability to represent the
client
iii. Existence of an attorney-client relationship (Togstad v. Vesely, Otto, Miller
& Keefe)
1. A retainer and actual retention are not required for an attorney-client
relationship to exist and that may give rise to a malpractice claim.
2. The core of this relationship is the attorney providing advice to a
person which the attorney expects will be followed.
3. In this case, Mrs. Togstad (Plaintiff) requested and received legal advice
from Miller (Defendant). It was totally reasonable for Miller (Defendant)
to have expected the Togstads (Plaintiff) would follow his advice, which is
exactly what they did. Consequently, for purposes of a malpractice action,
an attorney-client relationship did exist between the Plaintiff and
Defendant.
B. Lawyer’s Responsibilities as Agents
i. General
1. Lawyers are the agents of their clients who, in turn are considered
principles
2. Therefore, a client is bound by what the lawyer does or fails to do,
regardless of the client’s own actions or culpability
3. Law of agency principles apply to lawyers, clients, and third parties
ii. Express and Implied Authority
1. A client may explicitly give a lawyer “express” authority to act on the
client’s behalf
2. Alternatively, a client may give a lawyer a general instruction that
implicitly allows the lawyer to take certain actions on the client’s behalf
3. Express authority and implied authority are both considered actual
authority
4. In many cases, either type of authority binds a client to a lawyer’s actions
a. However certain actions taken by lawyers may not be valid unless the
lawyers have express rather than implied authority
iii. Apparent Authority
1. Even if the agent has either express nor implied authority, she may have
“apparent” authority
2. When a client tells a third party that the client’s lawyer has the authority to
settle a claim on his behalf, the third party may rely on the lawyer’s

43
subsequent actions, even if the client did not actually authorize those
actions
3. Apparent authority also is sometimes found if a principal places an agent
in a position that causes a third person reasonably to believe that the
principal had given the agent express authority
iv. Authority to Settle
1. A few states conclude that merely by hiring lawyers to represent them in
litigation, clients authorize their lawyers to settle cases
2. In most states the mere fact that a lawyer represents a client in litigation
does not provide implied or apparent authority to allow the lawyer to
settle the case
C. Lawyers’ Duties of Competence, Honesty, Communication, and Diligence
i. Rule 1.1 Competence
A lawyer shall provide competent representation to a client. Competent
representation requires the legal knowledge, skill, thoroughness and
preparation reasonably necessary for the representation.
ii. Comment [2] of Rule 1.1
A lawyer need not necessarily have special training or prior experience to
handle legal problems of a type with which the lawyer is unfamiliar. A
newly admitted lawyer can be as competent as a practitioner with long
experience. Some important legal skills, such as the analysis of precedent,
the evaluation of evidence and legal drafting, are required in all legal
problems. Perhaps the most fundamental legal skill consists of
determining what kind of legal problems a situation may involve, a skill
that necessarily transcends any particular specialized knowledge. A lawyer
can provide adequate representation in a wholly novel field through
necessary study. Competent representation can also be provided through
the association of a lawyer of established competence in the field in
question.
iii. Competence in Criminal Cases
1. The Sixth Amendment requires that a criminal ∆ be provided with a
lawyer whose work meets at least the minimum standard of being
“effective”
2. Many judges are reluctant to overturn a criminal conviction because a
different lawyer might have done a better job for a ∆
iv. 6 Amendment Requirements for Ineffective Counsel (Strickland v.
th

Washington)
1. Facts. After being sentenced to death, Petitioner filed for a writ of
Habeas Corpus on the grounds that he was given ineffective assistance of
counsel.
2. Rule of Law. Error alone is insufficient to prove that a defendant was
deprived of their constitutional right to effective assistance of counsel.
3. Reasoning (O’Connor). What the Sixth Amendment encompasses when
it considers effective assistance:
First, the benchmark for judging any claim of ineffectiveness must be
whether the counsel’s conduct so undermined the proper functioning
44
of the judicial process that the entire trial cannot be relied upon as just
in result
Second, the defendant must meet a two prong test in order to show
that assistance of counsel was so defective as to require reversal or
setting aside of a death sentence:
(1) the counsel’s performance must be deficient, and
(2) that deficient performance must have prejudiced the defendant so
much as to have deprived him of a the right to a fair trial;
When judging the performance of an attorney, counsel must be given a
great deal of latitude, considering all circumstances. each case must be
considered on a case-by-case basis; and …
v. Examples of 6th Amendment Ineffective Assistance of Counsel
1. It can be ineffective assistance of counsel if a criminal defense lawyer
doesn’t know about or doesn’t advise the client about the immigration
consequences of a criminal matter
2. It can be ineffective assistance of counsel if a lawyer fails to tell a client
about a plea offer or provides had advice to a client about a plea offer
D. Lawyers’ Duty of Diligence
i. Rule 1.3: Diligence
A lawyer shall act with reasonable diligence and promptness in
representing a client
ii. Comments [1],[2],[3],[4] for Rule 1.3
[1] A lawyer should pursue a matter on behalf of a client despite
opposition, obstruction or personal inconvenience to the lawyer, and
take whatever lawful and ethical measures are required to vindicate a
client's cause or endeavor. A lawyer must also act with commitment and
dedication to the interests of the client and with zeal in advocacy upon
the client's behalf. A lawyer is not bound, however, to press for every
advantage that might be realized for a client. For example, a lawyer may
have authority to exercise professional discretion in determining the
means by which a matter should be pursued. See Rule 1.2. The lawyer's
duty to act with reasonable diligence does not require the use of
offensive tactics or preclude the treating of all persons involved in the
legal process with courtesy and respect.
[2] A lawyer's work load must be controlled so that each matter can be
handled competently.
[3] Perhaps no professional shortcoming is more widely resented than
procrastination. A client's interests often can be adversely affected by
the passage of time or the change of conditions; in extreme instances,
as when a lawyer overlooks a statute of limitations, the client's legal
position may be destroyed. Even when the client's interests are not
affected in substance, however, unreasonable delay can cause a client
needless anxiety and undermine confidence in the lawyer's
trustworthiness. A lawyer's duty to act with reasonable promptness,
however, does not preclude the lawyer from agreeing to a reasonable
request for a postponement that will not prejudice the lawyer's client.

45
[4] Unless the relationship is terminated as provided in Rule 1.16, a
lawyer should carry through to conclusion all matters undertaken for a
client. If a lawyer's employment is limited to a specific matter, the
relationship terminates when the matter has been resolved. If a lawyer
has served a client over a substantial period in a variety of matters, the
client sometimes may assume that the lawyer will continue to serve
on a continuing basis unless the lawyer gives notice of withdrawal.
Doubt about whether a client-lawyer relationship still exists should be
clarified by the lawyer, preferably in writing, so that the client will not
mistakenly suppose the lawyer is looking after the client's affairs when the
lawyer has ceased to do so. For example, if a lawyer has handled a judicial
or administrative proceeding that produced a result adverse to the client
and the lawyer and the client have not agreed that the lawyer will handle
the matter on appeal, the lawyer must consult with the client about the
possibility of appeal before relinquishing responsibility for the matter. See
Rule 1.4(a)(2). Whether the lawyer is obligated to prosecute the appeal for
the client depends on the scope of the representation the lawyer has
agreed to provide to the client. See Rule 1.2.
[5] To prevent neglect of client matters in the event of a sole practitioner's
death or disability, the duty of diligence may require that each sole
practitioner prepare a plan, in conformity with applicable rules, that
designates another competent lawyer to review client files, notify each
client of the lawyer's death or disability, and determine whether there is a
need for immediate protective action. Cf. Rule 28 of the American Bar
Association Model Rules for Lawyer Disciplinary Enforcement (providing
for court appointment of a lawyer to inventory files and take other
protective action in absence of a plan providing for another lawyer to
protect the interests of the clients of a deceased or disabled lawyer).
E. Lawyers’ Duties of Candor and Communication
i. Rule 1.4 Communication
(a) A lawyer shall:
(1) promptly inform the client of any decision or circumstance with
respect to which the client's informed consent, as defined in Rule 1.0(e),
is required by these Rules;
(2) reasonably consult with the client about the means by which the
client's objectives are to be accomplished;
(3) keep the client reasonably informed about the status of the matter;
(4) promptly comply with reasonable requests for information; and
(5) consult with the client about any relevant limitation on the lawyer's
conduct when the lawyer knows that the client expects assistance not
permitted by the Rules of Professional Conduct or other law.
(b) A lawyer shall explain a matter to the extent reasonably necessary to
permit the client to make informed decisions regarding the representation
ii. Rule 2.1 Advisor
In representing a client, a lawyer shall exercise independent
professional judgement and render candid advice. In rendering advice,
a lawyer may refer not only to law but to other considerations such as
46
moral, economic, social and political factors, that may be relevant to the
client’s situation.
[Comment 1] “A client is entitled to straightforward advice expressing the
lawyer's honest assessment. Legal advice often involves unpleasant facts and
alternatives that a client may be disinclined to confront. In presenting advice,
a lawyer endeavors to sustain the client's morale and may put advice in as
acceptable a form as honesty permits. However, a lawyer should not be
deterred from giving candid advice by the prospect that the advice will be
unpalatable to the client.”
[Comment 2] “Advice couched in narrow legal terms may be of little value
to a client, especially where practical considerations, such as cost or effects
on other people, are predominant. Purely technical legal advice, therefore,
can sometimes be inadequate. It is proper for a lawyer to refer to relevant
moral and ethical considerations in giving advice. Although a lawyer is not a
moral advisor as such, moral and ethical considerations impinge upon most
legal questions and may decisively influence how the law will be applied.”
[Comment 3] When a client who is educated in legal matters, a lawyer may
accept the client’s request for advice in purely technical terms.
[Comment 5] “In general, a lawyer is not expected to give advice until asked
by the client. However, when a lawyer knows that a client proposes a course
of action that is likely to result in substantial adverse legal consequences to
the client, the lawyer’s duty to the client under Rule 1.4 may require that the
lawyer offer advice if the client’s course of action is related to the
representation. Similarly, when a matter is likely to involve litigation, it may
be necessary under Rule 1.4 to inform the client of forms of dispute
resolution that might constitute reasonable alternatives to litigation.
iii. Candor in Counseling
1. Questions that might be relevant in assessing whether a lie is justifiable
include the following:
Is the subject lied about either trivial or private
Is anyone harmed by the law
Is the purpose of the lie to protect someone?
Does the person lied to have a right to know or a strong interest in
knowing the truth
Is there a reason to tell a lie, can the problem be solved without lying
If you lie, will you need to tell other lies to cover up the first one?
Is the subject duties impose by contract in addition to those imposed
by the Ethics Code?
2. A lawyer and client may agree that the lawyer will comply with higher
standards of performance than those set by the ethics codes
3. The ethics codes articulates a minimum standard of performance
4. A lawyer may not agree to a higher contractual standard if the duty might
cause the lawyer to violate another ethical rule
F. Who Calls the Shots
i. Rule 1.2 Scope of Representation and Allocation of Authority Between
Client and Lawyer

47
1. (a) Subject to paragraphs (c) and (d), a lawyer shall abide by a client's
decisions concerning the objectives of representation and, as required by
Rule 1.4, shall consult with the client as to the means by which they are to
be pursued. A lawyer may take such action on behalf of the client as is
impliedly authorized to carry out the representation. A lawyer shall abide
by a client's decision whether to settle a matter. In a criminal case, the
lawyer shall abide by the client's decision, after consultation with the
lawyer, as to a plea to be entered, whether to waive jury trial and whether
the client will testify.
(b) A lawyer's representation of a client, including representation by
appointment, does not constitute an endorsement of the client's political,
economic, social or moral views or activities.
(c) A lawyer may limit the scope of the representation if the limitation is
reasonable under the circumstances and the client gives informed consent.
(d) A lawyer shall not counsel a client to engage, or assist a client, in
conduct that the lawyer knows is criminal or fraudulent, but a lawyer may
discuss the legal consequences of any proposed course of conduct with a
client and may counsel or assist a client to make a good faith effort to
determine the validity, scope, meaning or application of the law.
ii. Restatement in re 1.2
1. The restatement states that except for decisions reserved for clients and in
the absence of an agreement on these matters, a lawyer may take “any
lawful measure within the scope of representation that is reasonably
calculated to advance a client’s objective.”
iii. Jones v. Barnes
1. Respondent presented his appointed counsel with a list of claims on which
to base his appeal. His attorney found these to be untenable, and
proceeded to argue the appeal on different grounds. Respondent now
claims ineffective assistance of counsel.
2. The Sixth Amendment’s “effective assistance of counsel” standard does
not require a court-appointed attorney to argue every non-frivolous point
raised by his client.
3. “No decision of this Court suggests that an indigent defendant has a
constitutional right to compel appointed counsel to press non-frivolous
points requested by the client, if counsel, as a matter of professional
judgment, decides not to present those points”
iv. 1.16(b) When a Lawyer May Withdraw Representation
(b) Except as stated in paragraph (c), a lawyer may withdraw from
representing a client if:
(1) withdrawal can be accomplished without material adverse effect on the
interests of the client;
(2) the client persists in a course of action involving the lawyer's services
that the lawyer reasonably believes is criminal or fraudulent;
(3) the client has used the lawyer's services to perpetrate a crime or fraud;
(4) the client insists upon taking action that the lawyer considers
repugnant or with which the lawyer has a fundamental disagreement;

48
(5) the client fails substantially to fulfill an obligation to the lawyer
regarding the lawyer's services and has been given reasonable warning that
the lawyer will withdraw unless the obligation is fulfilled;
(6) the representation will result in an unreasonable financial burden on
the lawyer or has been rendered unreasonably difficult by the client; or
(7) other good cause for withdrawal exists.
v. The Competent Adult Client
1. [Comment 1] of Rule 1.14 “The normal client-lawyer relationship is
based on the assumption that the client, when properly advised and
assisted, is capable of making decisions about important matters…”
2. Some decisions must be made instantly – lawyers cannot possibly consult
the client for everything – it would be impractical
vi. Rule 1.14 Clients with Diminished Capacity
1. (a) When a client's capacity to make adequately considered decisions in
connection with a representation is diminished, whether because of
minority, mental impairment or for some other reason, the lawyer
shall, as far as reasonably possible, maintain a normal client-lawyer
relationship with the client.
(b) When the lawyer reasonably believes that the client has diminished
capacity, is at risk of substantial physical, financial or other harm unless
action is taken and cannot adequately act in the client's own interest, the
lawyer may take reasonably necessary protective action, including
consulting with individuals or entities that have the ability to take action
to protect the client and, in appropriate cases, seeking the appointment
of a guardian ad litem, conservator or guardian.
(c) Information relating to the representation of a client with diminished
capacity is protected by Rule 1.6. When taking protective action pursuant
to paragraph (b), the lawyer is impliedly authorized under Rule 1.6(a) to
reveal information about the client, but only to the extent reasonably
necessary to protect the client's interests.
2. Comment [1] The normal client-lawyer relationship is based on the
assumption that the client, when properly advised and assisted, is capable
of making decisions about important matters. When the client is a minor
or suffers from a diminished mental capacity, however, maintaining the
ordinary client-lawyer relationship may not be possible in all respects. In
particular, a severely incapacitated person may have no power to make
legally binding decisions. Nevertheless, a client with diminished capacity
often has the ability to understand, deliberate upon, and reach conclusions
about matters affecting the client's own well-being. For example, children
as young as five or six years of age, and certainly those of ten or twelve,
are regarded as having opinions that are entitled to weight in legal
proceedings concerning their custody. So also, it is recognized that some
persons of advanced age can be quite capable of handling routine financial
matters while needing special legal protection concerning major
transactions.
3. Comment [6] In determining the extent of the client's diminished
capacity, the lawyer should consider and balance such factors as: the

49
client's ability to articulate reasoning leading to a decision, variability of
state of mind and ability to appreciate consequences of a decision; the
substantive fairness of a decision; and the consistency of a decision with
the known long-term commitments and values of the client. In
appropriate circumstances, the lawyer may seek guidance from an
appropriate diagnostician.
vii. Clients who May Have Mental Impairments
1. The rule contemplates that the lawyer may consult about the matter on
which the lawyer is representing the client with “individuals or entities”
other than the clients, such as members of the client’s family of public
agencies
viii. Can a lawyer avoid making decisions for a possibly impaired client by
getting someone else to make those decisions?
1. A lawyer who represents a client who has an intellectual impairment or a
diagnosis of mental illness has several options
a. Follow the clients instructions to the farthest extent possible
b. Impose their own ideas of what is best
c. Invite others to provide substitute guidance
2. If it seems necessary to give someone else legal authority over the client,
the lawyer might petition the court to appoint a guardian ad litem, a
conservator, or a guardian for the client
a. A guardian ad litem is charged with the duty to determine and
advocate for the best interests of the client; the best interests may not
correspond with the expressed wishes of the party
b. A conservator is given power to manage the financial affairs of the
client who thereby loses the power to buy, sell, and hold property
c. A guardian has even more authority. A guardian manages the client’s
financial affairs and may make medical and other person decisions for
the client
ix. Delinquency Cases
1. When lawyers represent children who are charged with juvenile defenses,
lawyers typically represent older children, and the cases resemble criminal
cases
2. Most lawyers therefore follow norms of representation similar to the
norms they follow when representing adults in court
G. Terminating a Lawyer-Client Relationship
i. Duties to the Client at the Conclusion of the Relationship
1. Most lawyer-client relationships end when all the work on the relevant
matter has been completed
2. When the work is finished, the lawyer must return to the client “any
papers and property to which the client is entitled” and must return any
unearned payment that the client may have made (1.16(d))
3. The relationship is not entirely over at this point because the lawyer has a
duty to protect client confidences, a duty that continues indefinitely (1.6,
comment 20)

50
ii. If a client has not paid the bill at the end of the representation, may the
lawyer keep the client’s documents until the client pays?
1. Can keep unless retention would “unreasonably harm the client”
iii. Grounds for Termination Before Work is Completed
1. See Rule 1.16
iv. Rule 1.16 Declining or Terminating Representation
(a) Except as stated in paragraph (c), a lawyer shall not represent a client
or, where representation has commenced, shall withdraw from the
representation of a client if:
(1) the representation will result in violation of the rules of
professional conduct or other law;
+ A lawyer must withdraw if representation will require the lawyer to
violate the law, including the state’s rules of professional conduct
+ If the client has already used the lawyer’s service to commit a
crime or fraud but continued representation will not result in a new
or continuing crime or fraud, the lawyer may withdraw but is not
required to do so
+ If the client insists on action the lawyer finds repugnant
(2) the lawyer's physical or mental condition materially impairs the
lawyer's ability to represent the client; or
(3) the lawyer is discharged.
+ A lawyer must withdraw if the client fires them
+ A client always has the right to change lawyers and need not give a
reason, although a client for whom a lawyer has been appointed may
not change lawyers without the court’s permission
(b) Except as stated in paragraph (c), a lawyer may withdraw from
representing a client if:
(1) withdrawal can be accomplished without material adverse effect on the interests
of the client;
(2) the client persists in a course of action involving the lawyer's services that the
lawyer reasonably believes is criminal or fraudulent;
+ A lawyer may withdraw if the client doesn’t pay the lawyer’s fee, but the
lawyer must first warn the client that nonpayment will lead to withdrawal
(3) the client has used the lawyer's services to perpetrate a crime or fraud;
(4) the client insists upon taking action that the lawyer considers repugnant or with
which the lawyer has a fundamental disagreement;
(5) the client fails substantially to fulfill an obligation to the lawyer regarding the
lawyer's services and has been given reasonable warning that the lawyer will
withdraw unless the obligation is fulfilled;
(6) the representation will result in an unreasonable financial burden on the lawyer
or has been rendered unreasonably difficult by the client; or
+ Unreasonable financial burden arises when the matter turns out to be
more complex than the lawyer originally anticipated
+ A client is “unreasonably difficult” when the client refuses to cooperate
(7) other good cause for withdrawal exists.
(c) A lawyer must comply with applicable law requiring notice to or
permission of a tribunal when terminating a representation. When
ordered to do so by a tribunal, a lawyer shall continue representatio n
notwithstanding good cause for terminating the representation.
(d) Upon termination of representation, a lawyer shall take steps to the
extent reasonably practicable to protect a client's interests, such as
giving reasonable notice to the client, allowing time for employment of
51
other counsel, surrendering papers and property to which the client is
entitled and refunding any advance payment of fee or expense that has
not been earned or incurred. The lawyer may retain papers relating to
the client to the extent permitted by other law.

v. Fees after withdrawing from representation


1. When a lawyer withdraws from representing a client, may the lawyer
collect a fee for the work that the lawyer did prior to withdrawal?
It depends. Some courts allow lawyers to collect fees based on the
reasonable value of the work done until that point. Others may not
allow fees if the lawyer’s reasons aren’t persuasive enough.
H. Rule 1.8(b) Lawyer’s Duty to Not Use Information to Disadvantage of
Client
(b) A lawyer shall not use information relating to representation of a client
to the disadvantage of the client unless the client gives informed consent,
except as permitted or required by these Rules.

A. Introduction
i. General
1. Lawyers are supposed to avoid conflicts of interest
2. When a lawyer undertakes representation of a client, the lawyer owes the
client a duty of loyalty and a duty to protect confidential information
3. Conflict rules guide lawyers on how to distinguish among three conflict
categories
a. Conflicts that on close examination do not present a real risk of harm
to one or more clients, so the lawyer may proceed without advising the
client or asking for consent
b. Conflicts that may be adequately addressed if the affected clients are
willing to provide consent after the lawyer explains all the potential
problems
c. Conflicts that are so serious that even consent would not solve the
problem, and the lawyer should turn down the second client (or
withdraw, if the lawyer already has begun representation
ii. Possible Consequences of Representing a Client in the Face of Conflict
1. Legal Sanctions
a. Disqualification
b. Discipline
c. Malpractice liability
d. Injunction against representation
e. Fee Forfeiture
2. Business repercussions
a. Client may retain a different lawyer

52
b. Client may mistrust you
c. Your professional reputation may suffer
iii. Difficulties of Study of Conflict of Interest
1. Technical lingo
2. Conflicts with whom?
A lawyer can have conflicts caused by obligations to present clients, to
former clients, or to some other person
Can have conflicts because of the obligation of lawyers with whom the
lawyer works or used to work
Conflict between the client’s interests and their own
3. An intake or midstream
Conflicts can develop on the outset of representation or later on
Conflicts when companies merge
Conflicts when new associates are hired
4. Factual context is everything
Conflict can relate the relationship between two clients, to the subject
matter at issue in a case, or to a specific issue that arises in litigation
5. Special rules for particular types of work
6. Complex solutions
7. Inconsistent rules and fuzzy standards
B. Conflict Rule Chart
Type of Simultaneous Is one lawyer Lawyer(s) work for the Relevant
conflict or sequential or more than same firm Rules
representation one lawyer
involved?
Concurrent Simultaneous One lawyer Yes 1.7, 1.8, 1.13
Successive Sequential One lawyer Yes 1.9, 1.11(a),
(c), (d), 1.12(a)
& (b)
Imputed Simultaneous Two or more Concurrently in same firm 1.10, 1.11(b),
or sequential lawyers but may previously have been 1.12(c)
in a different firm

C. General Rules
i. Rule 1.7 Conflict of Interest: Concurrent Conflicts
(a) Except as provided in paragraph (b), a lawyer shall not represent a client if
the representation involves a concurrent conflict of interest. A concurrent
conflict of interest exists if:
(1) the representation of one client will be directly adverse to
another client; or
(2) there is a significant risk that the representation of one or more
clients will be materially limited by the lawyer's responsibilities to
another client, a former client or a third person or by a personal
interest of the lawyer.

53
(b) Notwithstanding the existence of a concurrent conflict of interest under
paragraph (a), a lawyer may represent a client if:
(1) the lawyer reasonably believes that the lawyer will be able to
provide competent and diligent representation to each affected
client;
(2) the representation is not prohibited by law;
(3) the representation does not involve the assertion of a claim by
one client against another client represented by the lawyer in the
same litigation or other proceeding before a tribunal; and
(4) each affected client gives informed consent, confirmed in
writing.
ii. Comment [2] of Rule 1.7: Requirements for Resolution of Conflict of
Interest Problem
[2] Resolution of a conflict of interest problem under this Rule requires
the lawyer to: 1) clearly identify the client or clients; 2) determine
whether a conflict of interest exists; 3) decide whether the
representation may be undertaken despite the existence of a conflict, i.e.,
whether the conflict is consentable; and 4) if so, consult with the clients
affected under paragraph (a) and obtain their informed consent,
confirmed in writing. The clients affected under paragraph (a) include
both of the clients referred to in paragraph (a)(1) and the one or more
clients whose representation might be materially limited under paragraph
(a)(2).
iii. Rule 1.10 Imputation of Conflict of Interest and Firms
(a) While lawyers are associated in a firm, none of them shall knowingly
represent a client when any one of them practicing alone would be
prohibited from doing so by Rules 1.7 or 1.9, unless
(1) the prohibition is based on a personal interest of the disqualified
lawyer and does not present a significant risk of materially limiting the
representation of the client by the remaining lawyers in the firm; or
(2) the prohibition is based upon Rule 1.9(a) or (b) and arises out of the
disqualified lawyer’s association with a prior firm, and
(i) the disqualified lawyer is timely screened from any
participation in the matter and is apportioned no part of the fee
therefrom;
(ii) written notice is promptly given to any affected former client
to enable the former client to ascertain compliance with the
provisions of this Rule, which shall include a description of the
screening procedures employed; a statement of the firm's and of
the screened lawyer's compliance with these Rules; a statement that
review may be available before a tribunal; and an agreement by the
firm to respond promptly to any written inquiries or objections by
the former client about the screening procedures; and
(iii) certifications of compliance with these Rules and with the
screening procedures are provided to the former client by the
screened lawyer and by a partner of the firm, at reasonable
intervals upon the former client's written request and upon
termination of the screening procedures.

54
(b) When a lawyer has terminated an association with a firm, the firm is not
prohibited from thereafter representing a person with interests materially
adverse to those of a client represented by the formerly associated lawyer and
not currently represented by the firm, unless:
(1) the matter is the same or substantially related to that in which the
formerly associated lawyer represented the client; and
(2) any lawyer remaining in the firm has information protected by Rules
1.6 and 1.9(c) that is material to the matter.
(c) A disqualification prescribed by this rule may be waived by the affected
client under the conditions stated in Rule 1.7.
(d) The disqualification of lawyers associated in a firm with former or current
government lawyers is governed by Rule 1.11.
iv. Rule 1.0 (k) “Screened”
“Screened” denotes the isolation of a lawyer from any participation in a
matter through the timely imposition of procedures within a firm that are
reasonably adequate under the circumstances to protect information that
the isolated lawyer is obligated to protect under these Rules or other law.
D. Evaluating Concurrent Conflicts
i. Overview of Rule 1.7
1. Rule 1.7 first explains what is considered to be a problematic conflict
between two present clients (or one present and one prospective client)
2. It then explains that for some, but not all, of these conflicts, a lawyer may
seek informed consent from the affected clients and may proceed if
consent is obtained
3. 1.7 addresses how a lawyer should analyze conflicts between two present
clients or between two current obligations of the lawyer
4. The rule identifies two types of conflicts that might preclude
representation of a client:
One client’s interests might be “directly adverse” to those of another
client
There might be a “significant risk” that representation also could be
materially limited by another obligation of the lawyer – to another
client, to a former client, or to a third person. A representation also
could be materially limited by a conflict between the lawyer’s and
client’s interests
5. If a conflict is present, a lawyer should evaluate under 1.7(b), whether they
may continue to represent the affected clients if they give informed
consent after learning about the conflicts
ii. Direct Adversary and Material Limitation (Rule 1.7)
1. Direct Adversity
a. A conflict is said to involve “direct adversity” to the interests of a
client if the lawyer’s conduct on behalf of one client requires the
lawyer to act directly against the interests of another client
b. Includes both litigation and nonlitigation
c. Comment [6] Loyalty to a current client prohibits undertaking
representation directly adverse to that client without that client's

55
informed consent. Thus, absent consent, a lawyer may not act as an
advocate in one matter against a person the lawyer represents in some
other matter, even when the matters are wholly unrelated. The client
as to whom the representation is directly adverse is likely to feel
betrayed, and the resulting damage to the client-lawyer relationship is
likely to impair the lawyer's ability to represent the client effectively. In
addition, the client on whose behalf the adverse representation is
undertaken reasonably may fear that the lawyer will pursue that client's
case less effectively out of deference to the other client, i.e., that the
representation may be materially limited by the lawyer's interest in
retaining the current client. Similarly, a directly adverse conflict may
arise when a lawyer is required to cross-examine a client who appears
as a witness in a lawsuit involving another client, as when the
testimony will be damaging to the client who is represented in the
lawsuit. On the other hand, simultaneous representation in unrelated
matters of clients whose interests are only economically adverse, such
as representation of competing economic enterprises in unrelated
litigation, does not ordinarily constitute a conflict of interest and thus
may not require consent of the respective clients.
d. Comment [7] Directly adverse conflicts can also arise in transactional
matters. For example, if a lawyer is asked to represent the seller of a
business in negotiations with a buyer represented by the lawyer, not in
the same transaction but in another, unrelated matter, the lawyer could
not undertake the representation without the informed consent of
each client.
2. Material Limitation Comment [8]
a. Even if there is no direct adversity, a conflict exists if representation
of one client would be materially limited by one of the other
responsibilities of the lawyer
b. The other responsibilities that might materially limit a lawyer’s
representation of a client include obligations to
Another present client
A former client
Someone else to whom a lawyer owes a duty
Someone other than the client who is paying the lawyer’s fee
The lawyer’s own financial, employment, personal, or other
interest
c. To evaluate whether a conflict is present, a lawyer must ask two
questions:
i. How likely is it that a difference in interests will evaluate?
ii. If there likely is such divergence, would it materially interfere
with the lawyer’s advice to or representation of a client
iii. Personal Interest Conflicts
1. Comment [11] When lawyers representing different clients in the same
matter or in substantially related matters are closely related by blood or
marriage, there may be a significant risk that client confidences will be
revealed and that the lawyer's family relationship will interfere with both
loyalty and independent professional judgment. As a result, each client is
56
entitled to know of the existence and implications of the relationship
between the lawyers before the lawyer agrees to undertake the
representation. Thus, a lawyer related to another lawyer, e.g., as parent,
child, sibling or spouse, ordinarily may not represent a client in a matter
where that lawyer is representing another party, unless each client gives
informed consent. The disqualification arising from a close family
relationship is personal and ordinarily is not imputed to members of firms
with whom the lawyers are associated. See Rule 1.10.
iv. How to Evaluate Conflicts
1. To resolve a concurrent conflict under Rule 1.7, a lawyer must:
a. Clearly identify the client or clients and determine whether each is a
present client of former client
b. Determine whether a conflict of interest exists
c. Decide whether the lawyer is permitted to represent the client despite
the existence of a conflict, and
d. If so, consult with clients affect under paragraph (a), obtain their
informed consent, and send written confirmation to the client of the
informed consent
2. If a consentable conflict is identified before a client is accepted, and the
affect client gives informed consent after the lawyer explains the possible
problems that the conflict might generate, the lawyer may go forward
despite the conflict
v. Non-Consentable Conflicts
1. To determine whether a conflict is consentable, Rule 1.7(b) directs a
lawyer to ask
a. Whether she reasonably believes that she will be able to provide
competent and diligent representation to the relevant clients
b. Whether the representation is prohibited by law and
c. Whether the representation involves litigation in which the lawyer is
representing one client against another client with whom the lawyer is
representing in that matter
2. Comments 15-17
a. [15] Consentability is typically determined by considering whether the
interests of the clients will be adequately protected if the clients are
permitted to give their informed consent to representation burdened
by a conflict of interest. Thus, under paragraph (b)(1), representation
is prohibited if in the circumstances the lawyer cannot reasonably
conclude that the lawyer will be able to provide competent and diligent
representation. See Rule 1.1 (competence) and Rule 1.3 (diligence).
b. [16] Paragraph (b)(2) describes conflicts that are nonconsentable
because the representation is prohibited by applicable law. For
example, in some states substantive law provides that the same lawyer
may not represent more than one defendant in a capital case, even
with the consent of the clients, and under federal criminal statutes
certain representations by a former government lawyer are prohibited,
despite the informed consent of the former client. In addition,

57
decisional law in some states limits the ability of a governmental client,
such as a municipality, to consent to a conflict of interest.
c. [17] Paragraph (b)(3) describes conflicts that are nonconsentable
because of the institutional interest in vigorous development of each
client's position when the clients are aligned directly against each other
in the same litigation or other proceeding before a tribunal. Whether
clients are aligned directly against each other within the meaning of
this paragraph requires examination of the context of the proceeding.
Although this paragraph does not preclude a lawyer's multiple
representation of adverse parties to a mediation (because mediation is
not a proceeding before a "tribunal" under Rule 1.0(m)), such
representation may be precluded by paragraph (b)(1).
vi. Informed Consent
1. General
a. If the conflict is consentable, a lawyer may ask the affected clients
whether they would like to waive the conflict by giving informed
consent
b. If the client declines to give consent, the lawyer cannot take on or
continue the conflict work
c. The lawyer must orally explain to the client the risk, advantages, and
possible alternatives to the lawyer going forward with the
representation
d. A client’s oral consent must be confirmed in writing by the lawyer
2. Comments 18, 19
a. [18] Informed consent requires that each affected client be aware of
the relevant circumstances and of the material and reasonably
foreseeable ways that the conflict could have adverse effects on the
interests of that client. See Rule 1.0(e) (informed consent). The
information required depends on the nature of the conflict and the
nature of the risks involved. When representation of multiple clients in
a single matter is undertaken, the information must include the
implications of the common representation, including possible effects
on loyalty, confidentiality and the attorney-client privilege and the
advantages and risks involved. See Comments [30] and [31] (effect of
common representation on confidentiality).
b. [19] Under some circumstances it may be impossible to make the
disclosure necessary to obtain consent. For example, when the lawyer
represents different clients in related matters and one of the clients
refuses to consent to the disclosure necessary to permit the other
client to make an informed decision, the lawyer cannot properly ask
the latter to consent. In some cases the alternative to common
representation can be that each party may have to obtain separate
representation with the possibility of incurring additional costs. These
costs, along with the benefits of securing separate representation, are
factors that may be considered by the affected client in determining
whether common representation is in the client's interests.
vii. Comment 20 Consent Confirmed in Writing

58
1. [20] Paragraph (b) requires the lawyer to obtain the informed consent of
the client, confirmed in writing. Such a writing may consist of a document
executed by the client or one that the lawyer promptly records and
transmits to the client following an oral consent. See Rule 1.0(b). See also
Rule 1.0(n) (writing includes electronic transmission). If it is not feasible
to obtain or transmit the writing at the time the client gives informed
consent, then the lawyer must obtain or transmit it within a reasonable
time thereafter. See Rule 1.0(b). The requirement of a writing does not
supplant the need in most cases for the lawyer to talk with the client, to
explain the risks and advantages, if any, of representation burdened with a
conflict of interest, as well as reasonably available alternatives, and to
afford the client a reasonable opportunity to consider the risks and
alternatives and to raise questions and concerns. Rather, the writing is
required in order to impress upon clients the seriousness of the decision
the client is being asked to make and to avoid disputes or ambiguities that
might later occur in the absence of a writing.
viii. Comments 23-25 Conflicts in Litigation
1. [23] Paragraph (b)(3) prohibits representation of opposing parties in the
same litigation, regardless of the clients' consent. On the other hand,
simultaneous representation of parties whose interests in litigation may
conflict, such as coplaintiffs or codefendants, is governed by paragraph
(a)(2). A conflict may exist by reason of substantial discrepancy in the
parties' testimony, incompatibility in positions in relation to an opposing
party or the fact that there are substantially different possibilities of
settlement of the claims or liabilities in question. Such conflicts can arise
in criminal cases as well as civil. The potential for conflict of interest in
representing multiple defendants in a criminal case is so grave that
ordinarily a lawyer should decline to represent more than one
codefendant. On the other hand, common representation of persons
having similar interests in civil litigation is proper if the requirements of
paragraph (b) are met.
2. [24] Ordinarily a lawyer may take inconsistent legal positions in different
tribunals at different times on behalf of different clients. The mere fact
that advocating a legal position on behalf of one client might create
precedent adverse to the interests of a client represented by the lawyer in
an unrelated matter does not create a conflict of interest. A conflict of
interest exists, however, if there is a significant risk that a lawyer's action
on behalf of one client will materially limit the lawyer's effectiveness in
representing another client in a different case; for example, when a
decision favoring one client will create a precedent likely to seriously
weaken the position taken on behalf of the other client. Factors relevant
in determining whether the clients need to be advised of the risk include:
where the cases are pending, whether the issue is substantive or
procedural, the temporal relationship between the matters, the significance
of the issue to the immediate and long-term interests of the clients
involved and the clients' reasonable expectations in retaining the lawyer. If
there is significant risk of material limitation, then absent informed

59
consent of the affected clients, the lawyer must refuse one of the
representations or withdraw from one or both matters.
3. [25] When a lawyer represents or seeks to represent a class of plaintiffs or
defendants in a class-action lawsuit, unnamed members of the class are
ordinarily not considered to be clients of the lawyer for purposes of
applying paragraph (a)(1) of this Rule. Thus, the lawyer does not typically
need to get the consent of such a person before representing a client suing
the person in an unrelated matter. Similarly, a lawyer seeking to represent
an opponent in a class action does not typically need the consent of an
unnamed member of the class whom the lawyer represents in an unrelated
matter.
ix. Withdrawal and Disqualification
1. If the lawyer discovers a nonconsentable conflict they must withdraw
from representation (Rule 1.16)
x. Imputation of Concurrent Conflicts
1. A lawyer can have a conflict of interest because his firm represents two
clients who interests conflict or because his firm represents one client
whose interests conflict with those of a client represented by his partner
2. The imputation rules apply to conflicts presented by all lawyers in a firm
but do not preclude representation based on conflicts presented by other
nonlawyer staff
3. A firm must maintain a conflicts-checking system that lists the clients and
former clients of lawyers and nonlawyer employees to identify cases in
which screening is necessary

A. Conflicts Between Current Clients in Civil Litigation


i. Suing a Current Client
1. A lawyer may never be involved in litigation in which the lawyer
represents one client in a lawsuit against another
2. Lawyers are not allowed to sue a current client unless the lawyer
reasonably believes that they can represent both without adverse impact
on either and unless both clients give informed consent
ii. Cross-Examining a Current Client
1. A lawyer can have an adverse relationship to a current client in litigation
even if the client is not a party in the case in which the conflict arises
2. One such situation occurs if a lawyer is called upon to cross-examine one
of her client in a trial involving charges against another of her clients
iii. Representation of Co-Plaintiffs or Co-Defendants in Civil Litigation
1. For a lawyer to sue one client on behalf of another presents perhaps the
most serious type of conflict of interest
2. If a lawyer jointly represents two clients who are both either ∏s or ∆s in a
lawsuit, their relationship is not one of direct adversity, but their interests
might conflict anyway

60
a. One client might have a potential claim against the other
b. If the clients are co-∏s, they might be suing a ∆ whose limited assets
would make it impossible to satisfy both their claims
c. If a settlement is proposed, the two ∏s might have different views on
whether to settle. The ∆ might try to trade one client’s claim off
against the other
d. If a lawyer represents two ∆s who each have some responsibility for
the harm that is the subject of the suit, then each might seek to avoid
liability by asserting that the other is responsible
e. If the clients are ∏s in a lawsuit seeking injunctive relief as well as
damages, they might disagree as to what the remedy should be
3. If a lawyer determines the conflict is consentable, to truly obtain informed
consent, the lawyer must disclose all the facts and circumstances that
might adversely affect the client’s interests.
a. See Comment 18, Rule 1.7
b. The disclosure should include:
The conflicting or potentially conflicting interests of the other clients
The potential courses of action that may be foreclosed or limited by
the joint representation
The effect of representation upon the client’s confidential information
and on the attorney-client privilege
Any reservations the lawyer would have about the representation if the
lawyer were representing only the client being advised, and
The consequences on the representation if one client later withdraws
their consent to the joint representation
B. Positional Conflicts
i. Representing Economic Competitors in Unrelated Matters (Rule 1.7,
Comment [6])
1. Loyalty to a current client prohibits undertaking representation directly
adverse to that client without that client's informed consent. Thus, absent
consent, a lawyer may not act as an advocate in one matter against a
person the lawyer represents in some other matter, even when the matters
are wholly unrelated. The client as to whom the representation is directly
adverse is likely to feel betrayed, and the resulting damage to the client -
lawyer relationship is likely to impair the lawyer's ability to represent the
client effectively. In addition, the client on whose behalf the adverse
representation is undertaken reasonably may fear that the lawyer will
pursue that client's case less effectively out of deference to the other
client, i.e., that the representation may be materially limited by the lawyer's
interest in retaining the current client. Similarly, a directly adverse conflict
may arise when a lawyer is required to cross-examine a client who appears
as a witness in a lawsuit involving another client, as when the testimony
will be damaging to the client who is represented in the lawsuit. On the
other hand, simultaneous representation in unrelated matters of clients
whose interests are only economically adverse, such as representation of
competing economic enterprises in unrelated litigation, does not ordinarily

61
constitute a conflict of interest and thus may not require consent of the
respective clients.
C. Conflicts Involving Prospective Clients
i. Rule 1.18 Duties to Prospective Client
(a) A person who consults with a lawyer about the possibility of forming a client-
lawyer relationship with respect to a matter is a prospective client.
(b) Even when no client-lawyer relationship ensues, a lawyer who has learned
information from a prospective client shall not use or reveal that information, except
as Rule 1.9 would permit with respect to information of a former client.
(c) A lawyer subject to paragraph (b) shall not represent a client with interests
materially adverse to those of a prospective client in the same or a
substantially related matter if the lawyer received information from the
prospective client that could be significantly harmful to that person in the matter,
except as provided in paragraph (d). If a lawyer is disqualified from representation
under this paragraph, no lawyer in a firm with which that lawyer is associated may
knowingly undertake or continue representation in such a matter, except as
provided in paragraph (d).
(d) When the lawyer has received disqualifying information as defined in paragraph
(c), representation is permissible if:
(1) both the affected client and the prospective client have given informed
consent, confirmed in writing, or:
(2) the lawyer who received the information took reasonable measures to
avoid exposure to more disqualifying information than was reasonably
necessary to determine whether to represent the prospective client; and
(i) the disqualified lawyer is timely screened from any
participation in the matter and is apportioned no part of the fee
therefrom; and
(ii) written notice is promptly given to the prospective client.
ii. Comment [2] of Rule 1.18
1. [2] A person becomes a prospective client by consulting with a lawyer
about the possibility of forming a client-lawyer relationship with respect
to a matter. Whether communications, including written, oral, or
electronic communications, constitute a consultation depends on the
circumstances. For example, a consultation is likely to have occurred if a
lawyer, either in person or through the lawyer’s advertising in any
medium, specifically requests or invites the submission of information
about a potential representation without clear and reasonably
understandable warnings and cautionary statements that limit the lawyer’s
obligations, and a person provides information in response. See also
Comment [4]. In contrast, a consultation does not occur if a person
provides information to a lawyer in response to advertising that merely
describes the lawyer’s education, experience, areas of practice, and contact
information, or provides legal information of general interest. Such a
person communicates information unilaterally to a lawyer, without any
reasonable expectation that the lawyer is willing to discuss the possibility
of forming a client-lawyer relationship, and is thus not a "prospective
client." Moreover, a person who communicates with a lawyer for the
purpose of disqualifying the lawyer is not a “prospective client.”

62
D. Representing Both Parties to a Transaction
i. Standards Applied if Both Buyer and Seller Want You to Represent Them
Both
1. If both a buyer and a seller want you to represent both of them, the
conflict is evaluated using the standards articulated in Rule 1.7
2. So whether representation is permitted and whether client consent is
required depends on whether there is a direct adversity conflict or a
material limitation conflict
3. If so the lawyer must evaluate whether she reasonably believes that she
will be able to provide competent and diligent representation to each
affected client
ii. How does a lawyer know whether she needs to obtain informed consent?
a. The relevant question is whether there is an actual or potential conflict
that is reasonably apparent to the lawyer
b. If the clients’ interest appear to be harmonious, there may be no need
to obtain consent
E. Representing Organizations
i. Rule 1.13 Organization as Client
(a) A lawyer employed or retained by an organization represents the organization
acting through its duly authorized constituents.

(b) If a lawyer for an organization knows that an officer, employee or other person
associated with the organization is engaged in action, intends to act or refuses to
act in a matter related to the representation that is a violation of a legal
obligation to the organization, or a violation of law that reasonably might be
imputed to the organization, and that is likely to result in substantial injury to
the organization, then the lawyer shall proceed as is reasonably necessary in
the best interest of the organization. Unless the lawyer reasonably believes that it
is not necessary in the best interest of the organization to do so, the lawyer
shall refer the matter to higher authority in the organization, including, if
warranted by the circumstances to the highest authority that can act on behalf
of the organization as determined by applicable law.

(c) Except as provided in paragraph (d), if


(1) despite the lawyer's efforts in accordance with paragraph (b) the highest
authority that can act on behalf of the organization insists upon or fails to
address in a timely and appropriate manner an action, or a refusal to act, that
is clearly a violation of law, and
(2) the lawyer reasonably believes that the violation is reasonably certain to
result in substantial injury to the organization, then the lawyer may reveal
information relating to the representation whether or not Rule 1.6 permits such
disclosure, but only if and to the extent the lawyer reasonably believes
necessary to prevent substantial injury to the organization.

(d) Paragraph (c) shall not apply with respect to information relating to a
lawyer's representation of an organization to investigate an alleged violation of
law, or to defend the organization or an officer, employee or other constituent

63
associated with the organization against a claim arising out of an alleged
violation of law.

(e) A lawyer who reasonably believes that he or she has been discharged
because of the lawyer's actions taken pursuant to paragraphs (b) or (c), or who
withdraws under circumstances that require or permit the lawyer to take action
under either of those paragraphs, shall proceed as the lawyer reasonably believes
necessary to assure that the organization's highest authority is informed of
the lawyer's discharge or withdrawal.

(f) In dealing with an organization's directors, officers, employees, members,


shareholders or other constituents, a lawyer shall explain the identity of the
client when the lawyer knows or reasonably should know that the organization's
interests are adverse to those of the constituents with whom the lawyer is
dealing.

(g) A lawyer representing an organization may also represent any of its directors,
officers, employees, members, shareholders or other constituents, subject to the
provisions of Rule 1.7. If the organization's consent to the dual representation
is required by Rule 1.7, the consent shall be given by an appropriate official of
the organization other than the individual who is to be represented, or by the
shareholders

ii. Factors Affecting Whether a Related Entity is a Client


Related entity more likely to Related entity less likely to be a
be a client if: client if:
The lawyer received confidential The lawyer no longer represents the
information from or provided initial corporate client
advice to the related entity such
as a subsidiary
The entity was controlled and The two entities become linked
supervised by the parent (merger) after the lawyer began
organization representation of the first entity
The original client could be
materially harmed by the suit
against the related entity

iii. Representing the Entity and Employees


1. A lawyer in a private practice who represents an organization may
represent a member or an employee of the organization unless the
interests of the organization and the individual conflict
2. In general, a lawyer who is an employee of the corporation and represents
the corporation may not undertake representation of shareholders of the
corporation
3. Authority is divided about whether a lawyer who works for an insurance
company may represent insured persons
iv. Duty to Protect Confidence and Employees

64
1. If an employee of a corporation discloses information to the corporation’s
lawyer in confidence, does the lawyer have an obligation the employee to
protect the confidence?
a. It depends.
i. If the lawyer represents only the organization and not any
employee, the lawyer generally has no duty to protect the
confidences of employees.
ii. The lawyer’s duty is to protect the information belonging to
the organization from others outside the organization
v. Responding to Unlawful Conduct by Corporate Officers and Other
Employees
1. In the course of representing an organization, a lawyer might learn that an
officer or employee has done something or is planning to do something
that is illegal or that would cause harm to the organization
2. The lawyer’s duty is to the organization, not to the senior executives
3. See Rule 1.13
vi. Entity Lawyers on Boards of Directors
1. Lawyers aren’t forbidden from sitting on the boards of directors of
organizations they represent
F. Representing Co-Defendants in Criminal Cases
i. Costs and Benefits of Joint Representation
1. The best chance for avoiding conviction may be in presenting a unified
defense
2. However, joint-representation could ultimately involve a significant
sacrifice of the interests of one client on behalf of another
ii. Ethics Rules and the Sixth Amendment
1. The sixth amendment provides that “In all criminal prosecutions, the
accused shall enjoy the right…to have the Assistance of Counsel for his
defense:
2. A ∆ can waive his right to have a lawyer who owes no loyalty to a co -∆
iii. Relevant Rules
1. 1.6 (Confidentiality), 1.7 (Concurrent Conflicts), 1.8(f) (Conditions for
Accepting Compensation), 1.10 (Imputation of Conflicts of Interests), 1.9
(Duties to Former Clients)
G. Representing Family Members
i. Representing Both Sides in a Divorce
1. As long as there is no disagreement about child custody or the division of
property there is no actual adversity, only technical adversity
2. Disagreements can arise later down the road as financial assets are divided
a. Some states prohibit the lawyer from representing both parties in the
divorce
b. But others allow it as long as the resulting agreement “seems fair”
ii. Representing Family Members in Estate Planning

65
1. What if the lawyer represents the husband and wife in the planning of
their estates (drafting and signing wills) and the husband reveals to the
lawyer that he wants to update the will and leave things to his mistress
(Florida Bar Opinion)
a. The lawyer must withdraw from the joint representation under the
facts presented.
b. An adversity of interests concerning the joint representation has arisen
– creating a conflict of interest
c. Lawyer should inform the wife and husband that a conflict has arisen
that precludes the lawyer’s continued representation of both parties
d. Lawyer cannot disclose the separate confidence to the wife
H. Representing Insurance Companies and Insured Persons
i. Fundamental Dilemma
1. The insurance lawyer’s dilemma is fundamentally about money
a. The lawyer is being paid the client (the insurer) to represent both itself
and another client (the insured)
b. This dilemma is governed in part by contract law and by insurance law,
and also the ethical rules
ii. Who is the client?
1. It depends, but either way the lawyer may not reveal confidential
information about the insured to the insurer
iii. When is there a conflict of interest between the insurer and the insured?
Rule 1.8(f)
1.8(f) A lawyer shall not accept compensation for representing a client from
one other than the client unless:
(1) the client gives informed consent;
(2) there is no interference with the lawyer's independence of professional
judgment or with the client-lawyer relationship; and
(3) information relating to representation of a client is protected as
required by Rule 1.6.
iv. What should the lawyer do if a conflict arises?
1. Act in the best interests of the client but don’t help them commit fraud
I. Rule 1.8 (g) Aggregate Settlements/Pleas
(g) A lawyer who represents two or more clients shall not participate in making
an aggregate settlement of the claims of or against the clients, or in a criminal
case an aggregated agreement as to guilty or nolo contendere pleas, unless each
client gives informed consent, in a writing signed by the client. The lawyer's
disclosure shall include the existence and nature of all the claims o r pleas
involved and of the participation of each person in the settlement.
J. Relevant Rules
1. 1.6 (Confidentiality), 1.7 (Concurrent Conflicts), 1.8(f) (Conditions for
Accepting Compensation), 1.8(g) (Aggregate Settlements/Pleas), 1.10
(Imputation of Conflicts of Interests), 1.9 (Duties to Former Clients)

66
A. Conflicts Between Present and Former Clients
i. Timing and Possible Implications
1. Timing:
a. A conflict with a former client’s interests may develop or become
apparent after a lawyer is well into some work on behalf of a current
client
2. In the event of a conflict between a former client and a present client, a
lawyer might
a. Betray confidences of a former client to a present client
b. Make adverse use of confidences that the lawyer learned during the
representation of the former client
c. Attack or challenge work that the lawyer did on behalf of the former
client
d. Engage in work that is in some other way disloyal to the former client
or at least causes the former client to feel betrayed
ii. Applicable Ethics Rules
1. Protection for former clients are provided by Rule 1.9
2. Protection for present clients are provide by Rule 1.7
iii. Is a conflict between a present and former client both a successive and
current conflict?
1. Yes
iv. Chart: Scenarios w/Present and Former Clients
Situation Possible Interest Harmed Can this conflict be waived? Rule
Lawyer knows (or Breach of commitment to keep Yes, by informed consent of 1.9
might’ve had access to) confidences of former client former client
info from a former client
that could be used
aversely to former client
New client wants lawyer Possible adverse use of Yes, by informed consent of 1.9
to sue lawyer’s former confidences of former client present and former clients,
client, whom lawyer depending on subject matter unless the conflict is so severe 1.7
represented for years on a that the lawyer could not
variety of matters Lawyer’s advocacy on behalf of reasonably believe that he could
new client might be provide competent and diligent
compromised by the relationship representation to the new client
with former client

v. Are the ethical rules less restrictive as to successive conflicts than they are
as to concurrent conflicts?
1. Yes, a lawyer’s duties to former clients are limited mainly to protecting
confidences, avoiding side-switching, and refraining from attacking the
work the lawyer did for the former client
B. Duties to Former Clients
67
i. What duties does a lawyer owe to a former client?
1. The primary duty that lawyers owe to former clients is to protect their
confidences
2. A lawyer must decline any new matter that presents a substantial risk that
the lawyer would make material adverse use of the former client’s
confidences, unless the former client consents.
ii. Rule 1.9 Duties to Former Clients
(a) A lawyer who has formerly represented a client in a matter shall not
thereafter represent another person in the same or a substantially
related matter in which that person's interests are materially adverse to
the interests of the former client unless the former client gives informed
consent, confirmed in writing.
(b) A lawyer shall not knowingly represent a person in the same or a
substantially related matter in which a firm with which the lawyer
formerly was associated had previously represented a client
(1) whose interests are materially adverse to that person; and
(2) about whom the lawyer had acquired information protected by
Rules 1.6 and 1.9(c) that is material to the matter; unless the former
client gives informed consent, confirmed in writing.
(c) A lawyer who has formerly represented a client in a matter or whose
present or former firm has formerly represented a client in a matter shall
not thereafter:
(1) use information relating to the representation to the disadvantage
of the former client except as these Rules would permit or require with
respect to a client, or when the information has become generally
known; or
(2) reveal information relating to the representation except as these
Rules would permit or require with respect to a client.
iii. Comment [3] “Substantially Related”
1. Matters are "substantially related" for purposes of this Rule if they
involve the same transaction or legal dispute or
if there otherwise is a substantial risk that confidential factual
information as would normally have been obtained in the prior
representation would materially advance the client's position in the
subsequent matter.
For example, a lawyer who has represented a businessperson and learned
extensive private financial information about that person may not then
represent that person's spouse in seeking a divorce.
Similarly, a lawyer who has previously represented a client in securing
environmental permits to build a shopping center would be precluded
from representing neighbors seeking to oppose rezoning of the property
on the basis of environmental considerations; however, the lawyer would
not be precluded, on the grounds of substantial relationship, from
defending a tenant of the completed shopping center in resisting eviction
for nonpayment of rent. Information that has been disclosed to the public
or to other parties adverse to the former client ordinarily will not be
disqualifying.
68
Information acquired in a prior representation may have been rendered
obsolete by the passage of time, a circumstance that may be relevant in
determining whether two representations are substantially related.
In the case of an organizational client, general knowledge of the client’s
policies and practices ordinarily will not preclude a subsequent
representation; on the other hand, knowledge of specific facts gained in a
prior representation that are relevant to the matter in question ordinarily
will preclude such a representation. A former client is not required to
reveal the confidential information learned by the lawyer in order to
establish a substantial risk that the lawyer has confidential information to
use in the subsequent matter. A conclusion about the possession of such
information may be based on the nature of the services the lawyer
provided the former client and information that would in ordinary
practice be learned by a lawyer providing such services
iv. Distinguishing Present and Former Clients
1. Can a lawyer drop one client to clear the way to take on representation of
a more lucrative matter for a new client?
a. It’s generally frowned upon, but if one of the following conditions is
satisfied, the lawyer may use the more lenient successive conflict
standards to evaluate the conflict
i. The lawyer withdraws at the natural end point in the
representation
ii. The client fires the lawyers for reasons other than the
impending conflict
iii. The client triggers a conflict for the lawyer by some action
that was unforeseeable to the lawyer. This type of conflict is
sometimes referred to as thrust upon a lawyer by the client.
In this case, the lawyer's withdrawal makes this client a former
client
iv. The lawyer withdraws for some other good reason
C. Evaluating Successive Conflicts
i. Critical question
1. is whether the conflict is serious enough that the lawyer may not go
forward with the new work
ii. Applying Rule 1.9 (General)
1. The lawyer must ask:
Is it the “same matter” as to the previous one
If not, is it substantially related to the previous one?
2. If one of the two questions is answered yes, the lawyer must ask whether
the new client’s interests are materially adverse to the interests of the
former client
3. If the matter is the same or substantially related and there is material
adversity, the lawyer may not go forward with the new representation
without the former client’s informed consent
4. There must be written confirmation of consent.
iii. “The Same Matter”
69
1. When is one matter “the same matter” as the previous one?
a. A matter can be anything that is the subject of representation:
litigation, a transaction, a subject on which a client requests advice
b. The most obvious same matter is a single transaction or lawsuit
c. It is the same matter if the new representation involves a document
that the lawyer was involved in producing
iv. “Substantial Relationship”
1. When is there a “substantial relationship” between one matter and
another?
a. Ask if there is some connection between the earlier matter and the
new matter
b. There are two possible connections:
i. Two matters might involve the same parties, the same lawsuit,
and the same legal issues, or the same overlapping facts
2. Questions to ask regarding whether there is a “substantial
relationship”
a. Do the matters involve the same transaction or legal dispute? If yes,
they are substantially related
b. Is there a substantial risk that a lawyer representing a client in a matter
like the one handled for the former client would normally have learned
confidential information in the first matter that would be used to
materially advance the new client’s position in the second matter? To
answer that question, ask:
i. What types of information would a lawyer handling a matter
like the first one normally acquire?
ii. Would that information provide the second client with a
material advantage?
c. If both these questions are answered yes, the two matters are
substantially related
d. If the matters are substantially related, the lawyer may not represent
the new client unless the former client gives informed consent
3. Does the existence of a “substantial relationship” between the
present and former matter depend on common facts or common
legal issues?
a. The relationship between the two matters depends on whether the
factual information would normally have been learned during the first
matter that could be used adversely to the first client during the
second representation
4. Does a “substantial relationship” depend on whether a lawyer
actually learned information in the first representation that could be
used adversely to the former client in the later matter?
a. No, the existence of a substantial relationship depends not on what
the lawyer actually learned, but one what kinds of confidences a lawyer
ordinarily would have learned in the prior matter that could be used in
the second matter that would materially advance the new client’s
position, presumably harming the former client
70
v. Material Adversity
1. According to rule 1.9, if a new representation is substantially related to a
previous one and presents material adversity to the interests of the former
client, the lawyer may not pursue the new matter without the consent of
the former client
D. Courses of Action Available to a Lawyer Who Discovers a Conflict Between a
Former and Present Client
1. A lawyer who encounters a conflict that impacts a former client might
choose one of three courses of action:
a. The lawyer might conclude that the two matters are unrelated and
simply proceed with the new matter without seeking consent
b. The lawyer might conclude that there is a substantial relationship
between the matters and material adversity and tell the new client that
the firm cannot accept the matter
c. The lawyer might disclose the problem and ask for consent
E. Representing the Competitor of a Former Client
i. A comment to rule 1.7 states that ordinarily representation of economic
competitors poses no serious conflict of interest
ii. If it is not a conflict to represent economic competitors concurrently, one would
think it would be less problematic to represent the economic interests of a former
client
F. Conflicts between the interests of a present client and a client who was
represented by a lawyer’s former firm
i. Analyzing Former Firm Conflicts  Rule 1.9 (b)
(b) A lawyer shall not knowingly represent a person in the same or a
substantially related matter in which a firm with which the lawyer
formerly was associated had previously represented a client
(1) whose interests are materially adverse to that person; and
(2) about whom the lawyer had acquired information protected by
Rules 1.6 and 1.9(c) that is material to the matter; unless the former
client gives informed consent, confirmed in writing.
ii. Using or Revealing a Former Client’s Confidences  Rule 1.9 (c)
(c) A lawyer who has formerly represented a client in a matter or whose
present or former firm has formerly represented a client in a matter shall
not thereafter:
(1) use information relating to the representation to the disadvantage of
the former client except as these Rules would permit or require with
respect to a client, or when the information has become generally known;
or
(2) reveal information relating to the representation except as these Rules
would permit or require with respect to a client.
G. Imputation of Former Client Conflicts to Affiliated Lawyers
i. General Rule

71
1. The general rule stated in 1.10 is that if one lawyer in a firm is barred by
rule 1.7 or 1.9 from working on a matter, none of the other lawyers in the
firm is permitted to work on the matter either unless:
a. The affected client agrees to waive the conflict, 1.19(c)
b. The conflict involves a “personal interest” of the lawyer and there is
no significant risk that the representation of the client by others in the
firm would be materially limited because of the conflict, Rule
1.10(a)(1)
c. The conflict arises under Rule 1.9(a) or Rule 1.9(b) and the firm sets
up a screen that comports with the requirements of Rule 1.10(a)(2) to
bar the conflicted lawyer from communication about or access to
documents relating to the new matter
ii. Other situations besides those listed in 1.10(a)(2) in which the Model Rules
permit screening?
1. Where the conflict involves work in which a lawyer was involved before
that person became a lawyer, Comment 4
2. Where the conflict involves work done by a former government lawyer
while that person was in the government and where the government
lawyer has joined a firm that seeks to represent or is representing a client
whose interests conflict with the government lawyer’s prior work
3. Where one lawyer received confidential information from a prospective
client who did not become an actual client, and the lawyer’s firm seeks to
represent or is representing a client whose interests conflict with those of
the prospective client
4. Where a lawyer is disqualified from working on a matter because the
lawyer previously worked on the matter as a judge, law clerk, or in some
other similar role

A. Legal Fees
i. Rule 1.5 Fees
(a) A lawyer shall not make an agreement for, charge, or collect an unreasonable fee or an
unreasonable amount for expenses. The factors to be considered in determining the
reasonableness of a fee include the following:
(1) the time and labor required, the novelty and difficulty of the questions involved,
and the skill requisite to perform the legal service properly;
(2) the likelihood, if apparent to the client, that the acceptance of the particular
employment will preclude other employment by the lawyer;
(3) the fee customarily charged in the locality for similar legal services;
(4) the amount involved and the results obtained;
(5) the time limitations imposed by the client or by the circumstances;
(6) the nature and length of the professional relationship with the client;
(7) the experience, reputation, and ability of the lawyer or lawyers performing the
services; and

72
(8) whether the fee is fixed or contingent.
(b) The scope of the representation and the basis or rate of the fee and expenses for which
the client will be responsible shall be communicated to the client, preferably in writing,
before or within a reasonable time after commencing the representation, except when the
lawyer will charge a regularly represented client on the same basis or rate. Any changes in the
basis or rate of the fee or expenses shall also be communicated to the client.
(c) A fee may be contingent on the outcome of the matter for which the service is rendered,
except in a matter in which a contingent fee is prohibited by paragraph (d) or other law. A
contingent fee agreement shall be in a writing signed by the client and shall state the method
by which the fee is to be determined, including the percentage or percentages that shall
accrue to the lawyer in the event of settlement, trial or appeal; litigation and other
expenses to be deducted from the recovery; and whether such expenses are to be deducted
before or after the contingent fee is calculated. The agreement must clearly notify the client
of any expenses for which the client will be liable whether or not the client is the prevailing
party. Upon conclusion of a contingent fee matter, the lawyer shall provide the client with a
written statement stating the outcome of the matter and, if there is a recovery, showing the
remittance to the client and the method of its determination.
(d) A lawyer shall not enter into an arrangement for, charge, or collect:
(1) any fee in a domestic relations matter, the payment or amount of which is contingent
upon the securing of a divorce or upon the amount of alimony or support, or property
settlement in lieu thereof; or
(2) a contingent fee for representing a defendant in a criminal case.
(e) A division of a fee between lawyers who are not in the same firm may be made only if:
(1) the division is in proportion to the services performed by each lawyer or each lawyer
assumes joint responsibility for the representation;
(2) the client agrees to the arrangement, including the share each lawyer will receive,
and the agreement is confirmed in writing; and
(3) the total fee is reasonable.
ii. Matter of Fordham  Fees When You Need to Become Competant
1. Although it was reasonable of Respondent in this case to take the case
even if he was not experienced in the relevant law, it was not reasonable
of him to expect the client to pay for his “education” in this field of law
and then charge him nearly ten times more than an experienced attorney
would have charged. Although the client consented to the terms presented
by Respondent, Respondent never gave him a proper estimate of how
much the case might cost and he had no reason to believe that the total
bill would be anything approaching $50,000-especially since the average
case in this area bills about 30 hours.
2. Although an attorney may bill a client for “learning time,” an attorney
should not expect his client to pay for his education in fields of law to an
extent that this becomes an unreasonable burden. A lawyer should
generally not accept employment regarding any area of the law in which he
is not properly qualified.
iii. 1.5 (b)  See Rule 7.1 Communications Concerning A Lawyer’s
Services
1. A lawyer shall not make a false or misleading communication about
the lawyer or the lawyer's services. A communication is false or
misleading if it contains a material misrepresentation of fact or law, or
omits a fact necessary to make the statement considered as a whole not
materially misleading.

73
iv. Other Rules That May Come Up As a Result
1.4 Communication
8.3 Reporting Professional Misconduct
8.4 Misconduct
5.2 Responsibilities of a Subordinate Lawyer,
5.3 Responsibilities Regarding Nonlawyer Assistance
v. Contingent Fees
1. There are strict rules if the fee is contingent on the result achieved for the
client
2. See Rule 1.5 (c)
3. There are no actual specified limits
B. Forbidden and Restricted Fee and Expense Arrangements
i. Rule 1.8 (i) Buying Legal Claims
1. (i) A lawyer shall not acquire a proprietary interest in the cause of
action or subject matter of litigation the lawyer is conducting for a client,
except that the lawyer may:
(1) acquire a lien authorized by law to secure the lawyer's fee or
expenses; and
(2) contract with a client for a reasonable contingent fee in a civil
case
ii. When is a lien okay?  Comment [16] Comment [2]
1. This is a business transaction and must comply with 1.8(a) A lawyer shall
not enter into a business transaction with a client or knowingly acquire an
ownership, possessory, security or other pecuniary interest adverse to
client unless:
(1) The transaction and terms on which the lawyer acquires the interest
are fair and reasonable to the client and are fully disclosed and
transmitted in writing in a manner that can be reasonably understood
by the client;
(2) The client is advised in writing of the desirability of seeking and is
then given a reasonably opportunity to seek the advice of independent
legal counsel on the transaction; and
(3) The client gives informed consent, in a writing signed by the client,
to the essential terms of the transaction and the lawyer’s role in the
transaction, including whether the lawyer is representing the client in
the transaction
iii. Rule 1.8 (e) Financial Assistance to a Client
(e) A lawyer shall not provide financial assistance to a client in
connection with pending or contemplated litigation, except that:
(1) a lawyer may advance court costs and expenses of litigation, the
repayment of which may be contingent on the outcome of the
matter; and
(2) a lawyer representing an indigent client may pay court costs
and expenses of litigation on behalf of the client.
iv. [Comment 10] Financial Assistance
74
1. Lawyers may not subsidize lawsuits or administrative proceedings brought
on behalf of their clients, including making or guaranteeing loans to their
clients for living expenses, because to do so would encourage clients to
pursue lawsuits that might not otherwise be brought and because such
assistance gives lawyers too great a financial stake in the litigation. These
dangers do not warrant a prohibition on a lawyer lending a client court
costs and litigation expenses, including the expenses of medical
examination and the costs of obtaining and presenting evidence, because
these advances are virtually indistinguishable from contingent fees and
help ensure access to the courts. Similarly, an exception allowing lawyers
representing indigent clients to pay court costs and litigation expenses
regardless of whether these funds will be repaid is warranted
a. LOAN v GIFT - contingent on winning the case, interest in winning
case
b. 10 states do not have this rule
v. Rule 1.8 (d) Publication Rights
1. (d) Prior to the conclusion of representation of a client, a lawyer shall not
make or negotiate an agreement giving the lawyer literary or media rights
to a portrayal or account based in substantial part on information relating
to the representation.
vi. Advance Payment of Fees and Nonrefundable Retainer Fees
1. Retainer Fee
a. Provides lawyer w/protection against a client who becomes unwilling
or unable to pay for services the lawyer has already rendered
b. The advance is deposited in the lawyer’s client trust account and the
lawyer withdraws portions of the advance as they are earned.
c. Unearned portion of the advance must be returned
2. Lump-Sum Payment
a. Paid in advance to secure the lawyer’s availability for a period of time
for a particular task
b. This type of retainer may be treated as having been earned when it is
received, because the payment secures the lawyer’s availability but does
not depend on the performance of any particular task
c. The lawyer may have to forgo other obligations to maintain his
availability
3. Non-Refundable Advance Payment
a. Controversial because it locks the client into the relationship with the
lawyer and tends to constrain the client from firing the lawyer if, for
example, the lawyer fails to perform adequately
b. In some states, public policy disfavors nonrefundable retainers as
inconsistent with a lawyer’s fiduciary obligations to their client
c. In certain states, lawyers require nonrefundable retainers, which are
tolerated to protect lawyers from defendants who may otherwise not
pay their bills.
C. Fee Disputes

75
i. Rule 1.8 (h) Prospective Limitations of Lawyers’ Liability and
Settlement of Claims Against Lawyers
(h) A lawyer shall not:
(1) make an agreement prospectively limiting the lawyer's liability
to a client for malpractice unless the client is independently
represented in making the agreement; or
(2) settle a claim or potential claim for such liability with an
unrepresented client or former client unless that person is advised
in writing of the desirability of seeking and is given a reasonable
opportunity to seek the advice of independent legal counsel in
connection therewith.
ii. Comment [14] of Rule 1.8(h) Fee Arbitration
1. 1.8(h) doesn’t “prohibit a lawyer from entering into an agreement with the
client to arbitrate legal malpractice claims, provided such agreements are
enforceable and the client is fully informed of the scope and effect of the
agreement
iii. Comment [14] of Rule 1.8(h) Limited Liability Entity
1. 1.8 (h) doesn’t “limit the ability of lawyers to practice in the form of a
limited-liability entity, where permitted by law, provided that each lawyer
remains personally liable to the client for his or her own conduct and the
firm complies with any conditions required by law, such as provisions
requiring client notification or maintenance of adequate liability
insurance.”
iv. Collection of Fees
1. If the client does not pay the lawyer may sue the client to collect
2. No improper methods of collection
3. A lawyer should not:
a. Commit any acts of harassment against a debtor or make frivolous
claims
b. Retain documents or unearned fees that should be turned over to a
client as leverage to secure payments of fees
c. Make false or misleading statements
d. Reveal information to a third party to get a client to pay a fee
D. Dividing Fees with Other Firms
i. Rule 1.5 (e) Dividing Fees w/Lawyers Not in Same Firm
(e) A division of a fee between lawyers who are not in the same firm may be
made only if:
a. (1) the division is in proportion to the services performed by each
lawyer or each lawyer assumes joint responsibility for the
representation;
b. (2) the client agrees to the arrangement, including the share each
lawyer will receive, and the agreement is confirmed in writing; and
c. (3) the total fee is reasonable.
ii. Referral Fees
1. A lawyer may collect a referral fee if:
76
a. The lawyer takes on “financial and ethical responsibility for the
representation as if the lawyers were associated in a partnership”
b. The proposed share that each lawyer receives must be disclosed to and
approved by the client
c. Fee-sharing arrangement must be confirmed in writing
AND
d. Total fee must be reasonable
iii. What does it mean for a lawyer to take “financial and ethical responsibility
for the representation?
1. Agree to share responsibility as if they were law partners in the event of
disciplinary action or a malpractice suit
2. Each lawyer has a duty of supervision under Rule 5.1
3. A lawyer who collects a referral fee and doesn’t plan to collaborate is
advised to follow the case closely because if the lawyer doing the case
violates ethics rules or is negligent, the referring lawyer could be
disciplined or sued
E. Sharing Fees w/Non Lawyers
i. Rule 5.4(a)
(a) A lawyer or law firm shall not share legal fees with a nonlawyer, except
that:
(1) an agreement by a lawyer with the lawyer's firm, partner, or associate
may provide for the payment of money, over a reasonable period of time
after the lawyer's death, to the lawyer's estate or to one or more
specified persons;
(2) a lawyer who purchases the practice of a deceased, disabled, or
disappeared lawyer may, pursuant to the provisions of Rule 1.17, pay to
the estate or other representative of that lawyer the agreed-upon
purchase price;
(3) a lawyer or law firm may include nonlawyer employees in a
compensation or retirement plan, even though the plan is based in
whole or in part on a profit-sharing arrangement; and
(4) a lawyer may share court-awarded legal fees with a nonprofit
organization that employed, retained or recommended employment of
the lawyer in the matter.
ii. District of Columbia and Other Countries (i.e. UK)
1. Lawyers may split fees w/nonlawyers
iii. “Runners” who find clients that are accident victims
1. In most states, lawyers may not pay referral fees to nonlawyers who send
them clients, except that they may pay lawyer referral services for making
information available about their practices
F. Rule 1.8 (f) Payment of Fees by Third Parties
i. Rule
(f) A lawyer shall not accept compensation for representing a client from
one other than the client unless:
a. (1) the client gives informed consent;
77
b. (2) there is no interference with the lawyer's independence of
professional judgment or with the client-lawyer relationship; and
c. (3) information relating to representation of a client is protected as
required by Rule 1.6.
ii. Comment [13]
1. [13] A lawyer may be paid from a source other than the client, including a
co-client, if the client is informed of that fact and consents and the
arrangement does not compromise the lawyer's duty of loyalty or
independent judgment to the client. See Rule 1.8(f). If acceptance of the
payment from any other source presents a significant risk that the lawyer's
representation of the client will be materially limited by the lawyer's own
interest in accommodating the person paying the lawyer's fee or by the
lawyer's responsibilities to a payer who is also a co-client, then the lawyer
must comply with the requirements of paragraph (b) before accepting the
representation, including determining whether the conflict is consentable
and, if so, that the client has adequate information about the material risks
of the representation.
G. Lawyers as Custodians of Client Property and Documents
i. Client Trust Accounts
1. One important aspect of lawyer-client relationship is lawyer’s management
of the client trust account, the bank account in which the lawyer keeps
funds that belong to various clients
ii. Rule 1.15 Safekeeping Property
(a) A lawyer shall hold property of clients or third persons that is in a
lawyer's possession in connection with a representation separate from
the lawyer's own property. Funds shall be kept in a separate account
maintained in the state where the lawyer's office is situated, or
elsewhere with the consent of the client or third person. Other
property shall be identified as such and appropriately safeguarded.
Complete records of such account funds and other property shall be kept
by the lawyer and shall be preserved for a period of [five years] after
termination of the representation.
(b) A lawyer may deposit the lawyer's own funds in a client trust
account for the sole purpose of paying bank service charges on that
account, but only in an amount necessary for that purpose.
(c) A lawyer shall deposit into a client trust account legal fees and
expenses that have been paid in advance, to be withdrawn by the
lawyer only as fees are earned or expenses incurred.
(d) Upon receiving funds or other property in which a client or third
person has an interest, a lawyer shall promptly notify the client or third
person. Except as stated in this rule or otherwise permitted by law or by
agreement with the client, a lawyer shall promptly deliver to the client
or third person any funds or other property that the client or third person
is entitled to receive and, upon request by the client or third person,
shall promptly render a full accounting regarding such property.
(e) When in the course of representation a lawyer is in possession of
property in which two or more persons (one of whom may be the
78
lawyer) claim interests, the property shall be kept separate by the
lawyer until the dispute is resolved. The lawyer shall promptly
distribute all portions of the property as to which the interests are not
in dispute.
iii. May a lawyer act as executor of a client’s estate?
1. A lawyer may recommend and may accept appointment as executor of a
client’s estate. The solicitation of such appointment is not prohibited by
Rule 1.8(c) because the payment would be for services rendered
2. BUT, a lawyer giving advice about whom to appoint as executor or
carrying out the functions of executor for a client might be improperly
biased by self-interest in doing so.
3. A lawyer must comply with 1.7 in giving such advice or acting as executor
for a client’s estate
H. Conflicts with Lawyers’ Personal or Business Interests
i. In General  1.7 (a)(2)
(a) Except as provided in paragraph (b), a lawyer shall not represent a
client if the representation involves a concurrent conflict of interest. A
concurrent conflict of interest exists if:
(1)…(2) there is a significant risk that the representation of one or
more clients will be materially limited by the lawyer's responsibilities
to another client, a former client or a third person or by a personal
interest of the lawyer.
1. Hypotheticals
a. See p 543 of textbook
ii. Rule 1.8 (a) Business Transactions between Lawyers and Clients
(a) A lawyer shall not enter into a business transaction with a client or knowingly
acquire an ownership, possessory, security or other pecuniary interest adverse to a
client unless:
(1) the transaction and terms on which the lawyer acquires the interest are
fair and reasonable to the client and are fully disclosed and transmitted in
writing in a manner that can be reasonably understood by the client;
(2) the client is advised in writing of the desirability of seeking and is given a
reasonable opportunity to seek the advice of independent legal counsel on
the transaction; and
(3) the client gives informed consent, in a writing signed by the client, to the
essential terms of the transaction and the lawyer's role in the transaction,
including whether the lawyer is representing the client in the transaction.
iii. Before you make a deal w/a client ask these questions:
1. Are the terms fair to the client?
2. Have you explained the terms to the client clearly and in writing?
3. Have you advised the client in writing that she should get legal advice
about the deal from a lawyer not associated with you?
4. Has the client had a chance to get advice from another lawyer?
5. Has the client given informed consent in writing to the terms of the deal
and to your role in the deal?

79
iv. Do restrictions on lawyer client business deals apply to every lawyer-client
contract?
1. No, restrictions don’t apply to:
a. A contract for legal services, unless part or all of the fee is to be paid
by the transfer of property to the lawyer
b. a contract in which the client sells the lawyer some product or service
the client normally sells to others
v. Can a lawyer accept stock or stock options instead of cash as payment for
legal services?
1. A lawyer may be paid in the form of stock or stock options, but this type
of transaction requires compliance with Rule 1.8.
2. The problem is if a lawyer becomes a shareholder, the lawyer has a
personal financial stake in the entity that might be at odds with the
interests of the entity as a whole
vi. Rule 1.8 (c) Gifts from Clients
(c) A lawyer shall not solicit any substantial gift from a client, including a
testamentary gift, or prepare on behalf of a client an instrument giving the
lawyer or a person related to the lawyer any substantial gift unless the
lawyer or other recipient of the gift is related to the client. For purposes
of this paragraph, related persons include a spouse, child, grandchild,
parent, grandparent or other relative or individual with whom the lawyer
or the client maintains a close, familial relationship.
vii. Unsolicited Gifts
1. Rule 1.8 (c) doesn’t prohibit a lawyer from receiving unsolicited gifts from
a client. But, if the gift is too large, the client could later sue for its return.
I. Sexual Relations w/Clients and Adverse Attorneys
i. Rule 1.8 (j) Sexual Relations w/Clients
(j) A lawyer shall not have sexual relations with a client unless a
consensual sexual relationship existed between them when the client -
lawyer relationship commenced.
NOTE: Not imputed to other lawyers in the firm
ii. With Adverse Attorneys
1. Normally not okay unless client shave been informed, advised of possible
problems, and given informed consent Rule 1.7, Comment 11
2. NOTE: Not imputed to other lawyers in the firm
J. Imputed Conflicts to Other Lawyers
i. Rule 1.8 (k) Extension of 1.8(a) through (i) to other attorneys in a firm
(k) While lawyers are associated in a firm, a prohibition in the foregoing
paragraphs (a) through (i) that applies to any one of them shall apply to all of
them
ii. Rule 1.10 (a)
1. See above
K. Other Rules That May Come Up As a Result
1.4 Communication

80
8.3 Reporting Professional Misconduct
8.4 Misconduct
5.2 Responsibilities of a Subordinate Lawyer,
1.16(d) Termination stuff
1.10 (a) Imputation to other lawyers in a firm for other conflicts between a
client’s interests and a lawyer’s personal interests.

A. Regulation of Government Lawyers


i. The law governing lobbying
1. Lobbying Disclosure Act
a. Requires lawyers who lobby federal legislators to register
b. The ABA found many weak spots
ii. Conflict of Interest and “Revolving Door” Statutes
1. 18 USC 205, was designed to prevent the revolving door of legislators
becoming lobbyist by making them wait a certain number of years before
transitioning over.
B. Successive Conflicts of Former and Present Government Lawyers
i. Background
1. Special conflicts rules apply to lawyers who move from government
practice to private practice. MR 1.11.
2. At least two facts distinguish former government lawyers who move into
private practice from private-practice lawyers who have moved from
employment in one law firm to employment in another law firm.
a. First, government lawyers represent the government (or the public or
the particular agency within which they worked). As a result, in later
private practice, any representation of private parties against the
government might be seen as former-client conflicts. Because of the
breadth of the potential disqualification from representation, this
result must be ameliorated.
b. Second, as a government lawyer, the stakes of the appearance of
impropriety are raised. The possibility of abuses of either relationships
with former colleagues still in government practice or of confidential
government information creates greater concern regarding private
practice subsequent to government practice.
3. Both of these facts have effects on the formulation of the special conflicts
rules that apply to former government lawyers.
ii. Present and former are subject to 1.9(c)
iii. Rule 1.11 (e) Defining “matter” in the context of 1.11
(e) As used in this Rule, the term "matter" includes:

81
(1) any judicial or other proceeding, application, request for a ruling or
other determination, contract, claim, controversy, investigation, charge,
accusation, arrest or other particular matter involving a specific party or
parties, and
(2) any other matter covered by the conflict of interest rules of the
appropriate government agency.
C. Conflicts of Former Government Lawyers in Private Practice
i. General
1. When a lawyer at the firm is disqualified because of work then did while at
the government others at the firm will still be able to pick up the work.
2. We use 1.11 not 1.10 for this  1.11 imposes less stringent screening
requirements than 1.10.
ii. Exceptions
1. Law Otherwise Permits
2. Not a ‘Matter’: The rule defines what it means by a “matter.” It is only
later private representation in connection with a government service
“matter” that triggers a conflicts-of-interest analysis. A matter includes a
wide variety of instances and actions that engage the agency with a
particular party or parties. The definition excludes rule drafting and other
agency actions that have more general application.
3. Consent
iii. 1.19 (a) doesn’t apply, but 1.11(a)(2) does
iv. Rule 1.11(a) Evaluating Conflicts for Former Government Lawyers
(a) Except as law may otherwise expressly permit, a lawyer who has formerly
served as a public officer or employee of the government:
(1) is subject to Rule 1.9(c); and
(2) shall not otherwise represent a client in connection with a matter in
which the lawyer participated personally and substantially as a public
officer or employee, unless the appropriate government agency gives
its informed consent, confirmed in writing, to the representation.

Similar to the treatment of former judges, the


mere fact that a lawyer was employed in a particular agency while that
agency was involved in a matter does not disqualify the lawyer under
the conflicts rules. Only when the lawyer has participated personally
and substantially will the conflicts rule’s disqualification threshold
issues be satisfied.
Personal participation is just that: The rule does not
apply if the lawyer’s agency or department was involved in a matter,
only when the lawyer was involved.
Even personal participation does not implicate the
conflicts rule when the participation is insubstantial. Mere presence,
for example, at a staff meeting at which a matter was discussed or
participation in a minor, tangential aspect of a matter will not
implicate the conflicts rule.

82
v. Rule 1.11(b) Screening a Former Government Lawyer
(b) When a lawyer is disqualified from representation under paragraph (a), no
lawyer in a firm with which that lawyer is associated may knowingly
undertake or continue representation in such a matter unless:
(1) the disqualified lawyer is timely screened from any participation in
the matter and is apportioned no part of the fee therefrom; and
(2) written notice is promptly given to the appropriate government
agency to enable it to ascertain compliance with the provisions of this
rule.

vi. Rule 1.11(c) Confidential Government Information and Former


Government Attorney
(c) Except as law may otherwise expressly permit, a lawyer having
information that the lawyer knows is confidential government
information about a person acquired when the lawyer was a public officer
or employee, may not represent a private client whose interests are
adverse to that person in a matter in which the information could be used
to the material disadvantage of that person. As used in this Rule, the term
"confidential government information" means information that has been
obtained under governmental authority and which, at the time this Rule is
applied, the government is prohibited by law from disclosing to the public or
has a legal privilege not to disclose and which is not otherwise available to the
public. A firm with which that lawyer is associated may undertake or
continue representation in the matter only if the disqualified lawyer is
timely screened from any participation in the matter and is apportioned no
part of the fee therefrom.

Except when law otherwise expressly permits such


representation, a former government lawyer is also prohibited from
representing private parties who are adverse to parties about whom
the lawyer has confidential information gained in the government
practice that could be used against the adverse party. MR 1.11( c).
Example: Lawyer formerly worked for the IRS. In that capacity,
Lawyer was privy to confidential information about Taxpayer’s
finances and assets. In private practice, Client approaches Lawyer
requesting representation against Taxpayer in a complex matter that
involves the finances and assets of Taxpayer. Lawyer has a conflict of
interest and may not represent Client against Taxpayer unless
Taxpayer consents.

The private party’s interests are the ones at risk in such


a case. As such, it is the private party and not the agency that must
waive the conflict if the lawyer is to be permitted to proceed. Such a
waiver is, of course, highly unlikely in such an instance.

vii. Bases for Disqualification of Former Government Lawyers

83
1. Subsequent work involved a ‘matter’ in which the government lawyer was
“personally and substantially involved’ while in the government. (This can
be cured by agency consent) Rule 1.11(a) (type 1)
2. Subsequent work could involve use of ‘confidential government
information’ about a person known to the lawyer in a way that would
materially disadvantage that person. (This cannot be cured by consent)
1.11(c) (type 2)
D. Conflicts of Government Workers Who Formerly Worked in Private Practice
i. Rule 1.11 (d) Special Conflicts for Current Government Lawyers
(d) Except as law may otherwise expressly permit, a lawyer currently
serving as a public officer or employee:
(1) is subject to Rules 1.7 and 1.9; and
(2) shall not:
(i) participate in a matter in which the lawyer participated
personally and substantially while in private practice or
nongovernmental employment, unless the appropriate
government agency gives its informed consent, confirmed in
writing; or
(ii) negotiate for private employment with any person who is
involved as a party or as lawyer for a party in a matter in which the
lawyer is participating personally and substantially, except that a
lawyer serving as a law clerk to a judge, other adjudicative officer
or arbitrator may negotiate for private employment as permitted
by Rule 1.12(b) and subject to the conditions stated in Rule
1.12(b).
1. NOTE: Also 1.9(a) “the same of substantially related matter”

A. History of Judicial Ethics Codes in the United States


1. ABA Code of Judicial Conduct was enacted in 1972 designed to be a
model code for judges as the MRs did not cover this issue effectively.
B. Overview of the Model Code of Judicial Conduct (Canons 1-4)
1. Canon 1
a. uphold the independence, integrity, and impartiality of the judiciary,
and shall avoid impropriety and the appearance of impropriety
2. Canon 2
a. a judge shall perform the duties of judicial office impartially,
competently, and diligently
3. Canon 3
a. a judge shall conduct the judge's personal and extrajudicial activities to
minimize the risk of conflict with the obligations of judicial office
4. Canon 4
84
a. A judge or candidate for judicial office shall not engage in political or
campaign activity that is inconsistent with the independence, integrity
or impartiality of the judiciary
C. Impartiality and fairness; avoidance of bias, prejudice, and harassment
i. The canons
1. Prevent a judge from being a party to an organization that is
discriminatory
ii. Model Code of Judicial Fairness 3.6
1. Model Code of Judicial Conduct bars a judge from holding membership in
any organization that practices invidious discrimination on the basis of
sexual discrimination

2.
iii. CC: 2.4 External Influences on Judicial Conduct
(A) A judge shall not be swayed by public clamor or fear of criticism.
(B) A judge shall not permit family, social, political, financial, or other
interests or relationships to influence the judge’s judicial conduct or
judgment.
(C) A judge shall not convey or permit others to convey the impression that
any person or organization is in a position to influence the judge.
D. Ex Parte Communications
i. Circumstances where ex parte conduct is permissible
1. Scheduling and administrative purposes
2. A judge may receive written advice of a disinterested expert
3. A judge may consult with court staff and court officials, must take
reasonable measures to not receive information not in the record.
4. If the judge is overseeing settlement then with the parties consent she can
meet with them separately.
5. If ex parte communication is permitted by other laws.
ii. CC: 2.9 Ex Parte Communications
(A) A judge shall not initiate, permit, or consider ex parte communications, or
consider other communications made to the judge outside the presence of the
parties or their lawyers, concerning a pending* or impending matter,* except
as follows:
(1) When circumstances require it, ex parte communication for
scheduling, administrative, or emergency purposes, which does not
address substantive matters, is permitted, provided:

85
(a) the judge reasonably believes that no party will gain a
procedural, substantive, or tactical advantage as a result of the
ex parte communication; and
(b) the judge makes provision promptly to notify all other
parties of the substance of the ex parte communication, and
gives the parties an opportunity to respond.
(2) A judge may obtain the written advice of a disinterested expert on
the law applicable to a proceeding before the judge, if the judge gives
advance notice to the parties of the person to be consulted and the
subject matter of the advice to be solicited, and affords the parties a
reasonable opportunity to object and respond to the notice and to the
advice received.
(3) A judge may consult with court staff and court officials whose
functions are to aid the judge in carrying out the judge’s adjudicative
responsibilities, or with other judges, provided the judge makes
reasonable efforts to avoid receiving factual information that is not
part of the record, and does not abrogate the responsibility personally
to decide the matter.
(4) A judge may, with the consent of the parties, confer separately with
the parties and their lawyers in an effort to settle matters pending
before the judge.
(5) A judge may initiate, permit, or consider any ex parte
communication when expressly authorized by law* to do so.
(B) If a judge inadvertently receives an unauthorized ex parte communication
bearing upon the substance of a matter, the judge shall make provision
promptly to notify the parties of the substance of the communication and
provide the parties with an opportunity to respond.
(C) A judge shall not investigate facts in a matter independently, and shall
consider only the evidence presented and any facts that may properly be
judicially noticed.
(D) A judge shall make reasonable efforts, including providing appropriate
supervision, to ensure that this Rule is not violated by court staff, court
officials, and others subject to the judge’s direction and control.
E. Disqualification of Judges
i. General
1. Cannon Rule 2.11 “a Judge shall disqualify himself . . .”
2. In any proceeding in which the judge's impartiality might reasonably be
questioned.
3. There are illustrative list of such situations (583)
ii. CC: 2.11 Disqualification
(A) A judge shall disqualify himself or herself in any proceeding in which the
judge’s impartiality* might reasonably be questioned, including but not limited
to the following circumstances:
(1) The judge has a personal bias or prejudice concerning a party or a
party’s lawyer, or personal knowledge* of facts that are in dispute in the
proceeding.

86
(2) The judge knows* that the judge, the judge’s spouse or domestic
partner,* or a person within the third degree of relationship* to either of
them, or the spouse or domestic partner of such a person is:
(a) a party to the proceeding, or an officer, director, general partner,
managing member, or trustee of a party;
(b) acting as a lawyer in the proceeding;
(c) a person who has more than a de minimis* interest that could be
substantially affected by the proceeding; or
(d) likely to be a material witness in the proceeding.
(3) The judge knows that he or she, individually or as a fiduciary,* or the
judge’s spouse, domestic partner, parent, or child, or any other member of
the judge’s family residing in the judge’s household,* has an economic
interest* in the subject matter in controversy or in a party to the
proceeding.
(4) The judge knows or learns by means of a timely motion that a party, a
party’s lawyer, or the law firm of a party’s lawyer has within the previous
[insert number] year[s] made aggregate* contributions* to the judge’s
campaign in an amount that [is greater than $[insert amount] for an
individual or $[insert amount] for an entity] [is reasonable and appropriate
for an individual or an entity].
(5) The judge, while a judge or a judicial candidate,* has made a public
statement, other than in a court proceeding, judicial decision, or opinion,
that commits or appears to commit the judge to reach a particular result
or rule in a particular way in the proceeding or controversy.
(6) The judge:
(a) served as a lawyer in the matter in controversy, or was associated
with a lawyer who participated substantially as a lawyer in the matter
during such association;
(b) served in governmental employment, and in such capacity
participated personally and substantially as a lawyer or public official
concerning the proceeding, or has publicly expressed in such capacity an
opinion concerning the merits of the particular matter in controversy;
(c) was a material witness concerning the matter; or
(d) previously presided as a judge over the matter in another court.
(B) A judge shall keep informed about the judge’s personal and fiduciary
economic interests, and make a reasonable effort to keep informed about the
personal economic interests of the judge’s spouse or domestic partner and
minor children residing in the judge’s household.
(C) A judge subject to disqualification under this Rule, other than for bias or
prejudice under paragraph (A)(1), may disclose on the record the basis of the
judge’s disqualification and may ask the parties and their lawyers to consider,
outside the presence of the judge and court personnel, whether to waive
disqualification. If, following the disclosure, the parties and lawyers agree,
without participation by the judge or court personnel, that the judge should
not be disqualified, the judge may participate in the proceeding. The
agreement shall be incorporated into the record of the proceeding.

87
F. Conflicts Rules for Former Judges, Law Clerks, Arbitrators, and Mediators
i. Model Rule 1.12 Former Judge, Arbitrator, Mediator or Other Third-
Party Neutral
(a) Except as stated in paragraph (d), a lawyer shall not represent anyone in
connection with a matter in which the lawyer participated personally and
substantially as a judge or other adjudicative officer or law clerk to such a
person or as an arbitrator, mediator or other third-party neutral, unless all
parties to the proceeding give informed consent, confirmed in writing.
. The personal
and substantial participated standard in 1.11 is the same for
judges. See p 594

(b) A lawyer shall not negotiate for employment with any person who is
involved as a party or as lawyer for a party in a matter in which the lawyer is
participating personally and substantially as a judge or other adjudicative
officer or as an arbitrator, mediator or other third-party neutral. A lawyer
serving as a law clerk to a judge or other adjudicative officer may negotiate
for employment with a party or lawyer involved in a matter in which the clerk
is participating personally and substantially, but only after the lawyer has
notified the judge or other adjudicative officer.
1. . (p 595) A judge
cannot seek employment from a party in one of her cases (1.12(b))
Uses present tense, seems to imply that the bar does not survive
after the adjudication is done (could this still be a conflict?)

(c) If a lawyer is disqualified by paragraph (a), no lawyer in a firm with which


that lawyer is associated may knowingly undertake or continue representation
in the matter unless:
(1) the disqualified lawyer is timely screened from any participation in
the matter and is apportioned no part of the fee therefrom; and
(2) written notice is promptly given to the parties and any
appropriate tribunal to enable them to ascertain compliance with the
provisions of this rule.
2. . (p 595) Judges can be screened from
issues which they may have worked on while on the bench, it will
not necessarily be imputed on the firm.

(d) An arbitrator selected as a partisan of a party in a multimember arbitration


panel is not prohibited from subsequently representing that party.

A. Model Rule 3.2 Expediting Litigation


A lawyer shall make reasonable efforts to expedite litigation consistent with
the interests of the client.

88
B. Investigation Before Filing a Complaint
i. Rule 3.1 Meritorious Claims and Contentions
1. A lawyer shall not bring or defend a proceeding, or assert or controvert
an issue therein, unless there is a basis in law and fact for doing so that
is not frivolous, which includes a good faith argument for an
extension, modification or reversal of existing law. A lawyer for the
defendant in a criminal proceeding, or the respondent in a proceeding that
could result in incarceration, may nevertheless so defend the proceeding
as to require that every element of the case be established.
ii. FRCP 11
1. Very similar to 3.1, except FRCP 11 has a “safe harbor” provision. A
lawyer may withdraw allegedly frivolous pleadings within 21 days of
opposing counsel’s motion complaining of a violation without the lawyer
being sanctioned beyond having to pay attorneys’ fees the opposing party
incurred for making the motion.
2. If a lawyer withdrew pursuant to safe harbor provision of FRCP 11, it’s
unlikely the bar counsel would file a charge against a lawyer for a frivolous
suit
3. However, a lawyer who initiates a federal court lawsuit in good faith and
later finds out (i.e. through discovery) that the lawsuit is groundless may
be subject to FRCP 11 sanctions
C. Truth and Falsity in Litigation
i. Rule 3.3 Candor Toward the Tribunal
(a) A lawyer shall not knowingly:
(1) make a false statement of fact or law to a tribunal or fail to correct
a false statement of material fact or law previously made to the tribunal
by the lawyer;
(2) fail to disclose to the tribunal legal authority in the controlling
jurisdiction known to the lawyer to be directly adverse to the position of
the client and not disclosed by opposing counsel; or
(3) offer evidence that the lawyer knows to be false. If a lawyer, the
lawyer’s client, or a witness called by the lawyer, has offered material
evidence and the lawyer comes to know of its falsity, the lawyer shall
take reasonable remedial measures, including, if necessary,
disclosure to the tribunal. A lawyer may refuse to offer evidence, other
than the testimony of a defendant in a criminal matter, that the lawyer
reasonably believes is false.
(b) A lawyer who represents a client in an adjudicative proceeding and who
knows that a person intends to engage, is engaging or has engaged in
criminal or fraudulent conduct related to the proceeding shall take
reasonable remedial measures, including, if necessary, disclosure to the
tribunal.
(c) The duties stated in paragraphs (a) and (b) continue to the conclusion of
the proceeding, and apply even if compliance requires disclosure of
information otherwise protected by Rule 1.6.

89
(d) In an ex parte proceeding, a lawyer shall inform the tribunal of all
material facts known to the lawyer that will enable the tribunal to make an
informed decision, whether or not the facts are adverse.
ii. Rule 8.4 Misconduct
It is professional misconduct for a lawyer to:
(a) violate or attempt to violate the Rules of Professional Conduct, knowingly
assist or induce another to do so, or do so through the acts of another;
(b) commit a criminal act that reflects adversely on the lawyer's honesty,
trustworthiness or fitness as a lawyer in other respects;
(c) engage in conduct involving dishonesty, fraud, deceit or misrepresentation;
iii. Rule 3.4 (e): Lawyer’s Duties in Trial
A lawyer shall not:
(e) in trial, allude to any matter that the lawyer does not reasonably
believe is relevant or that will not be supported by admissible
evidence, assert personal knowledge of facts in issue except when
testifying as a witness, or state a personal opinion as to the justness of a
cause, the credibility of a witness, the culpability of a civil litigant or
the guilt or innocence of an accused; or
iv. Chart for Which Rules Apply When….
1. Chart
Who Situation (court administrative Lawyer’s Obligation
Might Lie hearing or discovery)
or
Deceive
Lawyer Lawyer is considering making a false Lawyer must not do it. Rules 3.3(a)(1),
statement of fact or law to a judge 8.4
Client Lawyer knows that her client is Lawyer must counsel client and refrain
considering testifying falsely in court from asking client questions that would
or in a deposition elicit the false testimony. Rule 3.3(a)(3)
Civil Lawyer suspects but does not know If lawyer reasonably believes it is false,
Client or that panned testimony may be false; lawyer must refuse to offer the testimony
Witness in witness is not a criminal defendant – or may allow it. Rule 3.3(a)(3)
any
Proceeding
Criminal Lawyers suspects but does not know When defendant insists on testifying,
Defendant that planned testimony may be false; lawyer must allow it, if lawyer reasonably
witness is a criminal defendant believes but does not know it is false.
Rule 3.3(a)(3)
Client or Lawyer knows that her client or Lawyer must counsel client to correct the
Witness other witness has testified falsely record consider withdrawing; correct
during direct or cross-examination record if necessary to undo the effect of
the false evidence Rules. 3.3(b) and (c),
and Comment 10
Client or Witness has misled the court by Lawyer may have duty counsel client and
Witness making statements that are literally correct the record. Rules 3.3(b) and
true but deceptive 8.4(c)
Lawyer Lawyer knows of directly adverse Lawyer must bring it to the court’s
controlling legal authority that has attention (and may distinguish or explain
why it is not authoritative). Rule 3.3(a)(2)

90
not been disclosed by opposing
counsel

Lawyer Lawyer knows of facts adverse to No need to disclose unless the


client’s interest, not requested in proceeding is ex parte. Rule 3.3(d)
discovery or required to be disclosed
by a court rule
v. A Lawyer’s Duty if a Client Intends to Give Testimony the Lawyer Knows is
False Nix v. Whiteside
1. An attorney is precluded from taking steps or in any way assisting the
client in presenting false evidence or otherwise violating the law and
should admonish the client about his duty to inform the court if the
defendant perjures himself on the stand
vi. “Knowledge” of a Client’s Intent to Give False Testimony
1. In a civil case, if the lawyer reasonably believes the testimony is false, the
lawyer may refuse to offer it. Alternatively, the lawyer may give the cl ient
or other witness the benefit of the doubt and agree to present the
testimony.
2. Only if the lawyer ACTUALLY KNOWS the witness is going to testify
falsely must the lawyer refrain from offering.
3. If the person testifying is a criminal defendant, the lawyer MUST NOT
present false testimony if the lawyer knows that it is false, BUT MUST
allow testimony if they don’t know but ONLY REASONABLY
BELIEVES it to be false.
vii. Can lawyers protect themselves and their clients by deliberately not
knowing all the facts?
1. Up to a point. There are several obligations implicated that depend on
how much the lawyer knows:
1.2(d) Duty not to collaborate in a client’s crime
1.13 Duty to report misconduct of corporate officials to client
corporation
3.3 (b) Duty to correct client’s testimony the lawyer knows was false
viii. A lawyer’s duties if a client intends to mislead the court without lying 
Bronston v. United States
1. Under the Bronston standard, evasions and half-truths are not perjury
even if the speaker intends to mislead.
2. *But, lawyers must take care not to violate the disciplinary rules regarding
honesty, which could be violated by statements or conduct that is
misleading but not perjurious.
ix. False Impressions Created by Lawyers During Litigation
1. See OJ Simpson, The Drug Test Hypos
2. Relevant Rules 3.3, 3.4, 4.1
x. Preparing Witnesses Rule 3.4(b), (f)
A lawyer shall not:
(b) falsify evidence, counsel or assist a witness to testify falsely, or offer
an inducement to a witness that is prohibited by law;

91
(f) request a person other than a client to refrain from voluntarily giving
relevant information to another party unless:
(1) the person is a relative or an employee or other agent of a
client; and
(2) the lawyer reasonably believes that the person's interests will
not be adversely affected by refraining from giving such
information.
D. Concealment of Physical Evidence and Documents
i. Duties of criminal defense lawyers with respect to evidence of crimes
1. In criminal cases, prosecutors may not use discovery to obtain
information from defendants. BUT, law enforcement officials may obtain
search warrants to look for physical evidence.
2. Relevant Rule 3.4 (a)
ii. Rule 3.4(a) A Lawyer Shall Not Unlawfully Obstruct
A lawyer shall not…. unlawfully obstruct another party' s access to evidence
or unlawfully alter, destroy or conceal a document or other material having
potential evidentiary value. A lawyer shall not counsel or assist another
person to do any such act;
1. Comment [2] Documents and other items of evidence are often essential
to establish a claim or defense. Subject to evidentiary privileges, the right
of an opposing party, including the government, to obtain evidence
through discovery or subpoena is an important procedural right. The
exercise of that right can be frustrated if relevant material is altered,
concealed or destroyed.
Applicable law in many jurisdictions makes it an offense to destroy
material for purpose of impairing its availability in a pending proceeding
or one whose commencement can be foreseen. Falsifying evidence is
also generally a criminal offense. Paragraph (a) applies to evidentiary
material generally, including computerized information.
Applicable law may permit a lawyer to take temporary possession of
physical evidence of client crimes for the purpose of conducting a
limited examination that will not alter or destroy material
characteristics of the evidence. In such a case, applicable law may
require the lawyer to turn the evidence over to the police or other
prosecuting authority, depending on the circumstances.
NOTE:
State v. Olwell . A lawyer must turn over evidence after
inspecting and testing it within a reasonable period of time
In re Rider. A lawyer may not conceal evidence. Specifically, the
lawyer moved evidence to prevent a client from disposing of it
because moving the evidence helps conceal the evidence. The
attorney made himself “an accessory after the fact” to the
robbery.
People v. Meredith . A client told his lawyer where he put the
wallet (he tried to burn) of the guy he helped rob and kill. The
lawyer sent their investigator to find the wallet and turned it over
to police and withdrew as counsel. The client appealed and said
92
the investigator’s testimony violated attorney-client privilege, but
the Court disagreed because when the investigator removed the
wallet, the privilege was partly abrogated since when the
investigator removed the wallet, the investigator prevented the
police from finding it in the trash and testifying about the find.
The only way to prove the location of the wallet was to call the
investigator. Furthermore, the prosecutor protected what was left
of the privilege by not asking the investigator who hired him.
2. “Potential evidentiary value” Vague and unclear.
Professor Gregory Sisk argues a lawyer shouldn’t be punished if they
advise destruction of evidence “under circumstances where an
investigation was not reasonably anticipated.” However, in situations like
where a client gives a lawyer the gun they used to kill someone, the lawyer
should reasonably anticipate an investigation.
Also, even if a prosecutor accuses a lawyer of destroying or concealing
evidence, the lawyer will cite attorney-client or work-product privileges…
3. NOTE: Documents aren’t treated differently than physical evidence,
especially if the document is the only copy The duty not to conceal applies
to documents (see Morrell v. State).
iii. Sarbanes-Oxley law  Lawyers w/Posession of Documentary
Evidence
1. Whoever knowingly alters, destroys, mutilates, conceals, covers up,
falsifies, or makes a false entry in any record, document, or tangible object
with the intent to impede, obstruct, or influence the investigation or
proper administration of any matter within the juris-diction of any
department or agency of the United States or any case filed under title 11,
or in relation to or contemplation of any such matter or case, shall be
fined under this title, imprisoned not more than 20 years, or both
iv. Civil Cases Lmtd Obligation to Reveal 3.4 (a) and 3.4 (c)
A lawyer shall not….
(a) unlawfully obstruct another party' s access to evidence or unlawfully
alter, destroy or conceal a document or other material having potential
evidentiary value. A lawyer shall not counsel or assist another person
to do any such act; (SEE COMMENTS ABOVE)

(c) knowingly disobey an obligation under the rules of a tribunal
except for an open refusal based on an assertion that no valid obligation
exists;
v. Rule 3.4 (d) A lawyer’s duties in responding to discovery requests
A lawyer shall not:
(d) in pretrial procedure, make a frivolous discovery request or fail to
make reasonably diligent effort to comply with a legally proper
discovery request by an opposing party;
E. The Duty to Disclose Adverse Legal Authority
i. Rule 3.3(a)(2)
(a) A lawyer shall not knowingly:

93

(2) fail to disclose to the tribunal legal authority in the controlling
jurisdiction known to the lawyer to be directly adverse to the position of
the client and not disclosed by opposing counsel; or
F. Disclosures in Ex Parte Proceedings
i. Rule 3.3(d)
(d) In an ex parte proceeding, a lawyer shall inform the tribunal of all
material facts known to the lawyer that will enable the tribunal to make an
informed decision, whether or not the facts are adverse.
NOTE: The duty overrides the obligation to protect confidences
G. Improper Influences on Judges and Juries
i. Rule 3.5 Impartiality and Decorum of the Tribunal
A lawyer shall not:
(a) seek to influence a judge, juror, prospective juror or other official by
means prohibited by law;
(b) communicate ex parte with such a person during the proceeding unless
authorized to do so by law or court order;
(c) communicate with a juror or prospective juror after discharge of the
jury if:
(1) the communication is prohibited by law or court order;
(2) the juror has made known to the lawyer a desire not to
communicate; or
(3) the communication involves misrepresentation, coercion,
duress or harassment; or
(d) engage in conduct intended to disrupt a tribunal.
ii. Ex-Parte Communications w/Judges
1. See Rule 3.5 (b)
2. However, a lawyer may call judges’ secretaries or clerks to make routine
procedural inquiries about pending cases without having to notify the
other parties. But they may not discuss the substance of cases with judges
or their secretaries or clerks unless all parties’ representatives have been
notified and given an opportunity to participate
iii. Lawyer’s Comments to the Press Rule 3.6 Trial Publicity
(a) A lawyer who is participating or has participated in the investigation or
litigation of a matter shall not make an extrajudicial statement that the
lawyer knows or reasonably should know will be disseminated by means of
public communication and will have a substantial likelihood of materially
prejudicing an adjudicative proceeding in the matter.
COMMENT 4: Paragraph (b) identifies specific matters about
which a lawyer's statements would not ordinarily be considered
to present a substantial likelihood of material prejudice, and
should not in any event be considered prohibited by the general
prohibition of paragraph (a). Paragraph (b) is not intended to be
an exhaustive listing of the subjects upon which a lawyer may
make a statement, but statements on other matters may be
subject to paragraph (a).
94
(b) Notwithstanding paragraph (a), a lawyer may state:
(1) the claim, offense or defense involved and, except when
prohibited by law, the identity of the persons involved;
(2) information contained in a public record;
(3) that an investigation of a matter is in progress;
(4) the scheduling or result of any step in litigation;
(5) a request for assistance in obtaining evidence and information
necessary thereto;
(6) a warning of danger concerning the behavior of a person involved,
when there is reason to believe that there exists the likelihood of
substantial harm to an individual or to the public interest; and
(7) in a criminal case, in addition to subparagraphs (1) through (6):
(i) the identity, residence, occupation and family status of the
accused;
(ii) if the accused has not been apprehended, information
necessary to aid in apprehension of that person;
(iii) the fact, time and place of arrest; and
(iv) the identity of investigating and arresting officers or agencies
and the length of the investigation.
COMMENT 5: There are, on the other hand, certain subjects
that are more likely than not to have a material prejudicial effect
on a proceeding, particularly when they refer to a civil matter
triable to a jury, a criminal matter, or any other proceeding that
could result in incarceration. These subjects relate to:
(1) the character, credibility, reputation or criminal record
of a party, suspect in a criminal investigation or witness,
or the identity of a witness, or the expected testimony of
a party or witness;
(2) in a criminal case or proceeding that could result in
incarceration, the possibility of a plea of guilty to the
offense or the existence or contents of any confession,
admission, or statement given by a defendant or suspect
or that person's refusal or failure to make a statement;
(3) the performance or results of any examination or test
or the refusal or failure of a person to submit to an
examination or test, or the identity or nature of physical
evidence expected to be presented;
(4) any opinion as to the guilt or innocence of a
defendant or suspect in a criminal case or proceeding that
could result in incarceration;
(5) information that the lawyer knows or reasonably
should know is likely to be inadmissible as evidence in a
trial and that would, if disclosed, create a substantial risk
of prejudicing an impartial trial; or
(6) the fact that a defendant has been charged with a
crime, unless there is included therein a statement
explaining that the charge is merely an accusation and
that the defendant is presumed innocent until and unless
proven guilty.

95
(c) Notwithstanding paragraph (a), a lawyer may make a statement that a
reasonable lawyer would believe is required to protect a client from the
substantial undue prejudicial effect of recent publicity not initiated by
the lawyer or the lawyer's client. A statement made pursuant to this
paragraph shall be limited to such information as is necessary to mitigate the
recent adverse publicity.
(d) No lawyer associated in a firm or government agency with a lawyer subject
to paragraph (a) shall make a statement prohibited by paragraph (a).
iv. Impeachment of Truthful Witnesses
1. The model rules don’t explicitly address this problem.
2. Rule 4.4(a) says a lawyer may not use means that “have no substantial
purpose other than to embarrass, delay, or burden a third person…”
3. No reported disciplinary cases….
v. Statements by Lawyers During Jury Trials  Rule 3.7 Advocate Witness
(a) A lawyer shall not act as advocate at a trial in which the lawyer is likely to
be a necessary witness unless:
(1) the testimony relates to an uncontested issue;
COMMENT [3] …Paragraph (a)(1) recognizes that if the testimony
will be uncontested, the ambiguities in the dual role are purely
theoretical.
(2) the testimony relates to the nature and value of legal services
rendered in the case; or
COMMENT [3] … Paragraph (a)(2) recognizes that where the
testimony concerns the extent and value of legal services rendered in
the action in which the testimony is offered, permitting the lawyers to
testify avoids the need for a second trial with new counsel to resolve
that issue. Moreover, in such a situation the judge has firsthand
knowledge of the matter in issue; hence, there is less dependence on
the adversary process to test the credibility of the testimony.
(3) disqualification of the lawyer would work substantial hardship on
the client.
COMMENT [1] Combining the roles of advocate and witness can
prejudice the tribunal and the opposing party and can also involve a
conflict of interest between the lawyer and client.
COMMENT [4] Apart from these two exceptions, paragraph (a)(3)
recognizes that a balancing is required between the interests of the
client and those of the tribunal and the opposing party. Whether the
tribunal is likely to be misled or the opposing party is likely to suffer
prejudice depends on the nature of the case, the importance and
probable tenor of the lawyer's testimony, and the probability that the
lawyer's testimony will conflict with that of other witnesses. Even if
there is risk of such prejudice, in determining whether the lawyer
should be disqualified, due regard must be given to the effect of
disqualification on the lawyer's client. It is relevant that one or both
parties could reasonably foresee that the lawyer would probably be a
witness. The conflict of interest principles stated in Rules 1.7, 1.9 and
1.10 have no application to this aspect of the problem.

96
(b) A lawyer may act as advocate in a trial in which another lawyer in the
lawyer's firm is likely to be called as a witness unless precluded from
doing so by Rule 1.7 or Rule 1.9.
COMMENT [6] In determining if it is permissible to act as advocate in
a trial in which the lawyer will be a necessary witness, the lawyer must
also consider that the dual role may give rise to a conflict of interest
that will require compliance with Rules 1.7 or 1.9. For example, if
there is likely to be substantial conflict between the testimony of the
client and that of the lawyer the representation involves a conflict of
interest that requires compliance with Rule 1.7. This would be true
even though the lawyer might not be prohibited by paragraph (a) from
simultaneously serving as advocate and witness because the lawyer's
disqualification would work a substantial hardship on the client.
Similarly, a lawyer who might be permitted to simultaneously serve as
an advocate and a witness by paragraph (a)(3) might be precluded from
doing so by Rule 1.9. The problem can arise whether the lawyer is
called as a witness on behalf of the client or is called by the opposing
party. Determining whether or not such a conflict exists is primarily
the responsibility of the lawyer involved. If there is a conflict of
interest, the lawyer must secure the client's informed consent,
confirmed in writing. In some cases, the lawyer will be precluded from
seeking the client's consent. See Rule 1.7. See Rule 1.0(b) for the
definition of "confirmed in writing" and Rule 1.0(e) for the definition
of "informed consent."
COMMENT [7] Paragraph (b) provides that a lawyer is not
disqualified from serving as an advocate because a lawyer with whom
the lawyer is associated in a firm is precluded from doing so by
paragraph (a). If, however, the testifying lawyer would also be
disqualified by Rule 1.7 or Rule 1.9 from representing the client in the
matter, other lawyers in the firm will be precluded from representing
the client by Rule 1.10 unless the client gives informed consent under
the conditions stated in Rule 1.7.
vi. Statements by Lawyers During Jury Trials  Rule 3.4 (e) Allusion to
Irrelevant or Unsupported Matter
A lawyer shall not:
(e) in trial, allude to any matter that the lawyer does not reasonably
believe is relevant or that will not be supported by admissible
evidence, assert personal knowledge of facts in issue except when
testifying as a witness, or state a personal opinion as to the justness of a
cause, the credibility of a witness, the culpability of a civil litigant or
the guilt or innocence of an accused; …
H. Lawyers’ Duties in Non-Adjudicative Proceedings
i. Rule 3.9 Advocate in Nonadjudicative Proceedings
1. A lawyer representing a client before a legislative body or
administrative agency in a nonadjudicative proceeding shall disclose that
the appearance is in a representative capacity and shall conform to the
provisions of Rules 3.3(a) through (c), 3.4(a) through (c), and 3.5.
ii. Application of Rule 3.9

97
1.

A. Deception of Third Persons


i. Rule 4.1 Truthfulness In Statements To Others
In the course of representing a client a lawyer shall not knowingly:
(a) make a false statement of material fact or law to a third person; or
(b) fail to disclose a material fact to a third person when disclosure is necessary to
avoid assisting a criminal or fraudulent act by a client, unless disclosure is
prohibited by Rule 1.6.
ii. Comment [1] of Rule 4.1 Misrepresentation
[1] A lawyer is required to be truthful when dealing with others on a
client’s behalf, but generally has no affirmative duty to inform an
opposing party of relevant facts.
A misrepresentation can occur if the lawyer incorporates or affirms
a statement of another person that the lawyer knows is false.
Misrepresentations can also occur by partially true but misleading
statements or omissions that are the equivalent of affirmative false
statements.

98
For dishonest conduct that does not amount to a false statement or
for misrepresentations by a lawyer other than in the course of
representing a client, see Rule 8.4.
iii. Comment [2] of Rule 4.1  Statements of Fact; duties of truthfulness in
negotiation
[2] This Rule refers to statements of fact. Whether a particular statement
should be regarded as one of fact can depend on the circumstances.
Under generally accepted conventions in negotiation, certain types of
statements ordinarily are not taken as statements of material fact.
Estimates of price or value placed on the subject of a transaction and
a party’s intentions as to an acceptable settlement of a claim are
ordinarily in this category, and so is the existence of an undisclosed
principal except where nondisclosure of the principal would constitute
fraud.
Lawyers should be mindful of their obligations under applicable law to
avoid criminal and tortious misrepresentation.
iv. Comment [3] of Rule 4.1  Crime or Fraud by Client
[3] Under Rule 1.2(d), a lawyer is prohibited from counseling or assisting
a client in conduct that the lawyer knows is criminal or fraudulent.
Paragraph (b) states a specific application of the principle set forth in
Rule 1.2(d) and addresses the situation where a client’s crime or
fraud takes the form of a lie or misrepresentation.
Ordinarily, a lawyer can avoid assisting a client’s crime or fraud by
withdrawing from the representation.
Sometimes it may be necessary for the lawyer to give notice of the
fact of withdrawal and to disaffirm an opinion, document,
affirmation or the like.
In extreme cases, substantive law may require a lawyer to disclose
information relating to the representation to avoid being deemed to
have assisted the client’s crime or fraud.
If the lawyer can avoid assisting a client’s crime or fraud only by
disclosing this information, then under paragraph (b) the lawyer is
required to do so, unless the disclosure is prohibited by Rule 1.6.
v. Lawyers’ duties of truthfulness in fact investigation
1. In some cases, a lawyer or their investigator may believe they can only
obtain required information by misrepresenting their identity or the
purpose of their inquiry
2. See pp 692-695
3. The issue of undercover investigations by nongovernment lawyers remains
unsettled.
4. Friend requests on Facebook See pp 696-697
vi. Receipt of inadvertently transmitted information, including metadata
1. See Rule 4.4 (b) and Comment [2] of Rule 4.4
vii. Rule 4.4 Respect For Rights Of Third Persons
(a) In representing a client, a lawyer shall not use means that have no substantial
purpose other than to embarrass, delay, or burden a third person, or use methods
of obtaining evidence that violate the legal rights of such a person.
99
Comment [1] Responsibility to a client requires a lawyer to
subordinate the interests of others to those of the client, but that
responsibility does not imply that a lawyer may disregard the
rights of third persons. It is impractical to catalogue all such rights,
but they include legal restrictions on methods of obtaining
evidence from third persons and unwarranted intrusions into
privileged relationships, such as the client-lawyer relationship.

(b) A lawyer who receives a document or electronically stored information relating


to the representation of the lawyer's client and knows or reasonably should know
that the document or electronically stored information was inadvertently sent shall
promptly notify the sender.
Comment [2] Paragraph (b) recognizes that lawyers sometimes
receive a document or electronically stored information that was
mistakenly sent or produced by opposing parties or their lawyers. A
document or electronically stored information is inadvertently sent
when it is accidentally transmitted, such as when an email or letter
is misaddressed or a document or electronically stored information is
accidentally included with information that was intentionally
transmitted. If a lawyer knows or reasonably should know that
such a document or electronically stored information was sent
inadvertently, then this Rule requires the lawyer to promptly
notify the sender in order to permit that person to take protective
measures. Whether the lawyer is required to take additional
steps, such as returning the document or electronically stored
information, is a matter of law beyond the scope of these Rules,
as is the question of whether the privileged status of a document or
electronically stored information has been waived. Similarly, this Rule
does not address the legal duties of a lawyer who receives a
document or electronically stored information that the lawyer knows
or reasonably should know may have been inappropriately
obtained by the sending person. For purposes of this Rule,
‘‘document or electronically stored information’’ includes, in addition
to paper documents, email and other forms of electronically stored
information, including embedded data (commonly referred to as
“metadata”), that is subject to being read or put into readable
form. Metadata in electronic documents creates an obligation under
this Rule only if the receiving lawyer knows or reasonably should
know that the metadata was inadvertently sent to the receiving
lawyer.
Comment [3] Some lawyers may choose to return a document or
delete electronically stored information unread, for example, when
the lawyer learns before receiving it that it was inadvertently sent.
Where a lawyer is not required by applicable law to do so, the
decision to voluntarily return such a document or delete electronically
stored information is a matter of professional judgment ordinarily
reserved to the lawyer. See Rules 1.2 and 1.4.
B. Restrictions on Contact w/Represented Persons

100
i. Rule 4.2 Communication With Person Represented By Counsel
In representing a client, a lawyer shall not communicate about the subject of the
representation with a person the lawyer knows to be represented by another lawyer
in the matter, unless the lawyer has the consent of the other lawyer or is authorized
to do so by law or a court order.
ii. Comments [2] and [3] of Rule 4.2  Applicability of the Rule
[2] This Rule applies to communications with any person who is
represented by counsel concerning the matter to which the
communication relates.
[3] The Rule applies even though the represented person initiates or
consents to the communication. A lawyer must immediately terminate
communication with a person if, after commencing communication, the
lawyer learns that the person is one with whom communication is not
permitted by this Rule.
iii. Comment [4] of Rule 4.2  Rule Doesn’t Apply to…
[4] This Rule does not prohibit communication with a represented person,
or an employee or agent of such a person, concerning matters outside the
representation. For example, the existence of a controversy between a
government agency and a private party, or between two organizations,
does not prohibit a lawyer for either from communicating with nonlawyer
representatives of the other regarding a separate matter. Nor does this
Rule preclude communication with a represented person who is seeking
advice from a lawyer who is not otherwise representing a client in the
matter. A lawyer may not make a communication prohibited by this Rule
through the acts of another. See Rule 8.4(a). Parties to a matter may
communicate directly with each other, and a lawyer is not prohibited from
advising a client concerning a communication that the client is legally
entitled to make. Also, a lawyer having independent justification or legal
authorization for communicating with a represented person is permitted
to do so.
iv. Comment [7] of Rule 4.2  Representing Businesses
[7] In the case of a represented organization, this Rule prohibits
communications with a constituent of the organization who supervises,
directs or regularly consults with the organization’s lawyer concerning the
matter or has authority to obligate the organization with respect to the
matter or whose act or omission in connection with the matter may be
imputed to the organization for purposes of civil or criminal liability.
Consent of the organization’s lawyer is not required for communication
with a former constituent. If a constituent of the organization is
represented in the matter by his or her own counsel, the consent by that
counsel to a communication will be sufficient for purposes of this Rule.
Compare Rule 3.4(f). In communicating with a current or former
constituent of an organization, a lawyer must not use methods of
obtaining evidence that violate the legal rights of the organization. See
Rule 4.4.
C. Restrictions on Contact with Unrepresented Persons
i. Rule 4.3 Dealing With Unrepresented Person

101
In dealing on behalf of a client with a person who is not represented by counsel, a
lawyer shall not state or imply that the lawyer is disinterested. When the
lawyer knows or reasonably should know that the unrepresented person
misunderstands the lawyer’s role in the matter, the lawyer shall make
reasonable efforts to correct the misunderstanding. The lawyer shall not give
legal advice to an unrepresented person, other than the advice to secure
counsel, if the lawyer knows or reasonably should know that the interests of such a
person are or have a reasonable possibility of being in conflict with the
interests of the client.
Comment [2] The Rule distinguishes between situations involving
unrepresented persons whose interests may be adverse to those of the
lawyer’s client and those in which the person’s interests are not in conflict
with the client’s. In the former situation, the possibility that the lawyer will
compromise the unrepresented person’s interests is so great that the Rule
prohibits the giving of any advice, apart from the advice to obtain counsel.
Whether a lawyer is giving impermissible advice may depend on the
experience and sophistication of the unrepresented person, as well as the
setting in which the behavior and comments occur. This Rule does not
prohibit a lawyer from negotiating the terms of a transaction or settling a
dispute with an unrepresented person. So long as the lawyer has explained
that the lawyer represents an adverse party and is not representing the
person, the lawyer may inform the person of the terms on which the lawyer's
client will enter into an agreement or settle a matter, prepare documents that
require the person's signature and explain the lawyer's own view of the
meaning of the document or the lawyer's view of the underlying legal
obligations.
ii. Respect for the Rights of Third Persons

D. Duties of Prosecutors
i. Undercover Investigations and Rules 4.1 and 4.2
1. There is little case law interpreting Rule 4.1 in application to criminal
investigations, but numerous cases have involved alleged violations of
Rule4.2 by prosecutors or delegates.
2. Supreme Court of MN referenced Comment 5 of Rule 4.2 in a decision:
a. [5] Communications authorized by law may include communications
by a lawyer on behalf of a client who is exercising a constitutional or
other legal right to communicate with the government.
Communications authorized by law may also include investigative
activities of lawyers representing governmental entities, directly or
through investigative agents, prior to the commencement of criminal
or civil enforcement proceedings. When communicating with the
accused in a criminal matter, a government lawyer must comply with
this Rule in addition to honoring the constitutional rights of the
accused. The fact that a communication does not violate a state or
federal constitutional right is insufficient to establish that the
communication is permissible under this Rule.
ii. Rule 3.8 Special Responsibilities Of A Prosecutor (emphasis on (a)(d)(f)(g)(h))
The prosecutor in a criminal case shall:
102
(a) refrain from prosecuting a charge that the prosecutor knows is not
supported by probable cause;
(b) make reasonable efforts to assure that the accused has been advised of the right
to, and the procedure for obtaining, counsel and has been given reasonable
opportunity to obtain counsel;
(c) not seek to obtain from an unrepresented accused a waiver of important pretrial
rights, such as the right to a preliminary hearing;
(d) make timely disclosure to the defense of all evidence or
information known to the prosecutor that tends to negate the guilt of
the accused or mitigates the offense, and, in connection with
sentencing, disclose to the defense and to the tribunal all unprivileged
mitigating information known to the prosecutor, except when the
prosecutor is relieved of this responsibility by a protective order of the
tribunal;
(e) not subpoena a lawyer in a grand jury or other criminal proceeding to present evidence
about a past or present client unless the prosecutor reasonably believes:
(1) the information sought is not protected from disclosure by any applicable
privilege;
(2) the evidence sought is essential to the successful completion of an ongoing
investigation or prosecution; and
(3) there is no other feasible alternative to obtain the information;
(f) except for statements that are necessary to inform the public of the
nature and extent of the prosecutor's action and that serve a legitimate
law enforcement purpose, refrain from making extrajudicial comments
that have a substantial likelihood of heightening public condemnation
of the accused and exercise reasonable care to prevent investigators,
law enforcement personnel, employees or other persons assisting or
associated with the prosecutor in a criminal case from making an
extrajudicial statement that the prosecutor would be prohibited from
making under Rule 3.6 or this Rule.
(g) When a prosecutor knows of new, credible and material evidence
creating a reasonable likelihood that a convicted defendant did not
commit an offense of which the defendant was convicted, the
prosecutor shall:
(1) promptly disclose that evidence to an appropriate court or
authority, and
(2) if the conviction was obtained in the prosecutor’s jurisdiction,
(i) promptly disclose that evidence to the defendant unless a
court authorizes delay, and
(ii) undertake further investigation, or make reasonable
efforts to cause an investigation, to determine whether the
defendant was convicted of an offense that the defendant did
not commit.
(h) When a prosecutor knows of clear and convincing evidence
establishing that a defendant in the prosecutor’s jurisdiction was
convicted of an offense that the defendant did not commit, the
prosecutor shall seek to remedy the conviction.

103
A. Unmet Need for Legal Services
i. General
1. Wealthiest ⅕ of the population are always able to afford legal services
2. Each year about half of low-middle income households face a major legal
issue
3. However, an overwhelming of them do not preclude the issue or use the
legal system
4. Only 39% of middle income and 29% of low income households used the
legal system
B. Right to Counsel for Indigent Litigants
i. Criminal defendants (753)
1. Until the decision in Gideon v. Wainwright, 372 U.S. 335 (1963),
provision of representation for indigent criminal defendants was a hit -or-
miss matter, which some states had and others had not. The Supreme
Court, in determining that the Sixth Amendment right to counsel requires
that counsel be appointed in criminal matters for indigent defendants,
generated the impetus for public defender agencies and court appointment
systems for criminal representation in places without a public defender
agency.
2. Spend 5.3 billion annually on criminal defense services.
3. 2% of what is spent to police and prosecutors
4. Three ways the state funds defense work of indigent persons
5. Funding public defender's offices
6. Contracts with lawyers
7. Appointments from the bench
ii. Rule 6.2 Accepting Appointments
A lawyer shall not seek to avoid appointment by a tribunal to represent a person
except for good cause, such as:
(a) representing the client is likely to result in violation of the Rules of
Professional Conduct or other law;
(b) representing the client is likely to result in an unreasonable financial
burden on the lawyer; or
(c) the client or the cause is so repugnant to the lawyer as to be likely to
impair the client-lawyer relationship or the lawyer's ability to represent the
client.

iii. Parties in Civil and Administrative Proceedings


1. People who need to go to court to resolve family, landlord-tenant,
consumer, or other “civil” problems don’t have a right to receive court -
appointed counsel.
2. This includes immigration proceedings
3. The courts have consistently rejected right to counsel claims in civil cases
under Due Process Clause

104
C. Civil Legal Aid
i. General (Legal Services Corporation)
1. No comparable right to counsel exists for most civil cases. As such, states
are not required to provide legal service for the poor in most civil matters.
Nonetheless, through the federal funding of the Legal Services
Corporation and additional funding by states and localities, legal aid or
legal services offices exist to serve a portion of the population that would
otherwise be unable to afford to retain a lawyer in civil matters. Because
the funding is inadequate to the task, the continued unserved public need
is substantial
ii. Conflicts of Interest for Legal Services Corporations
1. There are limitations to the types of cases LSCs can take on
2. See p 774 for list
iii. Other Civil Legal Services (775)
1. Law school clinics annually make up about 3 million hours of work
a. However, many clinics will avoid litigation against corporations to
avoid backlash towards its students.
iv. The IOLTA Controversy
1. States would use the small amount of interest earned on holding clients’
money (“interest on lawyers’ trust accounts”) to help fund legal aid
2. Supreme Court said no because this was a deprivation of property
D. Fee Shifting Statutes
i. General
1. Basically, some statutes allow plaintiffs to recover their fees from
defendants if the plaintiffs prevail …
ii. 42 USC 1983
1. Allows the court to shift fees in a number of civil matters
2. Fee shifting helps stimulate legal services for those who cannot afford it
iii. Fee Waiver as Term of Settlement
1. Evans v. Jeff D. (1986) (on pp 777-778)
a. In class action suits, a prevailing party is free to waive his right to
attorney’s fees, so long as the waiver is negotiated for and is approved
by the district judge as part of an overall settlement plan.
iv. Who is a “prevailing party” entitled to attorneys’ fees?
1. Buckhannon Bd. & Care Home v. W. Va. Dep’t of Health & Hum. Res.
a. The prevailing party is the one who receives relief from the courts and
not an out-of-court act or settlement
b. Ginsburg dissented and said “this will impede access to courts”
E. Pro-Bono Representation
i. Rule 6.1 Voluntary Pro Bono Public Service
Every lawyer has a professional responsibility to provide legal services to
those unable to pay. A lawyer should aspire to render at least (50) hours of
pro bono public legal services per year. In fulfilling this responsibility, the
lawyer should:
105
(a) provide a substantial majority of the (50) hours of legal services without
fee or expectation of fee to:
(1) persons of limited means or
(2) charitable, religious, civic, community, governmental and
educational organizations in matters that are designed primarily to
address the needs of persons of limited means; and
(b) provide any additional services through:
(1) delivery of legal services at no fee or substantially reduced fee to
individuals, groups or organizations seeking to secure or protect civil
rights, civil liberties or public rights, or charitable, religious, civic,
community, governmental and educational organizations in matters
in furtherance of their organizational purposes, where the payment of
standard legal fees would significantly deplete the organization's
economic resources or would be otherwise inappropriate;
(2) delivery of legal services at a substantially reduced fee to persons
of limited means; or
(3) participation in activities for improving the law, the legal system
or the legal profession.
In addition, a lawyer should voluntarily contribute financial support to
organizations that provide legal services to persons of limited means.
ii. Background to Pro Bono Representation
1. Amendments in 1993 and 2002 to Model Rule 6.1 have come as close as
the organized bar has to imposing a requirement on individual lawyers to
render pro bono service. Nonetheless, even the 1993 version of Model
Rule 6.1 remains aspirational and not mandatory. A lawyer is not subject
to discipline for failing to render pro bono service.
a. Setting a goal: Model Rule 6.1 sets a goal for each lawyer of 50 hours
of pro bono service per year.
b. Appropriate services: Model Rule 6.1 suggests some of the following
as appropriate for pro bono service: providing service at no fee or
reduced fee for those of limited means; service to religious, civic,
governmental, educational, or charitable organizations at no fee or
reduced fee; activities to improve the law, such as participation in bar
committees that draft model legislation; and providing financial
support for organizations that provide legal service for those of
limited means.
c. Setting priorities: Model Rule 6.1 encourages lawyers to spend the
bulk of their 50 hours per year rendering service at no fee, either
directly for those of limited means or to organizations that directly
serve those of limited means.
iii. Loan Forgiveness and Scholarship for Public Service
1. Georgetown has public service scholarship
2. Federal government loan forgiveness (IBR & PAYE)
F. Restrictions on Participation by Nonlawyers in Providing Legal Services
i. Unauthorized Practice of Law Statutes
1. Conn. Gen. Stat. 51-88 (2006)

106
a. A person who has not been admitted as an attorney under the
provisions of section 51-80 shall not: (1) Practice law or appear as an
attorney-at-law for another, in any court of record in this state, (2)
make it a business to practice law, or appear as an attorney-at-law for
another in any such court,…
ii. Forms of Unauthorized Practice
Unauthorized practice may occur when a licensed lawyer practices outside the
jurisdiction in which the license was granted or when those not licensed
engage in the practice of law.
a. Extraterritorial practice of licensed lawyers: Lawyers licensed in one
jurisdiction commit unauthorized practice violations when they
practice in another jurisdiction without obtaining permission from the
second jurisdiction’s courts. See § III.D. A lawyer who engages in the
extraterritorial, unauthorized practice of law cannot state a claim for a
fee.
b. Multijurisdictional practice: A wide range of simple realities of life and
law practice have created a powerful impetus for the states to allow
lawyers licensed in other states or countries to engage in regular but
temporary practice in states in which they are not licensed as the needs
of their clients dictate. More than admission to practice pro hac vice,
the movement to create multijurisdictional practice rules addresses
activities in and out of court. A report of the ABA’s Commission on
Multijurisdictional Practice, proposing a relaxation on temporary
admission to practice in a variety of circumstances, was adopted by the
ABA in August 2002.
iii. The Prohibition of Multidisciplinary Practice
1. What is Multidisciplinary Practice?
a. Refers to hiring non-lawyers to assist the lawyer with various task.
These are not simply office help, but when a non-lawyer becomes a
partner and shares in the profit.
b. Basically, you can’t have a lawyer/medical office
2. Relevant Rules
a. 5.4(a) bars lawyers from sharing profits with non-lawyers
b. 5.4(b) bans lawyers from forming partnerships with non-lawyers
c. 5.4(d) prohibits a lawyer from practicing for profit in association when
a non-lawyer owns an interest.
iv. The Prohibition of Nonlawyer Investment in Law Firms
1. Law firms cannot be sold on the public markets
2. Non-lawyers may not own law firms
3. Ethics 20/20 expressed its willingness to reevaluate the issue
a. Released an opinion that it would be ok if the ownership was less than
25%of the total value.
b. Revoked quickly due to massive controversy
v. Rule 5.4 Professional Independence of a Lawyer
(a) A lawyer or law firm shall not share legal fees with a nonlawyer,
except that:
107
(1) an agreement by a lawyer with the lawyer's firm, partner, or associate
may provide for the payment of money, over a reasonable period of time
after the lawyer's death, to the lawyer's estate or to one or more
specified persons;
(2) a lawyer who purchases the practice of a deceased, disabled, or
disappeared lawyer may, pursuant to the provisions of Rule 1.17, pay to
the estate or other representative of that lawyer the agreed-upon purchase
price;
(3) a lawyer or law firm may include nonlawyer employees in a
compensation or retirement plan, even though the plan is based in
whole or in part on a profit-sharing arrangement; and
(4) a lawyer may share court-awarded legal fees with a nonprofit
organization that employed, retained or recommended employment of
the lawyer in the matter.
(b) A lawyer shall not form a partnership with a nonlawyer if any of the
activities of the partnership consist of the practice of law.
(c) A lawyer shall not permit a person who recommends, employs, or
pays the lawyer to render legal services for another to direct or regulate
the lawyer's professional judgment in rendering such legal services.
(d) A lawyer shall not practice with or in the form of a professional
corporation or association authorized to practice law for a profit, if:
(1) a nonlawyer owns any interest therein, except that a fiduciary
representative of the estate of a lawyer may hold the stock or interest of
the lawyer for a reasonable time during administration;
(2) a nonlawyer is a corporate director or officer thereof or occupies
the position of similar responsibility in any form of association other than
a corporation ; or
(3) a nonlawyer has the right to direct or control the professional
judgment of a lawyer.

A. Advertising Legal Services


i. Bates v. State Bar of Arizona (822)
1. AZ Statue: A lawyer shall not publicize himself or his partners or associate
or any other lawyers affiliated with him or his firm . . . in newspaper or
magazine, etc . . .
2. SOCTUS said the rule violated the First and Fourteenth Amendments.
Justice Blackmun argued that commercial speech does merit First
Amendment protection given the important functions it serves in society,
such as providing consumers with information about services and
products, and helping to allocate resources in the American system of
free-enterprise. The Court held that allowing attorneys to advertise would
not harm the legal profession or the administration of justice, and, in fact,

108
would supply consumers with valuable information about the availability
and cost of legal services.
ii. Rule 7.2 Advertising
(a) Subject to the requirements of Rules 7.1 and 7.3, a lawyer may advertise
services through written, recorded or electronic communication, including
public media.
(b) A lawyer shall not give anything of value to a person for recommending
the lawyer's services except that a lawyer may
(1) pay the reasonable costs of advertisements or communications permitted
by this Rule;
(2) pay the usual charges of a legal service plan or a not-for-profit or qualified
lawyer referral service. A qualified lawyer referral service is a lawyer referral
service that has been approved by an appropriate regulatory authority;
(3) pay for a law practice in accordance with Rule 1.17; and
(4) refer clients to another lawyer or a nonlawyer professional pursuant to an
agreement not otherwise prohibited under these Rules that provides for the
other person to refer clients or customers to the lawyer, if
(i) the reciprocal referral agreement is not exclusive, and
(ii) the client is informed of the existence and nature of the
agreement.
(c) Any communication made pursuant to this rule shall include the name and
office address of at least one lawyer or law firm responsible for its content.
iii. Rule 7.1 Communications Concerning A Lawyer's Services
A lawyer shall not make a false or misleading communication about the lawyer
or the lawyer's services. A communication is false or misleading if it contains
a material misrepresentation of fact or law, or omits a fact necessary to make
the statement considered as a whole not materially misleading.
iv. Rule 7.4 Communication of Fields of Practice and Specialization
(a) A lawyer may communicate the fact that the lawyer does or does not practice in
particular fields of law.
(b) A lawyer admitted to engage in patent practice before the United States Patent
and Trademark Office may use the designation "Patent Attorney" or a substantially
similar designation.
(c) A lawyer engaged in Admiralty practice may use the designation "Admiralty,"
"Proctor in Admiralty" or a substantially similar designation.
(d) A lawyer shall not state or imply that a lawyer is certified as a specialist in a
particular field of law, unless:
(1) the lawyer has been certified as a specialist by an organization that has
been approved by an appropriate state authority or that has been accredited
by the American Bar Association; and
(2) the name of the certifying organization is clearly identified in the
communication.

v. Rule 7.5 Firm Names And Letterheads


(a) A lawyer shall not use a firm name, letterhead or other professional designation
that violates Rule 7.1. A trade name may be used by a lawyer in private practice if it
does not imply a connection with a government agency or with a public or
charitable legal services organization and is not otherwise in violation of Rule 7.1.
109
(b) A law firm with offices in more than one jurisdiction may use the same name
or other professional designation in each jurisdiction, but identification of the
lawyers in an office of the firm shall indicate the jurisdictional limitations on those
not licensed to practice in the jurisdiction where the office is located.
(c) The name of a lawyer holding a public office shall not be used in the name of a
law firm, or in communications on its behalf, during any substantial period in
which the lawyer is not actively and regularly practicing with the firm.
(d) Lawyers may state or imply that they practice in a partnership or other
organization only when that is the fact.
vi. Rule 5.4 Professional Independence Of A Lawyer
1. See above
B. Solicitation of Clients
i.General
1. The court has showed greater willingness to allow solicitation from
lawyers.
ii. Ohralik v. Ohio
1. In February 1974, 18-years-olds Carol McClintock and Wanda Lou
Holbert were seriously injured when an uninsured motorist hit the vehicle
McClintock was driving in their hometown of Montville, Ohio. When
Albert Ohralik, a local attorney, learned of the accident, he visited
McClintock in the hospital and offered to represent her in exchange for a
portion of the proceeds collected from her insurer. Ohralik also
approached Holbert at her home and obtained her oral assent to
representation, which he secretly tape-recorded.
2. The Court held that a state may constitutionally discipline a lawyer for
soliciting clients in person and for financial gain under circumstances
likely to pose dangers that the state has a right to prevent. The Court
further held that the state need not show actual harm or injury to the
solicited clients to sustain a disciplinary action.
iii. Shapero v. Kentucky
1. Petitioner wished to have the right to mail solicitations to “potential
clients” drawn from a list of people who had recently had a foreclosure
suit filed against them. The Kentucky Bar Association denied this request,
and he now appeals.
2. A blanket prohibition on lawyer advertising is a direct violation of the
First Amendment protections afforded to commercial speech.
iv. Rule 7.3 Solicitation of Clients
(a) A lawyer shall not by in‑person, live telephone or real-time electronic
contact solicit professional employment when a significant motive for the
lawyer's doing so is the lawyer's pecuniary gain, unless the person contacted:
(1) is a lawyer; or
(2) has a family, close personal, or prior professional relationship with the
lawyer.

110
(b) A lawyer shall not solicit professional employment by written, recorded or
electronic communication or by in‑person, telephone or real-time electronic contact
even when not otherwise prohibited by paragraph (a), if:
(1) the target of the solicitation has made known to the lawyer a desire not
to be solicited by the lawyer; or
(2) the solicitation involves coercion, duress or harassment.
(c) Every written, recorded or electronic communication from a lawyer soliciting
professional employment from anyone known to be in need of legal services in a
particular matter shall include the words "Advertising Material" on the outside
envelope, if any, and at the beginning and ending of any recorded or electronic
communication, unless the recipient of the communication is a person specified in
paragraphs (a)(1) or (a)(2).
(d) Notwithstanding the prohibitions in paragraph (a), a lawyer may participate with
a prepaid or group legal service plan operated by an organization not owned or
directed by the lawyer that uses in‑person or telephone contact to solicit
memberships or subscriptions for the plan from persons who are not known to need
legal services in a particular matter covered by the plan.
C. Legal Culture in Certain Practice Settings
i. What’s covered in the book
1. Large firms (856)
a. Law Firms in the Movies (857)
b. Ethical Member of an Unhappy, Unhealthy, and Unethical Profession
(860)
2. Small firms (863)
a. Salaries and attrition (863)
b. Setting one’s own schedule (863)
c. Bringing in business (866)
d. Promotion in small firms (866)
e. Other features of small-firm life (866)
f. Urban versus rural practice (867)
g. Gender patterns in small firms (868)
h. The future of small firms (868)
i. Small firms and the Internet (869)
3. Government and nonprofit organizations (869)
4. Work settings for lawyers: Culture and satisfaction (871)
D. The business of law practice in the twenty-first century (875)
i. Minor Things Covered in the Book
The 2008 recession: Impact on the legal profession (876)
Structural changes in private law practice (879)
Temporary and contract lawyers (880)
Lawyers in retail stores (882)
The Internet as a substitute for legal services (883)
Offshoring and onshoring (884)
Model Rules Revisions Open the Door for Lawyers (890)

111
Globalization of law practice (895)
New methods of financing law firms and legal work (895)
ii. Multistate practice: A challenge to state-based licensing (General)
1. Most states have adopted 5.5’s amendment
iii. Rule 5.5 Unauthorized Practice Of Law; Multijurisdictional Practice Of
Law
(a) A lawyer shall not practice law in a jurisdiction in violation of the regulation of
the legal profession in that jurisdiction, or assist another in doing so.
(b) A lawyer who is not admitted to practice in this jurisdiction shall not:
(1) except as authorized by these Rules or other law, establish an office or
other systematic and continuous presence in this jurisdiction for the
practice of law; or
(2) hold out to the public or otherwise represent that the lawyer is admitted
to practice law in this jurisdiction.
(c) A lawyer admitted in another United States jurisdiction, and not disbarred or
suspended from practice in any jurisdiction, may provide legal services on a
temporary basis in this jurisdiction that:
(1) are undertaken in association with a lawyer who is admitted to practice
in this jurisdiction and who actively participates in the matter;
(2) are in or reasonably related to a pending or potential proceeding before
a tribunal in this or another jurisdiction, if the lawyer, or a person the
lawyer is assisting, is authorized by law or order to appear in such
proceeding or reasonably expects to be so authorized;
(3) are in or reasonably related to a pending or potential arbitration,
mediation, or other alternative resolution proceeding in this or another
jurisdiction, if the services arise out of or are reasonably related to the
lawyer's practice in a jurisdiction in which the lawyer is admitted to practice
and are not services for which the forum requires pro hac vice admission;
or
(4) are not within paragraphs (c) (2) or (c)(3) and arise out of or are
reasonably related to the lawyer's practice in a jurisdiction in which the
lawyer is admitted to practice.
(d) A lawyer admitted in another United States jurisdiction or in a foreign
jurisdiction, and not disbarred or suspended from practice in any jurisdiction or the
equivalent thereof, or a person otherwise lawfully practicing as an in-house counsel
under the laws of a foreign jurisdiction, may provide legal services through an
office or other systematic and continuous presence in this jurisdiction that:
(1) are provided to the lawyer's employer or its organizational affiliates, are
not services for which the forum requires pro hac vice admission; and
when performed by a foreign lawyer and requires advice on the law of this
or another U.S. jurisdiction or of the United States, such advice shall be
based upon the advice of a lawyer who is duly licensed and authorized by
the jurisdiction to provide such advice; or
(2) are services that the lawyer is authorized by federal or other law or rule
to provide in this jurisdiction.
(e) For purposes of paragraph (d):
(1) the foreign lawyer must be a member in good standing of a recognized
legal profession in a foreign jurisdiction, the members of which are

112
admitted to practice as lawyers or counselors at law or the equivalent, and
subject to effective regulation and discipline by a duly constituted
professional body or a public authority; or,
(2) the person otherwise lawfully practicing as an in-house counsel under
the laws of a foreign jurisdiction must be authorized to practice under this
rule by, in the exercise of its discretion, [the highest court of this
jurisdiction].

19. IN CLASS HYPOS


The Midnight Phone Call
You’re a tax, wills, gifts, and estates lawyer, with one partner who does eminent domain
work. A client calls at midnight asking you to defend her 19-year old son Jeff, charged with
vehicle theft, saying “Would you please represent Jeff? I will pay whatever it takes to get the
charges dismissed.”
What do you want to know before you answer her question?
o What does Jeff want?
o Who owns the car? (conflict check)
o How much can you charge Joanna?
o Does she understand you can’t guarantee results in a criminal case?
o Is there a conflict because the eminent domain lawyer might represent the county?
o Do I have any background in criminal defense – can I even handle this without first
training or apprenticing? Am I competent to handle this case? How much time
would it take to get competent, and is that time (/money) worth it to Jeff?
o It sounds like she’s expecting you to “keep her informed of any developments” –
but those may be confidential!
Note: confidential information and privilege are different!
o Where did the alleged crime occur? Can I even practice there?
o Does Jeff have a right to have you represent them?
He has a right to A lawyer, but not to the one of his choice.
One of the liberties a lawyer has is the ability to choose your clients
o Do you have to go to the jail at midnight? If you are going to represent him, you
might want to.
1-1 The New Country
Not super relevant
1-2 Weed
Iowa bar application asks, “Are you currently, or have you been in the last three years,
engaged in the illegal use of drugs? If Yes, give complete details below” or attached. *57
You have smoked weed during law school—most recently three weeks ago. Use and
possession of weed is illegal in Iowa and federal law.
You’ve heard a bar admissions committee member say applicants who answer affirmatively
would be denied admission to the bar.
Considerations
 Relevant Rule: A bar applicant shall not knowingly make a false statement
 “Currently” means recently enough so that the condition or impairment may have an
ongoing impact. (which pot doesn’t, but it’s within the last three years)
 You have the right to assert the 5th Am. privilege against self-incrimination if you have
reasonable cause to believe answering may expose you to the possibility of criminal prosecution.

113
 Contents of the application are public, subject to limitations – including that examiners “shall
not disclose information relating to the criminal history or prior misconduct of the applicant.”
 Anyone can report on you (i.e. friends who saw you at parties)
Nora’s Answer
 YES, in the past 3 years I have engaged in the illegal use of drugs, but w/o any ongoing impact
o explain circumstances
o note: its lack of impact on your performance as student and professional; note its legal
in many states; note you have now completely stopped
My thoughts
 Depends on whether the law student was a student in Iowa/state that criminalizes marijuana v.
state where recreational marijuana is legal.
Phil
 Do research.
o Has a question like this been legally challenged?; What happens to people who
answer yes?; Talk to a drug lawyer
Consequences
 Of telling the truth You might be asked to rat out other people
 Of lying  It could come out in future security clearances, office runs
 Of pleading the Fifth
o They’re going to put the screws on you to waive it so you can be honest with them
o They’ll likely ask why you pled the fifth
o Could potentially deny you admission to the bar for taking the fifth
Alternatives
 Go to another state and wait there three years so you can answer honestly
 Sue the bar (cool story bro)
Reality
 Phil called Iowa’s Board of Bar Examiners Chair, and asked if applicants who admitted to
smoking weed were denied.
 Mr. Brown said he couldn’t remember anyone admitting to it. Said he’d look into it, but never
called back, despite 4 messages.
 Clerk of Iowa SC said sometimes people say yes and when they do, they’re brought in for oral
examination and their application might be delayed or denied (but he couldn’t recall any denial).
 People who invoke the 5th get called in for interrogation and always end up waiving it
 Clerk said they just like to collect information

1-3 Doctored Resume


Not super relevant
2-1 Dying Mother
Hypo where woman doesn’t follow-up on will and then family calls when she’s on her
deathbed and cannot sig/is already dead (can’t remember)
 Are good intentions a defense to an unethical charge?
 You would be forcing assistants to partake in illegal activity as well
Real Case – Attorney did as the children asked and one child changed their mind and used
the forged signature against the attorney
Defense: sympathy for client, and his client was Gloria, not the children
COA: he entered into a client-attorney relationship by counseling them about the will, and
ultimately disbarred in MD. The BA bar additionally followed up and suspended him for
about 3 years.

114
2-2 “I’m Not Driving”
You’re a first year law student. Your buddy Ryan obviously has a drinking problem. You’ve
tried talking to him a few times, but have gotten nowhere. What should you do? *87-88
Keep in touch and: try again; give him leaflets; talk to an advisor about what to do, what
resources are out there (Dean Bailin can get people help on a confidential basis); Talk to
friends about an intervention; Be there for him and try to help keep him out of serious
trouble; Maybe get counselling for yourself or go to al-alon if it’s really stressing you out too
(codependency)
What if you do nothing, and he passes out in class smelling in alcohol; this keeps happening. What should
you do? What if Ryan almost gets hit by a car?

2-3 Exculpatory Evidence


You learn from a lawyer friend that he suppressed exculpatory evidence for someone who
was then convicted and imprisoned for murder, and is now on death row. You ask him to
tell you the defendant’s name, but he doesn’t, and he dies two months later. You do nothing.
Five years later, you read about a man who must be the defendant in question. The case will
stay closed unless a court ordered it reopened. What should you do? *95
8.3 requires reporting of serious ethical violations you are aware of
 “Know” is satisfied because it’s the word of a good, long-time friend who would have no
incentive to lie to you.
 This raises a “substantial question” as to your friend’s honesty and fitness as a lawyer.
Other reasons to report
 If not, the guy will die
Good idea to report but you risk:
 Being disciplined under 8.3
 Your friend being wrong and the blood test actually showing the defendant did it – and then
you might look like a jerk for ratting out a dead friend, and might either way get disciplined
Class Options
 Do nothing
 Tell defendant’s lawyer (38%)
 Tell the prosecutor
o Under Brady, the prosecutor’s office has an obligation to share exculpatory evidence
with the defense. They are actually the most able to rectify this situation. But the
prosecutor might report you to scapegoat you for the miscarriage of justice.
 Tell the bar association; tell the press
 Other (38%)
o Tell people anonymously (but LaVoie will start issuing subpoenas)
Real Life case: In re Riehlmann 2005 WL 106470 (La. 2005)
“Five years later, Thompson's attorney found the suppressed lab report that proved
Thompson didn't do it. When Riehlmann found out, he wrote an affidavit stating that
Deegan told him he had intentionally suppressed the evidence. Riehlmann later testified to
knowing he should’ve reported Deegan’s misconduct, but didn’t.
 Rule 8.3(a) - you have to report known instances of another attorney's misconduct.
 Rule 8.4(c) - an attorney cannot engage in conduct involving fraud, deceit or
misrepresentation.
 Rule 8.4(d) - an attorney cannot engage in conduct prejudicial to administration of justice.
Office of Disciplinary Counsel charged Riehlmann w/violating Rule 8.3(a) and 8.4(c)
Riehlmann argued that 8.3(a) merely requires that an attorney possessing unprivileged
knowledge of a Rules violation shall report such knowledge to an authority empowered to
115
investigate the facts, and he did eventually report his information to Thompson's attorney,
who reported it to the Court. Also argued 8.3(a) doesn’t give a specific time limit for
reporting
The Hearing Committee’s Findings and Recommendations
 Found that since Deegan never told Riehlmann the name of the defendant, Riehlmann
never had "knowledge" of the violation and therefore didn't violate 8.3(a).
 Found Riehlmann had a broader obligation to ensure that justice is fairly administered, and
by his complete inaction Riehlmann violated 8.4(d).
 Recommended that Riehlmann be found to have violated 8.4(d), but not 8.3(a).
The Disciplinary Board’s Recommendations
 Riehlmann be found to have violated Rule 8.3(a) and Rule 8.4(d).
 While the Hearing Board found that you need clear and convincing evidence of
knowledge to have a 8.3(a) violation, the Disciplinary Board thought that the standard
should be when a reasonable lawyer would have "a firm opinion that the conduct in
question more likely than not occurred."
o In other words, the disciplinary board thought you don't need all the details before
you are required to report, you just need to have a sufficient knowledge that
something happened.
o The disciplinary board found that while there is no specific time limit, reporting
must occur within a reasonable time under the circumstances.
The Appellate Court Decision
 Affirmed the Disciplinary Board’s recommendations
 Found 8.3(a) has three requirements:
o The attorney must possess unprivileged knowledge of a Rules violation.
 Since a tribunal will do the investigation later, the reporting attorney
doesn't need "proof" just a reasonable amount of evidence.
o The attorney must report that knowledge.
 The report must be timely.
o Must report to a tribunal or authority empowered to investigate the violation.
 In LA, it’s the Office of Disciplinary Counsel, not, opposing side's attorney.
 Riehlmann was disciplined w/a Public Reprimand
Other Outcomes
Thompson (“LaVoie”) was released after 18 years in prison. He sued the former DA of
Louisiana for not training prosecutors to not supress exculpatory evidence, and a court
awarded him $14mill. The case went all the way to the US SC, which held, 5-4, that
Thompson was not entitled to any redress.

2-4 The Little Hearing


You’re hired by a solo immigration practitioner. Despite your complete inexperience on
immigration, your boss says you have to show up to an asylum hearing, saying “it’s just a
little hearing” [not for the client! it’s possibly life or death] and that you can learn everything
you need to know about an imm law textbook he hands you. *103
The interview goes horribly; you need an interpreter but have none and can’t get one. Your
boss says he has to do more tomorrow. You want to quit. Your boss (who has 1500 active
cases, with more interviews and hearings scheduled each workday than he can attend) says
since he charges clients modest fees, he can only give them modest services – i.e. it’s better
than nothing.
Rule 1.1 requires competent representation – but you think you’re not.
Should you show up to work tomorrow?
Is there anything that Pat should or must do?
116
Relevant Rules
5.2(b): “A subordinate lawyer does not violate the Rules if that lawyer acts in accordance
with a supervisory lawyer’s reasonable resolution of an arguable question of
professional duty”
Phil Comments
One of the major ethical pickles in immigration law is the need exceeds the number of
lawyers, so there are some lawyers (like Helman) who just take as many clients as possible, at
low rates, and represent them badly. (Better to represent 6 very well?)
In Class Options/Voting
Do as Helman says
Quit now (Better that than being fired)
Something else (specify). 87%
Quit (but with notice, so that clients who are counting on you don’t get left high
and dry
Do research on the norms
Report
Further Hypo
Pat goes back to work. Her spouse says she’s being immature; should stick it out for a year
to gain experience and a good recommendation; we need the income. Life is compromise;
nobody is pure. They fight. Pat storms out and comes to you for advice. Do the best you
can; try to improve it on the inside?

3-1 Your Dinner w/Anna (145)


Dinner with old friend, Anna, after first day at work. You consider saying you worked on
civil suit against the local police department on behalf of a client whose wrist was broken by
a police officer. Is this okay (disclosing your working on a police brutality case)?
It seems to be okay, but probably not if you were representing the police
department. This is a story that’s likely in the news so you need to be careful with
details.
Alternatively, word could get out and 1.6(a) says: “A lawyer shall not reveal
information relating to the representation of a client unless the client gives informed
consent, the disclosure is impliedly authorized in order to carry out the
representation, or the disclosure is permitted by paragraph (b).”
Anna tells you about her work counselling parents of kids with serious or terminal illness.
You tell her: That you were assigned to work on a police brutality case; your client’s first
name, Joey (!?); the name of the bar where the place happened; that he was very drunk;
relationships between Joey and those involved; that your boss (gives his first name) thinks
her client might have been the instigator; the police officer’s name; the investigator’s name
(!?); y’all’s research strategy
Is this okay?
No, it violates 1.6. Comment 4 suggests this level of detail “could reasonably lead to
the discovery of [information relating to the representation of a client] by a third
person.” It is not enough to just use the client’s first name and/or swear someone to
secrecy. You CAN however talk to other people in your law firm about the case.
You also are in public so your subjective assessment that no one is listening isn’t
enough to permit a conversation in a restaurant. Once you are a lawyer, you have to
be careful of the space you are in.
NOTE: also be careful about unencrypted emails. Hacking’s prevalence
suggests unencrypted email may not be enough (see ABA)
117
Is it okay to complain about clients?
While it is probably okay to blow off steam, it’s not okay if the client is identifiable.
Nonetheless, it’s also somewhat disrespectful and unprofessional

3-2 The Missing Persons, Scene 1*158


Lawyers generally should protect as confidential most info about past criminal activities by
clients. Garrow has a history of brutal violent crime against young people. You’re assisting
his court-appointed lawyer. You learn that he had a troubled childhood, including sex with
animals and drinking animal blood as a child. You thus think he should plead not guilty by
reason of insanity, in exchange for providing info about the whereabouts of the other
missing teens. Garrow agrees. Your co-counsel hypnotizes Garrow, who tells you where the
bodies are, but Garrow did not grant permission to share the information with the
prosecutor. You aren’t entirely sure if Garrow is telling the truth (he’s a wily guy).
What do you do?
14% of people said tell the police, prosecutor, or press about the bodies’ location while
revealing your identification
Pros: helps loved ones find resolution and closure; could save lives; conscience;
brings serial murderer to justice for two other murders (not your job though).
Cons: cops would then likely find your client’s DNA, hair, footprints, etc., so this
would disadvantage your client
7% of people said the same except to hide your identification
Considerations:
Does hiding your ID help at all?  Maybe not because they’ll likely connect it to
your client and your client will find and be pissed
Also possible they won’t take tip seriously
Could Garrow have “impliedly authorized” you to reveal the info?  Probably not
unless he agrees it helps his insanity defense.
Does Garrow’s insanity give you the right to disclose?  Maybe it makes the
hypnosis more worrying.
46% of people said to get more information (i.e. site investigation and see if bodies are there;
offer info in exchange for a plea deal)
Pros: Then you know the truth. Might find the girls still alive! Could then make
more informed decision about what to do next. Could offer to trade the info for
accepting the insanity plea (but would need client’s permission to make that offer).
Cons: Risk that could be tampering with evidence/disturb the crime scene. Could
enrage prosecutor for withholding this information. Could lead someone else to the
bodies (which you might not want to do). If you find and free the girls, you’re in a
different boat – must incriminate your client immediately.
Hiring a Private Investigator: Is it better or worse than going yourself? They
would be bound by Rule 1.6 – but might still break confidentiality. They might be
subpoenaed too. You lose control of the information. There could be a co-
conspirator there potentially, so it could be dangerous!

32% of people said do nothing and defend your client


Pros: You maintain important interests (public policy interest in having clients
disclose their crimes to their attorneys)
Cons: Interests of the girls; if they are alive, ‘

Additional Considerations
118
If they are alive  1.6(b)(1) says you can tell if it might “prevent reasonably certain
death or substantial bodily harm” – if either are still alive, you could do the latter (but
not reasonably certain)
If they are dead  A state law requires the reporting of the discovery of bodies –
and Rule 1.5b6 requires disclosure “to comply with other law or a court order.” So
this applies.
If a judge won’t let you withdraw from a live case, you just can’t!

3-3 The Missing Persons Scene 2 (*163)


You conduct an investigation and find and photograph the remains of the girls without
disturbing them. One of the girls’ fathers comes to see you and asks if you know anything.
What do you do?
8% said tell the father (or the authorities) the girl is dead
Pros: No guilt when going to sleep at night (personal value conflicting w/the rule);
consideration to society and victims’ family outweighs obligation to the defendants
and the rules
Cons: Breach of confidentiality sets dangerous precedent from public policy
perspective
12 % said refuse to tell and go to trial, where you plead insanity, which will be unlikely to
work so Garrow will likely go to prison for life for a separate murder and the bodies may
never be found
Pros:
Cons:
77% said don’t tell father, but offer the prosecutors info on the girls in exchange for a plea
bargained acceptance of Garrow’s insanity defense (which is what Garrow wants).
What Happened In Real Life?
Garrow’s lawyers (Belge and Armani) chose option 3, but the prosecutor got pissed. The
prosecutor threatened criminal charges against the lawyers for withholding evidence and
obstructing justice. The prosecutor had a lip reader “eavesdrop” on conversations between
Garrow and his attorneys.
The next month, Armani’s law office was broken into and searched three times. But, the
evidence was hidden in Armani’s father’s basement. In trial, the defense pled insanity, which
the jury rejected. Garrow was convicted of the one murder and sentenced to prison, though
he confessed to the multiple murders, thus revealing the lawyers had concealed their
knowledge of the bodies for months. The public was furious and Armani and Belge made a
public statement, explaining their silence but they got death threats and Molotov cocktails
thrown at their houses.
Belege was charged w/violating NY public health statute requiring people to report deaths
to the authorities.
Issue: Was Belge excused from reporting the location of the bodies, either by the
ethical duty of confidentiality, or the attorney-client privilege?
Holding: Yes, dismissed the indictment on grounds of privileged communication
and in the interest of justice.
Analysis: The public charges obstruction of justice or becoming an accomplice after
the fact. But the Constitution protects the individual accused of a crime from “the
inroads of the state” *167 including the 5th’s privilege against self-incrimination. The
attorneys were bound to uphold it and maintain a “sacred trust of confidentiality”
*167. Balancing Belge’s duty to Garrow v. his duty under the health law: the later is
the “trivia of a pseudo-criminal statute”, which is outweighed by the Constitution!
119
Appeal: The Prosecutor appealed in People v. Belge, which upheld the dismissal
because: The attorney-client privilege is not all-encompassing: an attorney must
“protect his client’s interests, but also must observe basic human standards of
decency” and not forget that the legal system needs to “accord justice to the interests
of society and its individual members”; We have serious concerns about the
consequences of absolute attorney-client privilege, but they’re outside the legal scope
of this appeal (i.e. lawyers operate under a special code that we’re not fully
comfortable with).
One girl’s parents asked for a disciplinary investigation and the committee found: the lawyers
did the right thing by not disclosing their knowledge; they would have been okay even if they
destroyed the photos they took; it wasn’t improper to use their knowledge in a plea bargain.

3-4 The Missing Persons Scene 3 (*169)


It turns out Garrow had been stalking Armani’s daughter 15-year-old, Dorina. Garrow got
himself transferred from a max-security prison to medium-security prison and pretended he
was paralyzed from gunshot wounds he got during his capture. Garrow escaped prison one
year after his conviction. Prison officials found a hit list in his cell, that included Armani’s
name, and yours (you are his lawyer now). The police ask you if you know where Garrow
might be hiding. You recall he mentioned to you, in confidence, about a place he used to
hide.
Should you tell the police?
Considerations: Duration of the duty to protect confidences: assuming he’s no longer your
client, can you now tell?
Rule 1.6 Comment 20: “The duty of confidentiality continues after the client-lawyer
relationship has terminated. See Rule 1.9(c)(2). See Rule 1.9(c)(1) for the prohibition
against using such information to the disadvantage of the former client.” *29
But 1.9(c)(2) says, “except as these Rules would permit or require with respect to a
client”, so actually it’s okay; see next bullet.
You can tell under 1.6(b)(1) (“to prevent reasonably certain death or substantial
bodily harm”)

3-5 Rat Poison


You’re representing Norton, who has been charged with involuntary manslaughter of a 4-
year-old child after he improperly used industrial rat poison to exterminate a home. He has
also used the poison in other homes. He has records of the locations but has dragged his
feet on sharing them with you, and you worry he may destroy them. The police haven’t
searched his house or made any other efforts to get Harry’s records, and he won’t let you
talk to the prosecutor.
What will you do?/Options
+ Tell the police to search his house (cuz he won’t give records & might destroy them).
Could they get a warrant to do so?
+ Report client to the prosecutor
+ Bargain w/the prosecutor
+ Don’t tell the prosecutor outright, but “accidentally” raise the idea in their minds that
Harry has a lot of clients and see if they put two and two together.
+ Implore your client to reveal the locations
+Put up ad and/or other publicity saying this kind of poison is dangerous
+Decline to represent him
+Try to find out where/how his records are kept
120
+Do nothing
Relevant Rules
1.6(b)(1) says you can only reveal info that will “prevent reasonably certain death or
substantial bodily harm” – but is it questionable that here it’s reasonably certain. One kid
already died but it’s unclear if other kids will be affected
Subsequent Events
Next, Harry tells you he keeps his records in a cigar-box in his house. Does that help? Do
you go to his house and ply him with pizza and try to get it?
Next, Harry lets you - copy his records, but instructs you to not tell the prosecutor that he
used the poison in other homes. Now what?
+ Do nothing; just defend him
+ Notify his customers that there’s a dangerous poison in their homes. YES. Though
then it opens him up to lawsuits, losing customers. Maybe tell them that it was an
accident.
+ Notify them and then go with Harry to their homes to make sure he actually
removes the poison. Or YES (but would he?)
+ Give his records to the police or prosecutor. No, cuz doesn’t guarantee they’ll
release the information as quickly as you can directly.
+ Other
In Real Life
The prosecutor eventually realized that there might be other homes that had the industrial
poison in them. The prosecutor got a search warrant for Harry’s house, seized the records.
Harry, fearful of prosecution, cooperated with the authorities, a government agency visited
the homes & removed the poison. Harry’s lawyer worked out a plea bargain for the single
count of negligent homicide, to which he pleaded guilty.

3-6 Reese’s Leases


You’re in private practice. Your biggest client, Executive Leasing, produces 60% of your
revenue. Without Executive, your firm would fail. You drafted the blank form that the
company then used for each lease, and you supplied legal opinions (for the benefit of the
bank) affirming that Executive Leasing Services owned the cars that it was leasing. You
created the lease form, & you have certified the company’s ownership of the cars. You
discover that Exec has been getting extra loans from a bank by altering existing leases &
drawing phony leases to make it appear that the volume of their business is larger than it
really is. You confront Exec, who said they would stop fiddling around with the paperwork.
Are the actions below forbidden, permitted or required by Rules 1.2(d), 1.6(b), 4.1?
1.2(d) “A lawyer shall not counsel a client to engage, or assist a client, in conduct
that the lawyer knows is criminal or fraudulent, but a lawyer may discuss the legal
consequences of any proposed course of conduct with a client and may counsel or
assist a client to make a good faith effort to determine the validity, scope, meaning or
application of the law.”
Comment 10: sometimes you’re supposed to withdraw if you
discover the person has been committing fraud
1.6(b) (above)
4.1 “In the course of representing a client a lawyer shall not knowingly: (a) make a
false statement of material fact or law to a third person; or (b) fail to disclose a
material fact to a third person when disclosure is necessary to avoid assisting a
criminal or fraudulent act by a client, unless disclosure is prohibited by Rule
1.6.”
121
Actions
+ Reveal the fraud to the bank or the DA if Reese refuses to do so.
PERMITTED under 1.6, because you MAY reveal “to prevent, mitigate or rectify
substantial injury to the financial interests or property of another that is reasonably
certain to result or has resulted from the client's commission of a crime or fraud in
furtherance of which the client has used the lawyer's services”
+ Counsel Exec of consequences of fraud, secure their promise that it won’t commit fraud
in the future, and keep representing it.
PERMITTED
+ Terminate representation w/o revelation
PERMITTED
What is best choice?
Withdrawing because although you’ll lose business, that’s better than being disciplined or
going to jail.
Scene 2 (handout)
You initially stick with client, but then discover they were still engaging in fraud. You decide
to withdraw from their representation. Exec tells you not to reveal the past frauds to anyone,
including the new lawyer, who’s a friend of yours.
Will you reveal the fraud or keep quiet?
Considerations: Now your work has been “in furtherance” of the fraud, so you’re definitely
PERMITTED to reveal.
Real Life
It was leasing computers. Lawyers turned a blind eye, and then consulted two Fordham law
professors for expert advice. The profs said they were okay. They eventually withdrew
representation. A new lawyer, a friend, came on, but was left in the dark about the fraud.
Eventually, the leasing company faced criminal charges, and the lawyers had to pay millions
for supporting the fraud. The professors later disclaimed responsibility for what had
happened, and later got awards for their ethical standards.

3-7 An Investment Project


You learn from a client that developers are planning to build a shopping center that will
increase the property of land at Lakeshore. You want to buy a parcel of land the client isn’t
interested in there to profit from the info your client has given you.
Is that okay?
Maybe not because 8.4 says, “you can’t (c) engage in conduct involving dishonesty.”
However, that doesn’t apply.
The relevant rule is 1.8(b)you cannot do things against your client’s interest based on
confidential stuff you learn from them. (prohibits using confidential information to the
disadvantage of the client). But here it wouldn’t harm the client, so you can do it just fine.

4-1 Murder for Hire


Client, Eddie Tilmann, is charged with murdering his father. His friend Sikorski is also
charged. You interviewed Eddie in jail, but other prisoners might have been able to overhear
your convo over the partitioned cubicles.
Eddie told you he planned to admit guilt, but changed his mind the next day. Cabrales, a
prisoner in the next cubicle, overheard the convo and is going to tell Sikorski. If Cabrales’
testimony is likely to be excluded, you might advise Eddie to go to trial. If not, you should
advise Eddie to plead guilty in exchange for a possible lighter sentence.
What will you do?
122
Advise Eddie that Cabrales’ testimony will likely be excluded, so he should go to trial as he
wishes.
Because Eddie & you talked in the most private place possible; it’s the jail’s fault, not
Eddie or the attorney, that someone overheard.
Advise Eddie that Cabrales will be allowed to testify about Eddie’s plan to admit guilt, so
he’d better go for a plea bargain instead of going to trial.
Because Cabrales isn’t bound by attorney-client privilege.
And public-interest-exception to the privilege: Sikorski has the right not to get locked away
for Eddie’s crime. You could cite *200 Villa case
But if arguing the opposite, you could distinguish this case from Vela. Here there’s a
dispute between Sikorski and Tillman over whether or not privilege applies. In Vela,
the two defendants on trial who wanted an exception to the privilege had a
constitutional right that conflicted the application of the privilage. Here, there are
conflicting rights, whereas in Vela, all the parties wanted the same thing.
In Real Life
The court upheld the privilege and suppressed the overheard testimony, noting the lack of
privacy in holding cells and that there was nowhere else for the lawyer and client to talk.
On appeal, Sikorski” argued for balancing his constitutional right to a fair trial
against the privilege, citing Vella.
But the appellate court affirmed upholding the privilege, noting the conversation was
not literally a confession, and distinguishing Vella on the ground that there, both
defendants had constitutional rights.

4-2 A Secret Confession


Your client Chester, has confessed to you that he alone murdered the Sosas. *225 But
Parnett, a fellow member of a pot distro ring, is about to be sentenced to life in prison for
the murder as a result of a plea bargain in a separate prosecution. Chester doesn’t want you
to reveal his confession, even though it might clear Parnett.
May you reveal Chester’s confession?
Yes
For public interest exception, i.e. to prevent a violation of constitutional rights in a
[different] criminal case? Nope?
No
Privilege may not apply, but you have an ethical duty under Rule 1.6a and 1.6b1 to maintain
confidentiality.

What if the judge rejects the plea bargain and Parnett is about to be sentenced to death
instead of life in prison? May you now reveal the confession?
Yes
1.6(b)(1) is triggered because it is imminent threat of death
No
Even if 1.6(b)(1) is triggered, it is a “may reveal” rule, whereas the privilege is
absolute
*200 Henderson is an example of where the potential death of a child overcame the
privilege
Alternatively, it’s possible the threat of death is not imminent and possibly unlikely

21 years pass. Chester dies in prison, leaving no family. Parnett has been in jail the whole
time.
123
Can you now reveal Chester’s confession and testify about it in a hearing to review Parnett’s
conviction?
Possibly relevant:
Restatement § 60: the duty of confidentiality
“during & after the representation of a client… the lawyer may not use or disclose
confidential client info … if there is a reasonable prospect that doing so will
adversely affect a material interest of the client or if the client has instructed
the lawyer not to use or disclose such info.” Comment c, consistent with Rule 1.6
Comment 20, says that the duty of confidentiality survives the client’s death.
Yes
Who cares now? You can help someone wrongly convicted and imprisoned
No
Duty of confidentiality survives the client’s death
Under the Restatement, privilege also survives the client’s death

You decide to tell Parnett’s lawyer of Chester’s confession. The lawyer asks for a new trial
for Parnett, and subpoenas you to testify at the hearing on the motion for a new trial. You
are about to tell the judge about the confession, but the judge says “stop right there. If you
testify, I will be compelled to report you to the state bar for violating your former
client’s attorney-client privilege, which outlasts his death.”
What do you do?
Comply with the judge’s command to stop right there.
Judge could sentence you for contempt, which could risk your license.
Testify anyway and face getting disciplined by the bar.
A wrongfully imprisoned man could be free. You have a reasonable argument that this is the
ethical thing to do.
And (Phil says) the judge is wrong. The privilege is just a rule of evidence, not a disciplinary
infraction.
In Real Life
The lawyer testified, the judge reported him, the bar dismissed the complaint. The “Parnett”
man didn’t get off despite the lawyer’s testimony.

4-3 Worldwide Bribery


p. 237 You order end to client’s bribery & notify US Attorney to pay a fee. US Attorney
wants interview summaries and records of bribery. What do you do?
Turn over requested material?
o Being subject to DOJ investigation is not fun
o Who besides employees are affected if you turn over these documents?
Claim privilege?
o What could you cite to conclude that the US Attorney was improperly asking for
summaries?
 He wanted you to waive attorney-client privilege and will be more
lenient…what is wrong with that statement? Client is being asked to
waive privilege which is prohibited because…
 DOJ POLICY - Deputy Phillips Memorandum and Policy Decision
says that they cannot ask (unless it goes to the level of the Deputy
General - US Attorney cannot ask)
Maybe partial cooperation and then claim privilege for others since gov isn’t allowed to ask
124
for certain things

5-1 The Washing Machine


p. 260, Client, who you know because he’s the father of your daughter’s friend, seeks help
because a retail store is suing him because he did not pay monthly installment fees for a
washing machine he got because he lost his job. He can’t afford to pay you, but you
represent him anyway. The contract Client originally signed has a clause that if the buyer
misses a payment, the buyer is in default and the seller is entitled to sue for all the remaining
balance, plus the seller’s attorney’s fees. So, the retail chain sued for that amount. Client
originally offered to return the washing machine, but the store said no.
The store’s lawyer called and asked about setting up a “payment plan” of of $25 a month,
which means your client would end up paying an extra 50 bucks by the end of it. Client said
he could probably pay 25 per month but not 60 and said the payment plan seemed like a
good solution.
What do you do?
DON’T VIOLATE 1.1
 Tell lawyer that your client accepts offer?
 2- Recommend that he make a counter-offer?
 3- Other?
 Further research the K, what is in written agreement
o Returning the machine provision
o Case law
 State law may say that if the sale was negotiated by door-to-doo and
fails to discuss returns, buyer can rescind at any time
 Law about the collection of the debt? Federal Fair Debt Collection
Act
o What other laws protect consumers?
 Predatory lending statute?
 Unconscionable K?
 Common law defense - FRAUD; lied about the quality of the
machine?
NOT JUST STANDARDS FOR DISCIPLINE, BUT ALSO PROFESSIONALISM.
STANDARDS FOR YOU.

5-2 A Desire to Investigate (273)


You are a PD assigned to Ellis Boiko, who was charged with a misdemeanor assault that has
punishment of up to six months in jail. Boiko was charged five months ago, but he only
asked for an attorney yesterday and his trial is scheduled for today. You get the file late in the
morning and speak with Boiko for 20 minutes. Boiko tells you there were five witnesses,
some of whom would confirm he acted in self-defense. You request a postponement; judge
just gives you until after lunch, saying you can interview the present witnesses over lunch.
But you don’t, because you think it’s not fair. After lunch, you repeat your request for a
postponement. The judge orders you to proceed and says if you don’t, you’ll be held in
criminal contempt and jailed immediately; if client is convicted, he can appeal on the basis of

125
ineffective assistance of counsel (but under the Strickland standard, it’s a high bar to prove).

What would you do?


Try the case as best possible and appeal if Boiko is convicted (60%)
Could make a statement on the record
Want to keep a good relationship with the judge who you’ll be in front of again and again
Refuse to try the case and be jailed for contempt (40%)
Tons of publicity, tons of street cred
MRs say you’d be violating the model rules by being incompetent

Real-life case
Lawyer Jones refused to proceed with the trial, and was jailed from the courtroom. Released
on board later that day. Same judge tried Jones for contempt, sentenced him to three days in
jail. Jones appealed, and the appellate court reversed, saying the judge’s denial was an abuse
of discretion and it would have been unethical of Jones to proceed to client and put client’s
constitutional rights to jail.

5-3 Torture *283


You are a new lawyer at the DOJ’s Office of Legal Counsel; your only client is the executive
branch of the federal government. Soon after, 9-11 happened, and the CIA captured Al
Qaeda member Abu Zubaydah. They want to use harsh interrogation techniques on him, but
are worried this might subject them to criminal liability for torture. The president’s national
security advisor authorized the “techniques” but wants to know if they’re legal. You’re
tasked with editing a memo on this question written by a coworker, McKee. The White
House counsel has said he wants the memo to be “aggressive” and risk-taking; your
supervisor, Farrell, agreed. Basically they want a memo to protect the CIA from future
torture prosecution. McKee’s memo concludes that Congress’s anti-torture statute only
applies when an interrogator intends to cause extreme pain – a very strained use of a
Medicare law.
Do you have an ethical obligation to rewrite the memo to qualify its legal analysis?
Yes?
No?
The client wants the memo as is
BUT, rule 1.2: “a lawyer shall abide by a client's decisions concerning the objectives
of representation and, as required by Rule 1.4, shall consult with the client as to the
means by which they are to be pursued”
Rule 5.2: lets junior person off the hook about an arguable question of duty when
their supervisor tells them to do something
If you think the “techniques” are not just criminal but also morally wrong and lead to
adverse foreign policy consequences, would you add those to the memo?
Yes?
What better reason to lose your job?
1.3 Diligence Argument  You have to diligently push on this
If you don’t write the memo, someone will be tortured and the CIA could be jailed
for torture
1.2(d) A lawyer may not help a client do something that is contrary to law
Real Life Case
John Yoo, deputy Attorney General, drafted the memo, and his boss Jay Bybee signed it. It
remained classified for two years, and was used to justify torture. Bush admin refused to
126
release it to Congress, but it was leaked to Washington Post. It caused outrage, it was so
ridiculous.
John Yoo returned to UC Berkeley and became a chair then.
Jay Bybee became a 9th Circuit Judge
DOJ Office of Professional Responsibility (Marshall Jarrett) examined whether Yoo and
Bybee had violated ethics; it included that Yoo had exercised deliberate professional
misconduct and Bybee had acted in reckless disregard for professional responsibility. They
suggested passing findings to ABA. Yoo and Bybee violated MR 1.2 and 1.4.
BUT, with the next (Obama) administration, the new DOJ rejected those findings and no
one took action.
Article about this argued the situation is a common one—corporate and government clients
often ask their lawyers for advice but really want them to cover their asses and bless sketchy
conduct. They will argue you should write a memo justifying their dodgy behavior.

5-4 The Package Bomber (303)


Package-bombing client refuses to submit to psych exam, even though “a mental health
defense appears to be the only way to protect [him] from the death penalty.” Instead, he
wants to argue that evidence should be suppressed, and that his bombings were actually to
protect humanity. Under MR 1.14, since he’s at risk of substantial physical harm, you can
take “reasonably necessary protective action” even so far as appointing him a guardian.
There are two phases of the trial:
Is he guilty, and if so was it first or second degree murder?
Should he get a life or death sentence?
Options
Go with what he wants. Stupid & unethical! He’s of diminished capacity and can’t make legal
decisions for himself. 35%
BUT, how do you know that?
Ask the judge to order a psych eval. Likely to fail, and likely to make client fire you. Doesn’t
prevent the harm, just passes the ethical buck. 12%
BUT, you could argue that under 1.14, you’re taking reasonable protective action by
involving the judge and psychiatrist
Try to persuade him to submit to a psych eval by lying, saying you won’t use the results in
phase 1 of the trial (and he thinks the trial will never get to that phase, because his own
brilliant defense will get him acquitted). Then use the eval results to persuade the judge to
let you make strategic decisions in the case. 54%
This could save his life, but he may fire you as a result and take over his defense and get
executed
But, if he finds out close enough to trial, the judge may not let him fire you because it would
cause undue delay.
Questions
Would any of these options violate any ethics rules or be inconsistent with the Jones case?
What about the 6th Amendment?
Rule 1.2: you must abide by your client’s about the objectives of his representation,
& shall consult with your client about decisions…
Here, his objectives are to save society from technological harm, and to not get a
psych eval
Rule 1.4: you must reasonably consult about the means the client’s goals are to be
met

127
6th Amendment: right to a lawyer (& effective assistance to counsel) – under
Strickland, a low bar. Very few people in criminal cases have success on the grounds
of the 6th.
Jones (6th Amendment case): client doesn’t have the right to demand certain
arguments from appointed counsel, if counsel decides against them in their
professional judgment. Therefore consistent. (although in Jones, you chose
from a multitude of defenses, whereas here you’re going in one opposite
direction)
MR1.14b ethics rule: You may be covered by this since he’s at risk of substantial
physical harm. Therefore, you can take “reasonably necessary protective action” even
so far as appointing him a guardian
If not, do the rules or Jones provide guidance in determining the right thing to do?
Yes
What’s your own moral judgment about these options?
Is it okay to lie to him to save his life?
Lying isn’t okay under 8.4(c)
But, if the guy is mentally ill, that may change things.
A la Tremblay (309), maybe talk to family members about what they think?
Remember that even seemingly incompetent people can make decisions through
supported decision-making with trusted helpers.

If you oppose the death penalty, would you be willing to do what your client asks even
though it would probably kill him?
No.
In Real Life
Ted Kaczynski’s lawyers kept their plan to plead mental illness totally in the dark. The
lawyers persuaded the judge to let them raise the mental health defense. The Unabomber
tried to fire them, but the judge said it was too late. Ted said he’d rather represent himself,
but the judge denied that too, saying it was too late. Ted tried to kill himself in jail, but failed.
The only way to prevent the mental health defense then was for him to plea guilty.
The lawyers managed to get the prosecutor to accept his guilty plea in exchange for a life,
not death, penalty. Ted appealed, & US Court of Appeals for the 9th Circuit denied. He later
wrote that he did not hate his attorneys. Academic wrote that capital defense lawyers more
often act paternalistically. Brennan dissent in Jones v. Barnes: having a lawyer is “one of the
many indignities visited upon someone who has the ill fortune to run afoul of the criminal
justice system.”

5-5 Vinyl Windows *310


An old lady scammed by a vinyl window company is confused about how to proceed with a
lawsuit they’ve taken against her. She has no family and is only close to her neighbor, Mrs.
Houston.
My Classmate’s Course of Action
Ask for more time and talk to her neighbor, probably with her.
Try to figure out her underlying interests or goals
I probably wouldn’t get the company to repair what they did, because that would get them
back in her house where she is vulnerable to being scammed or robbed again
If she still couldn’t decide, I would be tempted to fight a bit at least, so they didn’t think they
could do this again (to her or anyone else). But I’d put her future safety before getting
money back for her.
128
Try to determine her decision-making competence by talking to her
Use professional judgment to try to convince her to counterclaim
When you try to get a guardian for someone, how much info can you give to them?
MR 1.14 Comment 8 says ‘no more info than is necessary’ but need to make sure person
asking wouldn’t harm the client.
I.e. asked about a client? Stop and think and ask before giving out their info.

What if next Dept of Social Services agreed to be guardian, but lady would need to pay fees,
would need to agree to home inspection, and have someone else move to have them
appointed.?
Mrs. McCabe retained you to defend her. Will I:
Make a litigation decision for her unless she countermands you before trial in a couple days?
60%
Ask the judge for permission to withdraw, which will certainly result in a default judgment
against her?
See if Mrs. Houston or the Dept will be willing to be guardian ad litem? (i.e. sharing info
with them, making a motion they be appointed, them getting paid for their time). 29%
Other 12%
Explain the litigation decision as best as possible.
Consult with Legal Counsel for the Elderly or another lawyer who specifically deals with
people with diminished capacity

Scene 2: Mrs. McCabe’s mental capacity has clearly diminished alarmingly, and the hawks are
circling.
Should you file a counterclaim against Stormguard? Should you try to involve anyone else in
her affairs?
Considerations
MR 1.14 Comments 5 and 6 when a client is at risk of substantial physical, financial, or other
harm unless action is taken, and that a normal client-lawyer relationship can’t be maintained.
But Comment 7 says: use the least restrictive measure. Be careful.
Comment 9: emergency legal assistance where “the health, safety or a financial interest of a
person with seriously diminished capacity is threatened with imminent and irreparable harm”
Who
Dept of Social Services or Mrs. Houston
Additional Consideration
Guardian is an extreme step; need to look for other people who can help her in decisions –
social workers, Mrs. Houston, Legal Counsel for the Elderly – there should be more people.

In Real Life
Happened in a Georgetown consumer protection clinic – a $900 small claims case
The students contacted the Dept of Social Services, who arranged government social worker
visit, who decided she was unable to care for herself
Social worker asked Mrs. Houston to be conservator
Students got another lawyer for Mrs. Houston, and persuaded the court to postpone the
vinyl window case
Mrs. Houston told students to file counterclaim on the basis that contractor was unlicensed.
Mrs. Houston sold the house which paid for McCabe’s nursing home
The clinic students got all the money she paid the vinyl window company back, which went
towards her nursing home car
129
6-1 The Injured Passengers, Scene 1
p. 351 Two strangers injured in the same taxi accident. They ask you to represent them in a
suit against the cab driver. Jill suffered minor back injury requiring physical therapy. Reema
had several fractures & needs 5 weeks of hospitalization
Can you represent them both?
Need more information
 Their health needs
 If they really understand the conflicts that could arise by filing together (between
them and with you) i.e. that you representing them both would likely result in them
both getting less
o And that confidentiality would be a thing
o And that Jill especially would be better off getting her own lawyer!
 Would this representation be prohibited by law?
 Would representation of one or more clients will be materially limited by
representing the other?
o Does the taxi company’s insurance have a limit on compensation? If there’s a
cap, the clients might get played off against each other.
o Judge or disciplinary committee might not think you can provide competent
and diligent representation to both, because you’re likely to give more time
to the client who would get the bigger damages
 Are they willing to give written consent? And if they do, what if they have a fight
later? You might have to withdraw, or one of them might.

6-2 Food Poisoning


p. 353. You’re a lawyer at Samuels & Oto. You specialize in food poisoning cases. Fran
Gomez, a fourth-grader, got E.Coli at DeLuca’s. Carlton, another firm’s lawyer, asks you to
be co-counsel for Gomez’s case. But turns out a while back, DeLuca’s lawyer Kowalski had
talked about the case with one of your associates, MacBride. She asks you to withdraw. You
ask MacBride what they talked about, to assess whether a conflict of interest might exist.
Kowalski says that Fran visited a petting zoo that day! You don’t want to withdraw, and
neither does Fran’s family.
Under the rules, will the court likely disqualify you?
Rule 1.7
Rule 1.10
In Real Life
The trial court disqualified your firm because of the risk that the firm’s loyalty would be
messed up by the conversation with Kowalski. The court diverged slightly from the rules,
saying that representation would mess up the “fairness of the proceedings”.
 Phil thinks this was a wrong decision. MR 1.7 comments say the “mere possibility”
of possible harm to material interests is not enough. The whole point is to make sure
that your client is well-represented. By focusing on the “fairness of the proceedings”,
the court strayed from the client-focused purpose of the rule.

6-3 I Thought You Were My Lawyer


130
p. 358 You’re an ethics counsel at a firm. The firm is suing a bus company whose bus hit
Dori Hathaway. While that case was pending, your firm agreed to represent another client,
Kevin Bielaski, to sue his wife for divorce. Her name is Dori Hathaway. Bielaski wasn’t
involved in the accident, and both of them want to get divorced. They have no kids but have
property, and there may be a property division dispute. So Bielaski could come after any
money Hathaway wins. So representing Bielaski could adversely effect Hathaway, or vice
versa.
What should you advise the firm to do?
 First ask: is there a conflict?
o Rule 1.7a: can’t represent when
 (1) the representation of one client will be directly adverse to another
client;
 This applies because it would be Bielaski v. Hathaway.

 OR
(2) there is a significant risk that the representation of one or more
clients will be materially limited by the lawyer's responsibilities to
another client, a former client or a third person or by a personal
interest of the lawyer.
o Then assess consentability under Rule 1.7b:
 Rule 1.7b: (b) Even if there’s a concurrent conflict of interest under
paragraph (a), a lawyer may represent a client if:
 (1) the lawyer reasonably believes that the lawyer will be able to
provide competent and diligent representation to each
affected client;
 (2) the representation is not prohibited by law;
 (3) the representation does not involve the assertion of a claim by
one client against another client represented by the lawyer in the
same litigation or tribunal proceeding; and
 (4) each affected client gives informed consent, confirmed in
writing.
o Asking them both to consent in writing raises a MR 1.6
confidentiality issue – an informational conflict: if Kevin
knows Dori is getting the money, he might go after it.
(But what if they still tell each other everything? That
might change things.) So it’s a non-consentable conflict
under 1.7b.
What if it you had two offices in different cities, each of which would handle a case?
 Doesn’t matter under MR 1.10.
Can you continue to represent one of those clients, or do you have to withdraw from both?
 You can’t represent Kevin, because of the info you know about Dori.
If you choose one, which one should you choose?
 Dori, because you agreed to her first and have already worked on her case a lot.
 What you learned from Kevin shouldn’t really impact the bus accident case, so still
representing her should be fine.
 Kevin’s status with the firm is now a former client. Former clients have some rights,
but not under MR 1.7.

131
IN REAL LIFE
Bus victim fired the law firm after learning they’d agreed to represent their spouse in their
divorce. The firm sued him for a share of the money based on the work they had done so far
on his case. The court said the firm was wrong to have taken on the spouse’s divorce case
without getting both parties’ consent. But that because PR courses weren’t par for the course
at law school, it wasn’t their fault, so they still got some of the money they’d spent on the
case.

6-4 The Injured Passengers, Scene 2


p.364 Cab accident, joint representation and you agreed to represent both Jill and Reema,
who consented.
There is now a settlement offer - 350,000 for Reema and 50,000 for Jill.
How should you respond?
 Stop the negotiations and go talk to the clients
o 1.4(a)(3) requires this AND 1.2(a) requires client’s to approve any settlement
offer
ALSO, does this set up a material loyalty conflict under 1.7(a)(2)?
 Maybe, because now you have a $400k settlement on the table that you’d love to
keep, which might make you want to pressure Jill into accepting.
What if Reema says it sounds great and Jill reluctantly says “I guess so”?
 You might be obliged to say to Jill: get another lawyer, because it’s in your better
interests to go ahead with the litigation and not settle.
 You want to make sure that she understands the offer and actually has informed
consent
o What critical factors does Jill need to know? If she uses different attorney she
may get more $$. The consent may not have been really informed…getting
another attorney may be more $ and more of a hassle so may just go for the
resolution of the problem
Is the acceptance of this settlement a consentable conflict?
 Remember, it’s an objective standard – but doesn’t mean you have to squeeze every
possible dollar out of your opponent.
 If Jill says she’s just fine with all this and just wants it to be done and doesn’t need
the money, then it’s a consentable conflict.

Say both reject the settlement offer. You learn that the accident was caused in part
because of Jill’s drunken swearing. Liability in your jurisdiction is based on comparative
negligence, so if a jury finds Jill was 60% responsible for the accident, Reema will only be
able to collect 40% of the damages.
How does this affect your work on behalf of them?
 This makes them directly adverse because Reema would have to claim against Jill to
be maximally confident of getting the max amount – and we’d have a 1.7(b)(3)
problem (claim by one client against another client in the same litigation)
 Cannot represent both of them without damaging one of them
 If you were representing just Reema you could go after Jill
 Probably have to counsel Jill that it is in Reema’s interest to sue her
Can you withdraw from just Jill and represent Reema?

132
 Rule 1.9(c) would probably have to withdraw from both
 Even though they both want you still, you HAVE to withdraw

6-6 Top Gun (Legal Positions)


p. 369. Your city’s mayor has asked your firm to help the city sue the nation’s big gun
manufacturers, in an effort to curb gun violence in the city. Your firm agreed to do so, pro
bono. The trial is in six months, and you and 14 associates have been working on the case for
two years. The legal argument is that the companies support an illegal gun market that
criminals use to murder city residents, and thus are creating a “public nuisance” in violation
of common law. You’re seeking an injunction to prohibit them selling guns to dealers who
are known to have supplied them to criminals. One of your biggest clients, Global
Pharmaceuticals, calls, concerned that this could create harmful precedent. One of their
former painkillers is addictive and many people died from ODs. Apparently it was easy to
get the drug through a fake scrip at Rite-Way. Some families are suing Global and Rite-Way
for wrongful death. Global’s worried that, like the drug companies, they’re responsible for
harm caused by a downstream purchaser for the product. Global threatens to take their biz
elsewhere.
(1) Do you have to withdraw from representing the city in the gun suit?
Is there a conflict under 1.7? Is representation of the city limited?
   Yeah…could potentially argue a different argument, or be more willing to settle
1.7(a)(2) How do you determine if there is a material risk?
 [Comment 24] Ordinarily a lawyer may take inconsistent legal positions in different
tribunals at different times on behalf of different clients. The mere fact that
advocating a legal position on behalf of one client might create precedent adverse to
the interests of a client represented by the lawyer in an unrelated matter does not
create a conflict of interest. A conflict of interest exists, however, if there is a
significant risk that a lawyer's action on behalf of one client will materially limit the
lawyer's effectiveness in representing another client in a different case; for example,
when a decision favoring one client will create a precedent likely to seriously weaken
the position taken on behalf of the other client. Factors relevant in determining
whether the clients need to be advised of the risk include: where the cases are
pending, whether the issue is substantive or procedural, the temporal
relationship between the matters, the significance of the issue to the
immediate and long-term interests of the clients involved and the clients'
reasonable expectations in retaining the lawyer. If there is significant risk of
material limitation, then absent informed consent of the affected clients, the lawyer
must refuse one of the representations or withdraw from one or both matters
Reasons for withdrawing
 because you’re arguing for a new theory of liability that would be a material
limitation on your representation of Global – MR 1.7a2.
 So you won’t be loyal to the city, because you’ll be loyal to Global and soft-pedal
your arguments in order to avoid a precedent that would hurt Global *368
 Global may have had a reasonable expectation that its law firm wouldn’t hurt its
interests
Reasons for not withdrawing
 Reasonable expectation - own lawyer does pro bono work that might create a
precedent
133
 Temporal relationship - they’re happening at the same time so it won’t be binding
 We’re only in the trial level court, neither case is going to be a significant precedent,
if any
 Practical significance - don’t know if there will be precedent and what the precedent
will be
 Non-Rule Considerations
 Even if you’re not required to withdraw because of the alleged conflict, should
you withdraw from the gun case to avoid making one of your most important
clients uneasy?
 ALSO, city was client first, pro bono or not. Your associates have put two years
of work into this. Terrible PR.

(2) Even if not required, should it withdraw to avoid making one of its most
important clients uneasy?
Yes - good relationship with client
No - PR issue within the city

In Real Life
NYC is the city. The firm withdrew because of “certain potential positional conflicts” re
current clients, “to avoid any appearance it might have weakened its arguments on behalf of
the city.” – i.e. its ethical duties. But ethics scholars were skeptical. The case went to trial
without the firm, and the city won, but the 2nd Circuit reversed it based on the “Protection of
Lawful Commerce in Arms Act”

6-7 The Secret Affair


You started a small law firm with two friends, who are now your law partners. You need to develop a
client base quickly. To attract potential clients, you created a web page announcing that your firm
practices family law, and you put on the front page of your firm’s website a notice stating:
If you are facing a legal challenge involving family law, call us by phone or simply contact us
on line. If you have any questions or comments, feel free to contact us by filling out the form
below.
Below that message, the form asks for the website visitor’s name and email address and provides a
window in which the visitor can enter a question or comment of any length.
Above the window is a legend that reads
“Transmission of information is not intended to create, and receipt does not constitute,
an attorney-client relationship between the sender and the receiver.”
Below the window is a button labeled “SUBMIT”.
You are the only person in your firm who looks at incoming messages. You read and respond to the
inquiries as they arrive. Now in your fourth month of practice, you have received one from Maria
Decorsi. She writes:
My husband, Nicholas Decorsi, is living with another woman. We have been separated
for three months. I am looking for a lawyer to handle my divorce. I want a good property
settlement, and I would like to obtain sole custody of my 13-year-old son, who lives with
me. I had an extramarital affair several years ago. Nicholas doesn’t know about it. I
wouldn’t want him to find out because he’d probably try to use it to prevent me from
getting sole custody. I have enough money to pay you from my savings. But if you
represented me, would Nicholas have to find out about the affair that I had?
You responded as follows:

134
In response to your question, there is a five-year statute of limitations on adultery in your
state. If the affair occurred more recently than five years ago, there still are ways to keep
it confidential. But also, it’s not as detrimental as you seem to think. I would suggest,
however, that you do not leave messages about it on law firm websites. You do not know
if your husband has contacted one or more of the firms you contact like this (his name
does not appear in our records). But if he had and they received this information, it can
be used by them.
Maria replied saying that she had retained a different law firm, and that was the end of your contact
with her.
This morning, three weeks later after Maria told you that she was not going to retain your firm,
Nicholas Decorsi contacted you and asked you to represent him in a divorce action against his wife
Maria.
Was Maria a prospective client?
Comment [2] of Rule 1.18: “A person becomes a prospective client by consulting with a
lawyer about the possibility of forming a client-lawyer relationship with respect to a
matter. Whether communications, including written, oral, or electronic communications,
constitute a consultation depends on the circumstances. For example, a consultation is
likely to have occurred if a lawyer, either in person or through the lawyer’s advertising in
any medium, specifically requests or invites the submission of information about a
potential representation without clear and reasonably understandable warnings and
cautionary statements that limit the lawyer’s obligations, and a person provides
information in response. See also Comment [4]. In contrast, a consultation does not occur
if a person provides information to a lawyer in response to advertising that merely
describes the lawyer’s education, experience, areas of practice, and contact
information, or provides legal information of general interest. Such a person
communicates information unilaterally to a lawyer, without any reasonable expectation that
the lawyer is willing to discuss the possibility of forming a client-lawyer relationship, and is
thus not a "prospective client." Moreover, a person who communicates with a lawyer for the
purpose of disqualifying the lawyer is not a “prospective client.””

Comment [6] of Rule 1.18: “Even in the absence of an agreement, under paragraph (c), the
lawyer is not prohibited from representing a client with interests adverse to those of the
prospective client in the same or a substantially related matter unless the lawyer has received
from the prospective client information that could be significantly harmful if used in the
matter.

Arguments in Favor of Yes


 Consultation can include electronic communications.
 You can be a prospective client without being a client
 The attorney here actually gave advice, so maybe created an attorney-client relationship under
Togstad.
Arguments in Favor of No
 Unilateral information doesn’t create a prospective-client relationship
 The warning says this doesn’t make you our client; it doesn’t warn that you don’t have to keep
the information confidential.
 There was a clear and reasonably understandable warning and cautionary statement limiting the
lawyer’s obligations (Comment 2).

If she was, may you personally represent Nicholas in the divorce case?
NO?
 You receive information from Maria that could be harmful to her case.

135
 1.7(a)(2): there is a conflict if “there is a significant risk that the representation of one or
more clients will be materially limited by the lawyer's responsibilities to another client, a
former client or a third person or by a personal interest of the lawyer”
 But, here there is a risk to Maria, not Nicholas

CORRECT ANSWER
 Under 1.7(d)(1), it is okay if both parties give informed consen
o Under 1.7d1, if both parties give informed consent

If there are concerns about your representation of Nicholas, may your partners represent him?
 Yes, provided the conditions in (d) below are met, even though the conflict would be
imputed under MR 1.10.
 But there’s an argument that under MR1.18d2, you didn’t take “reasonable measures to
avoid exposure to more disqualifying information than was reasonably necessary to
determine whether to represent the prospective client” by having the box with the poor
warning on your website.

7-2 My Client’s Subsidiary


p. 391, You are an ethics advisor for Shelton & Cadenas, which represented Dori Hathaway
who was hit by a bus. She was seriously injured, and started a personal injury suit against the
bus company, Pearl Bus Co., and the driver. Two months later, you discover Pearl Bus is
owned by your client Transport Inc. (who you represent on entirely different matters). You
email your colleagues at your firm, and it seems nobody at your firm has ever worked for
Pearl. You call the president of Transport, Jason Kerr, for more info. He says:
Transport bought Pearl years ago; Transport has taken over a lot of Pearl’s
management functions and pay for a bunch of damage awards against it; After the
last such payment, Kerr insisted Pearl increase its insurance coverage, but helped pay
for it; As of a couple years ago, his son is the president of Pearl.
He asks you why you want to know.
What can you tell Kerr about the reason for your call to him?
 Can say you’re doing a potential conflict check, and that’s it.
o But Kerr might be mad by your mere consideration of doing something in
conflict w/Transport
Must the firm withdraw from representing Hathaway, or Transport, or both?
STEP 1: There is a clear conflict because Pearl is a client. It’s possible the increased
insurance wasn’t enough.
NOTE: one might argue Pearl isn’t a client because:
 You haven’t gotten any confidential info or given advice to Pearl.
 Pearl isn’t controlled or supervised by Transport.
 Harm to Pearl wouldn’t hurt Transport because they just increased
their insurance coverage.

STEP 2: If you can keep representing one or both, must you first obtain informed consent
of one or both of them?
   Yes.
 Regardless, it’s better for you and Dori to withdraw sooner rather than later in case
further conflicts arise.

136
STEP 3: In the absence of informed consent from both, must you withdraw from
representing Dori, Transport, or both?
 If Pearl is your client (which we said yes), then 1.7(a) is the issue since you are
representing Dori 1.7(b)(3) becomes a problem because you cannot sue a current
client
 Argument 1
o alter ego of Transport, two are linked by blood, financial control
 Argument 2
o have you learned anything from Transport that would advantage you in Pearl
representation? No evidence of any confidences provided here, new
insurance policy for Pearl would cover them if there was a judgment against
NOTE: Rule 1.10 screening?
 Does not permit it here and no exceptions apply
 Personal interest? (this is professional interest)
o Association with prior firm? (not here)

STEP 4: Suppose you conclude there is no conflict


 Therefore you do not need consent
 Dori has a pretty good case against Pearl, and your like to earn the fee
Withdraw anyway?
 Court determines if there is a conflict in the end, so what you may have thought was
ok may not be
 Should probably ask for consent under 1.7(b)
 Why might you not want to ask consent?
o Kerr may doubt loyalty, move business elsewhere

7-3 Police Brutality, Scene 1


p. 402, Several cops were indicted after brutally assaulting Louis Alston after his arrest.
Officer Tom Babbage pleaded guilty, but Alston says at least one other officer participated;
two more, Stone and Morton, face charges. The Policeman’s Benevolent Association (PBA)
has asked you and your law partner to represent Stone & Morton; if you agree, the PBA
would pay your fees.
What potential conflicts should you consider in deciding whether you can represent Stone
while your partner represents Morton?
 Is screening possible here? 1.10
o no personal interest, no prior association issue) so the exceptions do not apply. The
first sentence does, so act as if you are representing both.
 May find yourself in a later position where one is blaming the other?
o May find one is innocent and have to pin against another
o Test is an objective standard - what you reasonably believe you can provide
o Consent may or may not help

 You know that one of these two people did it. A plea deal or immunity may be offered. Can
you reasonably provide competent and diligent representation to both?
o Important Process –
 What would the court believe is reasonable for the standard?

137
 Using that standard to apply to the facts and what a reasonable person
would think
 ALSO: Rule 1.8f: you can’t get paid by a third person unless the represented client
gives informed consent, confidences are protected, and there’s “no interference with
[your] independence of professional judgment or with the lawyer-client relationship.”

Correct answer:
YES, if each of you reasonably believes that you could provide competent and diligent
representation and both clients consent in writing
o 1.6 Confidentiality:
o one of them might say something you can’t tell the other because it’s
confidential, but that you feel a duty to tell the other because it’s in their
interest
o 1.7(a)(2) says
o “there is a significant risk that the representation of one or more clients will be
materially limited by the lawyer's responsibilities to another client, a former
client or a third person or by a personal interest of the lawyer.”
o In such cases, “a lawyer may represent a client if:
(1) the lawyer reasonably believes that the lawyer will be able to provide
competent and diligent representation to each affected client;
(2) the representation is not prohibited by law;
(3) the representation does not involve the assertion of a claim by one
client against another client represented by the lawyer in the same
litigation or other proceeding before a tribunal; and
(4) each affected client gives informed consent, confirmed in writing.

7-4 Police Brutality, Scene 2


Alston filed civil suit against PBA alleging (1) enforced code of silence among officers (2) its K with
the PD provides that when police misconduct is suspected, no union member can be questioned for
48 hours. PBA wants you to represent them in civil case although you already represent the officers
in criminal case
Does 1.8(f) preclude this? (more specific, who funds it)
o Professor thinks its okay, but there are possible problems Something could arise later
o In general, there is presumption that people get to choose their own lawyer
Does 1.7 preclude signing the K and representing Morton, Stone, and PBA?
 This is a broader prohibition
 Is this non-consentable?
o In the civil suit, PBA might want to argue that there was no enforced code of silence
among officers, yet the officers might wanna argue that PBA instructed them to do so
o In criminal suit, possibly accuse a member of the board and they have a member of their
association that might in fact be guilty, and the board could fire you and you’d lose the
10 million K
o 1.7(a) would preclude this direct conflict unless consent from (b) and competent
and diligent work from (b)(1)
 Is this consentable?
o PBA and Ds have a common interest, all want criminal suit and civil suits to go away. If
you explain all of this to clients you could reasonably decide you can provide competent
and diligent representation

138
See Notes for more

7-5 Police Brutality, Scene 3


2 other officers say they will testify that Gutman (PBA member that was actually in bathroom, not
Stone or Morton). If you introduce this testimony, Stone may be acquitted, but could be used against
Gutman so they would be found liable for damages in Alton’s suit.
 This is the most exculpatory evidence that there is
 Don’t have any loyalty to Gutman, his position in the PBA may cause PBA to yank 10 mill K
 Under 1.7(b)(1) would a reasonable lawyer think that because this K can be yanked you cannot
adequately represent the officers?
 Would you need to forewarn PBA about the testimony? (with officers consent)
o Probably under 1.4 Communication
 Stone later convicted and appeals for attorney misrepresentation, though he consented in open
court
o Likely to prevail? Strickland v Washington
 1) Lawyers mistake must be serious
 2) D must show reasonably probability that the result would have been different
had there been effective assistance
o 5% of people get their convictions overturned
o Curcio n. 77 case - generally you cannot appeal
 Fulton exception - under unusual circumstances, it is appealable if it turns out
the conflict effected the representation
 Kyler - modification of the Strickland doctrine for conflict cases = instead of
proving actual prejudice, just prove there was a conflict that actually effected
representation
o Here lawyer failed to present witness (not cross) in order to serve PBA - there is an
argument to be made that not presenting the witness was ineffective assistance
 2d. Circuit found this a non-consentable issue - Fulton case
See Notes for more

7a The Leased Car


Missing

Florida Bar Op. 95-4


Sketchy shit where husband met with another lawyer to leave assets to his side bitch &
doesn’t want wife to know
o Confidentiality rule is now at stake - attorney has ethical obligation to protect confidences but to
also communicate to a client information that is relevant to the represent
o Believe confidentiality trumps?
o Other idea that communications relevant
o Concludes that lawyer must withdraw representation; conflict of interest has presented (Rule 1.7)
o OPINION - Disclosure is prohibited. RESTATEMENT - Disclosure is permitted. Loyalty to
wife is required.
In Real Life
 See Representing Family Members in Estate Planning

7-6 Representing the McCarthys


P 411. You wrote wills for the McCarthys, a married couple (Hugh & Joline), after you
explained possible conflicts & got their consent. In the will, they each left his or her property
to the other in case of death. The wills are done, so your job now is update them on tax

139
changes that might impact their wills. Six month later, Maureen Carr retained the firm in a
paternity suit against Mr. McCarthy – and because of a misspelling, this conflict was missed.
Your partner Kenney is representing Maureen. Hugh, who is indeed the father, hired Bucci
to represent him in the paternity suit. Bucci learned about the conflict.
Is there a conflict under 1.7?
Yes.
The representation of one client is directly adverse to another; or there is a significant risk that the
representation of one or more clients will be materially limited by the lawyer's responsibilities to
another client, a former client or a third person or by a personal interest of the lawyer.

Is the conflict consentable?


Need to consider 1.7(b)(1)-(3). It probably is not due to 1.7(b)(1) analysis.
1.7(b)(3)
 the representation does not involve the assertion of a claim by one client against another client
represented by the lawyer is the same litigation or other proceeding before a tribunal; and
o This does not bar consent; this is not the same litigation
o Classis conflict - divorce proceeding to negotiate agreement. Once it goes to divorce
court, cannot have the same lawyer in the same litigation. (b)(3) is pretty narrow.
1.7(b)(1)
 (1) the lawyer reasonably believes that the lawyer will be able to provide competent and
diligent representation to each affected client;
 1.10 imputes a COI to each lawyer in the firm if one lawyer has a non-personal conflict
If you were a single lawyer, would this rule preclude the representation?
o 1) Identify the Conflicts
 Who gets the money - may break up the will, now that he has a child their joint
assets wont have a common end point now aka if you are suing him high
likelihood he will want to take his tax business elsewhere to keep Hugh as client
 Cannot give competent advice to wife bc husband does not want her to find
out; doesn’t know about suit with child. She’s your client so she stands to lose
something if father has to give money to Maureen.
 Couple agreed that information disclosed by one spouse could become available
to the other. But this information was disclosed by Maureen and her lawyer.
 How might your loyalty to couple be compromised by your duty to Maureen?
 You know all about their financials…this would be of interest for
Maureen bc she’ll know what to sue for in course of child support
 There may not be discovery in this case…you have a 1.6 duty to couple
that conflicts with duty 1.1 and 1.3 duties to Maureen
o 2) Does (b)(1) preclude consent?
 Could you get informed consent from wife?
 You would have to tell her about the child…otherwise not truly
informed consent
 This will cause problem with husband, need his consent to tell her
about the suit which he may object to
 1.9(a) AND [Comment 3] (assume no consent)

If the conflict is consentable (it’s not, but still), what must you do to obtain consent?
1.7(b)(4) each affected client gives informed consent, confirmed in writing.
 Meaning you would have to disclose everything to all of them, which is a shit show.

140
Is it a good idea to explain the whole misunderstanding w/the processing to the clients and
refund? Or Stonewall?
 Invites a malpractice suit, but are you at a greater risk if you don’t tell anything?
 Sometimes all people want is a confession of error and an apology
 Most have a notice in K to disclose to insurance company any admittance of
negligence to the company and some company may require you to be forthcoming to
clients
If it’s not consentable, or if a party won’t consent, can you continue to represent these three,
or must it withdraw from representing all three of them?
 Can it withdraw from representing the McCarthys and keep representing Carr?
o 1.9 kicks in, is this a substantially related matter as with the McCarthy’s
o [Comment 3] in course of prior representation, you ORDINARILY would have
learned information that would have been beneficial to Joline. Yes; all the information
about the couple’s assets would be beneficial.
 Why is this the test? Want to avoid inquiring what the lawyer actually learned
from the client
 Can you withdraw from representing Carr and keep representing the McCarthys by
giving occasional tax advice re their wills?
o Carr is a former client so 1.9 kicks in, concerned about whether you are representing
another person in the same substantially related matter
o 1.4(b) duty to inform about things that would affect her interest…knowledge of the
paternity suit may give way to her wanting to separate her assets, rewrite her will so
that Maureen or her child wont inherit it
It’s hard for a former-client conflict to be nonconsentable – but if there isn’t consent, you
have a serious 1.9 problem.
The confidentiality agreement the McCarthy’s signed only applies to info they told the firm –
and you learned about the affair from elsewhere.
- I.e. next time make sure it applies to all relevant info
- But sometimes advance waivers are hard to enforce.

Next step
You’re going to withdraw from representing all of them.
What will you do about disclosing the paternity to Joline, assuming you have no consent to
do so from Hugh or Maureen?
Are you permitted to disclose?
 Permitted under the Restatement, but not required (although cite no authority for
this except for the actual case)
 BUT SEE Model Rule 1.6 Confidentiality

Are you required to disclose it?


 Joline has a reasonable expectation that relevant to the will all secretes would be
disclosed to her (Rule 1.4)
 Why does 1.4 not require? Is it trumped by some other ethical obligation? Would
you be favoring Hugh or Joline?

You could tell Joline you have to withdraw because a conflict of interest has arisen, or
because your husband has a secret I can’t tell you. (i.e. a noisy withdrawal!).

141
 Also could prevent her from filing a malpractice suit down the road.

You could refund everyone their money because it was your firm that messed up and missed
catching the conflict! And apologize profusely to everyone, so you don’t look shady.
 “We have to withdraw, we’re so sorry, here’s your money back. We’ll help you find
new lawyers.”
 People like apologies and admitting error.
 And you’re probably insured, and your insurance might cover the refund. Most
malpractice insurance contracts require you to inform the insurance company to
inform them if you’ve screwed up at all, so they can advise you how to avoid a
malpractice suit! (with counsel from their “loss prevention specialists”)

IN REAL LIFE
The NJ Supreme Court ruled that the spirit of the confidentiality agreement made disclosure
of the existence of the child permissible, just no name.
 BUT, refused to decide if disclosure was mandatory

8-1 Keeping in Touch


449. You worked on a day on taxes for Almond, and kept sending it a newsletter and asking
if they want your help, but Almond never hired you again.
Is Almond a current client, a former client, or neither?
 Current: Almond might think that you haven’t terminated the client-lawyer
relationship; you’re still giving them legal advice via a newsletter.
 Former: the client repeatedly declined to take you on for additional services. It’s
been five years, after one day of work. Calling such a company a former client
would restrict the marketplace.
Assuming Almond is a former client, can you represent Star, who wants to sue Almond for
unpaid bills, without Almond’s consent?
Maybe, because of the info you learned about Almond’s assets when you worked for them
five years ago.
BUT under MR 1.9 you probably can represent Star, because the representation isn’t
substantially related to the new matter.
 MR 1.9 (a) A lawyer who has formerly represented a client in a matter shall not
thereafter represent another person in the same or a substantially related matter
in which that person's interests are materially adverse to the interests of the
former client unless the former client gives informed consent, confirmed in
writing.
 (c) A lawyer who has formerly represented a client in a matter or whose present or
former firm has formerly represented a client in a matter shall not thereafter: (1) use
information relating to the representation to the disadvantage of the former
client except as these Rules would permit or require with respect to a client, or
when the information has become generally known; or (2) reveal information
relating to the representation except as these Rules would permit or require with
respect to a client.

142
8-2 Toxic Waste
P 451. You want to represent the state in a suit against NCC for flourochemical pollution. 20
years ago, a former partner at your firm represented NCC in its FDA application to produce
saran wrap containing flourochemicals. You have no records of this, and the partner has
died.
If he were alive, could he properly participate in suing NCC?
Probably not. 1.9 Comment 3 (“substantially related matter”)
 Is this substantially related?
o Passage of time can render conflict obsolete. Different factories from back
then, technology advances
o Both deal with the pollution of this chemical
o Highly likely Max reviewed the studies
o Don’t need to know exactly what you learned, just what a normal lawyer
would have learned
o Possible he doesn’t remember enough that would matter.

Assuming no, and he’s dead, but his wife’s at the firm.
Are other lawyers in the firm precluded from handling the suit?
 1.10b When a lawyer has terminated an association with a firm, the firm is not
prohibited from thereafter representing a person with interests materially adverse to
those of a client represented by the formerly associated lawyer and not currently
represented by the firm, unless:
 (1) the matter is the same or substantially related to that in which the formerly
associated lawyer represented the client; and
 (2) any lawyer remaining in the firm has information protected by Rules 1.6 and
1.9(c) that is material to the matter
BUT there is another lawyer there…what WOULD have been revealed to her should be the
test even though she doesn’t remember…

Suppose the firm’s representation of the state is barred by Rules 1.9 and 1.10.
Would a court be required to disqualify the firm from doing so?
 NO: The courts aren’t bound by the disciplinary rules. The rules just apply to
lawyers. The courts can make their own decisions (see Comment 20, Preamble &
Scope section of the MR). (unlike in disciplinary proceedings).

Suppose that the firm can’t sue NCC


Can it at least try to reach an amicable settlement with NCC, deferring to a different law firm
if litigation becomes necessary?
 NO: it’s still representation, whether it’s settlement negotiations or litigation. MR 1.9
refers to representation.

IN REAL LIFE
Covington incurred more than $2mill in discovery, representing the state against 3M, but
then the court disqualified Covington for its prior work for 3M. On remand, this was
theoretically upheld on the basis of 1.9 – but then waived it because 3M had waited 16
months between discovering the conflict and bringing it to the court’s attention!

143
8-3 A Brief Consultation
p. 471, Client, Summit Bank issues a loan to Headley Farm. Accuses Headley of violating the
agreement and failing to comply with certain restrictions on the use of the loans. Brenda
became new partner, implement screen to prevent her from learning about Headley litigation
since she was in the negotiation beforehand. 3 months later Headley says there is a conflict
and will not waive.
 screening must be reasonably adequate
 Screening pertains to rule 1.10 specifically, she was screened immediately upon
joining the firm
 Need certification of screening 1.10(a)(2)(iii) but there is not written request from
former client so Gucci
o (a)(2)(ii) - didn’t include the statement that review may be available before a
tribunal?
 Is there a 1.7 problem? Could she be considered as still representing Headley Farms?
o When you withdraw from a law firm are you withdrawing from all
representation?
 What’s the test if you are somebody’s lawyer?
o Client’s perception is what decides!!
o Arguable a position that she never withdrew and is still Headley’s lawyers
cannot be suing
See Notes

The Fordham Case


When is a fee excessive? Timothy Clark was stopped for speeding, and was charged with
drunk driving. His dad consulted lawyers, who quoted him $3000-$10000 represent him.
Dad met Fordham installing a burgler alarm on his house, and hired him. Fordham ended up
charging him $50,000 on the basis that he worked a 40 hour week for seven weeks. Dad paid
$10,000 but refused to make any additional acquittal, and complained with the Board
concerning Fordham’s fee.
Should F be disciplined for charging an excessive fee?
 Rule 1.5: Factors include
o The novelty and difficulty of the questions involved, and the skill requisite to
perform the legal service properly
o The fee customarily charged in the locality for similar legal services.
Is it okay, if the client agrees, for a lawyer to charge a client to learn a new area of law?
 Maybe yes, if you discuss in advance how long you think it will take – i.e. the basis of
your hourly rate, and your estimate of the time required.

9-1 An Unreasonable Fee?


Colin charged $60,000. What else would you want to know?
 Did hourly rates stay the same
 Look at Rule 1.5 for the factors
o Rule 1.5 should make lawyers give estimates of time and money, tell clients
what other lawyers would charge, make sure the client understands. Also
doesn’t require any fee estimate to be in writing!! That’s crazy! Periodic
billing might help as well.

144
Scenes from a Law Firm, Questions 1, 2, and 5

P 508. “Nicholas Farber” do any of these firm practices violate the ethics rules?
 “find a reason to call someone on each case once a month, bill for the call and for
writing a follow up memo
o Billing for talking about a case while at a ball game
o Make small modifications in standard forms and bill as if you had created the
document from scratch
o Move billed hours from one day to another so insurance company auditors
won’t question how many hours you spend in one day
 This one is pretty clearly lying. It’s a falsehood.
 Farber left his job.
 He stayed at the firm for over a year – super stressful
 Constant mental tug-of-war between what was ethical and what was required by
the partners
 He wanted to stay a year; left without having another job, and new firms figured
he must have been fired – he said he left for ethical reasons. He was unemployed
for 8 months.
o At interviews he would ask blunt Qs, like ‘how many hours do you
require someone to bill a year?”
o He found one with no billable requirements – that instead monitored the
workload, not the billable hours.
 The firm imploded a few years later due to money arguments between the
partners.
 He stayed at his new firm for four years, and then went to the government.

9-3 An Impoverished Client


You represent an impoverished client in a discrimination case, and if you won you’d get a
third of his $50,000. He’s super broke, can’t buy meals, is about to become homeless, and
phone has been disconnected.
Can you pay three months rent, give him grocery money, and buy him a prepaid cell phone?
YES:
 MR 1.8(e) could allow the cell phone
NO: (probably more right!)
 MR 1.8 Comment 10: can’t make loan to client for their living expenses

10 states have not adopted MR 1.8 re: financial assistance


Is this a situation in which you would violate a Rule?
1: comply and let the client starve
2: violate the rule, give the client money
3: other:
 Look into other resources for them
 Set up client meetings with meals
 Crowdfunding or raising money for other people to pay for him

145
Every state has a bar counsel who you can ask for an informal opinion about a possible rule
violation – risky, but it could cover your butt, especially if you follow it up in a writing to the
bar counsel reiterating their advice.

IN REAL LIFE
In CALS clinic, before the MD immigration court, an administrative agency. Client was
about to be evicted, and couldn’t afford to call the lawyers. DC had a rule that was slightly
different – let lawyers give “financial assistance which is reasonably necessary to permit the
client to institute or maintain the litigation or administrative proceedings”
 Which jurisdiction would apply? (i.e. DC’s choice of law rule – DC or MD). Hard to
parse the law!
 Students raised money for the client.
Thereafter, DC choice of law rule changed, so MD’s law would apply.
And a bar opinion upheld the ban on MR 1.8’s prohibition on helping indigent attorneys.

10-1 The District Attorney


P 569. As a private lawyer, you represented a 5 year old boy who got head injuries in a car
accident. 15 years later he’s charged with murder. You’re the DA (i.e. the prosecutor).
- Rule 1.11 sends you to 1.7 and 1.9.
o 1.1(d)(2)(i)
 Need DA’s (government agency) consent
o MR 1.11(d)1 – a government lawyer is subject to the conflicts rules re current
and former clients
o 1.7 Conflicts re current clients
 Current client is the people
 Test (a)(2) - is there a significant risk that the representation of one of
your clients will be materially limited by the lawyer’s responsibilities to
another client
 You may be fond of the kid that you once represented, you’re a public
figure and if you’re pulling punches you may not get reelected
o 1.9 Duties to former clients
 A lawyer who has formerly represented a client in a matter shall not
thereafter represent another person in the same or substantially related
matter in which that persons interests are materially adverse
 Comment 3: “substantially related” = if there’s a substantial risk that
confidential factually information that would normally have been
obtained in the prior representation would materially advance the client’s
position in the current matter.
 What is material adversity?
 Would the fact that the DA had information that D was not
mentally impaired at age 5 materially adverse?
 Is there an imputation to your subordinates?
 NO. 1.11 [Comment 2]
- Supreme Court of Connecticut - after notice of mental defense, DA had to disqualify but
different ADA at office was allowed to

10-2 A Trip to Monte Carlo

146
p. 590, Should defend your client zealously, but no proof they discussed the case on the
vacation
 vs. a reasonable person, objective standard, would conclude that there was a bias
 Like Scalia’s hunting trip?
o Different bc this is SCOTUS and cannot be appealed farther? Vs. a trivial K
clause?
o What did Scalia get out of it? Fly on Air Force 2 from a party in the case…

10-3 The Judge’s Former Professor


Missing
Professor Giller’s Hypo
Missing

11-2 Flight From Sudan, Scene 1


620, For asylum, must show a well-founded fear of political or religious persecution in his
home country. Editor in chief states that he knew all of his employees and that Barrgabi
never worked for him either under that name or another name.

Should you tell Barragabi about the convo with Al-Parah despite the possibility of
suicide?
YES
 Rule 1.4 requires updating a client of their case
 You need to look for any explanation of Al-Parah’s denial
 You want to know the real reason he’s scared because that might make it a stronger
case (or at least just the truth)
NO
 He might kill himself
 If you really want to get asylum for Barragabi, maybe you don’t want to know he’s
lying

Barragabi is baffled by Al-Parah’s denial. He is sticking to his guns. Will you file an
affidavit with Barragabi’s story as is?
Yes
No
Want to dig more into it
 If he is caught lying, he will definitely lose his case, and it will affect your reputation
too

Since you can’t tell who is telling the truth, should you ask the judge for permission
to withdraw from representing Barragabi? What’s coming up is the oral testimony.
YES
 Rule 1.16: you MAY withdraw if you reasonably believe he’s being fraudulent
 You don’t KNOW he’s lying
 Withdrawing would signal to the judge that you think he’s lying
 You might leave Barragabi without a lawyer and time to get a new one
NO

147
 Rule 3.3
 If the client tells a different story on the stand, you will know and then have an
obligation under 3.3 to take remedial measures: Comment 10 says what those
measures are: talk to your client, and if your client doesn’t come clean, you have to
tell the court. Your duty to report continues until all appeals are exhausted.

Did you violate MR 1.6 by contacting Al-Parah and discussing B’s case with him
without first obtaining B’s consent, or was his agreement to let you seek
corroboration sufficient?
YES
 Should have asked B for a list of people to talk to to corroborate, and checked with
him first
 Rule 1.4 need to communicate an important update in a case.
 Rule 1.6 lets you reveal info that is impliedly authorized in order to carry out the
representation – but client doesn’t know how confidentiality works here
 Al-Parrah might be a government spy – you might be putting your client at super-
risk
NO, his agreement to let you seek corroboration was sufficient
 His agreement to let you seek corroboration was sufficient (81%)
 Rule 1.2 ends vs. means (client controls the former, you control the latter)

Should you have asked permission in advance?


YES
 Him saying ‘don’t call’ doesn’t mean B’s lying, or that you think he’s lying
 Could say “I’ll only help him if he would help your case.”
NO
 Best to remain ignorant of whether or not he’s lying, because if you don’t know, you
can let him on the stand. “Deliberate ignorance.”
o This is what many criminal defense lawyers do routinely: they don’t ask their
clients “did you commit the crime?” Because if they admit they did, they
can’t let him on the stand to deny he did the crime. And no matter what, a
defendant is at a disadvantage in a criminal case, with the force of the US
government against them.

Advisory Considerations
The ABA has written about this issue, saying “lawyers who decline from questioning their
clients may be violating 3.3 and 1.1.” Opinion 87353
MR 1.1: in most cases, competence requires the lawyer to know everything they can about
their client’s case. But in some criminal and imm cases, the risk of knowing outweighs the
risk of not knowing – and then you have a really hard judgment to make.

You decide to confront Barragabi. He says Al-P’s memory must be faulty. He ends
your representation of him, and gets a new lawyer, Ms. Badrasian. Should you
o 1 – send your file to new lawyer and say nothing to her
 Maybe you’re helping him out by saying nothing to her; maybe it’s her
problem now.
o 2 - Send her the docs but warn her you don’t stand by it?

148
MR 4.1 You shouldn’t lie to a third person, or fail to disclose a material
fact to a third person when necessary to avoid assisting a fraud by a client
 MR ? you can’t help a client commit a fraud
 If all lawyers just pass the buck, this could cause systemic problems.
o 3 – Send her the file but refuse to send the draft affidavit?
 MR 1.16d: on ending representation, you have to “take steps to the
extent reasonably practicable to protect a client’s interests, such as …
surrending papers and property to which the client is entitled”
 It’s your work product – is your client entitled to it? Not necessarily. The
rules are fuzzy here and different from state to state.

What if you filed the affidavit before the call to Al-Parah, but now believe it’s false
and Barragabi has just discharged you and hired Badrasian?
 Now you’ve told the tribunal something you reasonably believe is false, so now you
need to ask the court to withdraw it.

Turns out B was an assassin for the ruling group! (According to a neighbor who
spoke confidentiality). Should you tell B about the neighbor’s allegation?
- YES
o He’s your client. You have a duty under 1.4 to communicate with your client
about something material.
- NO
o Rule 1.4 (c)(7), you can withhold info that would cause the client to have an
imprudent reaction (although not for your interests or the interests or
convenience of another person)

Additional Considerations in re Rule 3.3


[Comment 8] cautions that a lawyer cannot ignore an obvious falsehood
[Comment 10] if a lawyer knows that a client has testified falsely, required to persuade the
client to correct the record
Comments 15 + 6 are also important

11-3 Flight From Sudan, Scene 2


B’s roomie testified at B’s hearing, where he doesn’t mention that his provisional
recommendation for asylum was withdrawn. Should you tell the court?
YES
NO
 It’s not a lie, it’s just not a full statement of the truth
o Under the Bronston statement, the roomie’s statement is not perjury
 MR 3.3 doesn’t apply, because 3.3 applies to a material fact – this is immaterial, and
doesn’t apply to a fraud or a crime
o It’s the government’s job to ask good questions, and they screwed up
o Plus the government is in a position to know the status of roomie’s case.

IN REAL LIFE
During recess after the roomie’s testimony, the government’s lawyer approaches you
and asks “why you didn’t introduce into evidence a copy of the docs showing he was granted
asylum?” The student said “I have to go to the bathroom!” The government lawyer got
149
suspicious and later called Phil to complain that the student had broken the rules (which is
not actually true).
 If the judge grants B’s testimony and says she did so on the basis of M’s asylum
status, and the hearing is about to close, what should you do? Correct the judge, or
no?

OJ Simpson
Redecorating Simpson’s house is
1: ethically proper:
 If there’s no rule against it, you can do it
 You should do it to advocate zealously for your client!
 In an adversarial system, the defense’s job is to push the envelope, and it’s the
prosecutor’s job to push back (and the judge should be ethical too.)
2: ethically improper:
 Rule 3.5a: A lawyer may not seek to influence a juror by means prohibited by law
 Rule 3.4: A lawyer shall not unlawfully … alter a document or other material having
potential evidentiary value
o Comment 1: Fair competition in the adversary system is secured by
prohibitions against destruction or concealment of evidence …
 Rule 8.4c: catch-all anti-fraud, dishonesty, deceit, or misrepresentation rule.
Ezra Edelman (son of Georgetown law Prof.): exerpt from documentary he made re OJ
 “If the jury were Mexican, we would have had a mariachi band out front, and a
pinata at the top of the stairs!”
 Judge Ito really screwed up here.
 Assuming it’s not unethical to redecorate OJ’s house, and you were representing him,
would you redecorate his house?
 In a criminal case, the defense doesn’t have any burden to present any evidence at all.

11-4 The Drug Test


You work in a small law firm and represent Frederic Krause in a divorce action against his
wife, Maria. Maria’s lawyer has made a motion to suspend Frederic’s right to visit the
couple’s three-year-old daughter, Darlene. Maria claims that Frederic uses
methamphetamines. Under the law, a judge may suspend the visitation rights of any parent
who currently uses illegal drugs.
The judge scheduled a hearing on Maria’s motion in five days. In preparing for the hearing,
you asked Frederic whether he was using methamphetamines. He said that he did so at one
time while living with Maria, but that he had not used this drug in more than four years. You
advised him to get a drug test to prove that he was not using methamphetamines. Frederic
then went to your town’s hospital and asked to have his urine tested for methamphetamines.
He instructed the hospital to report the results to you.
The hospital’s laboratory technician telephoned you and reported that Frederic tested
negative for methamphetamines. However, the test screened for eight substances, and
Frederic had tested positive for marijuana. She offered to send you the printout.
There were no allegations of marijuana use in Frederic’s legal case, so you told the technician
that you needed a report showing only the methamphetamine results. You asked her to run a
new test, for methamphetamines only. She said that she could do only a multisubstance
screen.

150
May you thank her, tell her not to bother sending you the printout, and send Frederic to a
different laboratory, one that will test only for methamphetamines?
 Under 3.4a, you can’t destroy the initial report
 Under 3.4d, you couldn’t hide the report from a discovery request
 Under 3.4f, you can’t ask someone to refrain from giving info to another party
 But in the absence of a discovery request, you don’t have an obligation to turn over
adverse evidence.
Assuming there’s no ethical prohibition, and that there’s time to get a new test for Mr.
Krause, will you do so?
 If you don’t do it, you’re not zealously advocating for your client

Would it make a difference if he had tested positive for PCPs, not mj (PCPs can sometimes
induce irrational, violent behavior)? Would you still send him for a meth-only test? Assume
the child isn’t independently represented.
 No, because of the best interest of the unrepresented child.
 Some states have a rule that you have to consider the interests of an unrepresented
child.
 Could counsel your client under MR 2.1.
Forget about PCP. Now he has only tested positive for pot, which the lab results show. The
technician offers to send you a report that doesn’t show the pot. Would you accept?
Would you offer it as evidence in the court case?
NO:
 MR 3.4a you shouldn’t unlawfully alter, destroy, or conceal a document or other
material having potential evidentiary value. A lawyer shall not counsel or assist
another person to do any such act.
o But what does unlawfully mean? There has to be extrinsic law (statute, case
law) that makes the alteration bad.
 MR 3.4b: you mustn’t falsify evidence.
 Under MR 1.3, maybe it’s not diligent to expose your client to getting busted on this
YES:
 As an evidentiary matter, you’d have to be careful about how you characterize the
document (i.e. not call it the “full report”)
IN REAL LIFE
The lawyer introduced the report with the marijuana line cut off. If the judge asks, “Is this
the entire report?” Which of these responses is best?
OPTION 1. Lie. 0%
OPTION 2. Cage. “This is the full methamphetamines screen”
OPTION 3. The whole truth including disclosing mj use.
OPTION 4. Withdraw the exhibit.

IRL: the lawyer answered #2: “That’s what I have judge, that’s what I asked them to screen
for.” The judge told the state’s attorney, who brought criminal charges against the lawyer
and initiated disciplinary proceedings. The lawyer was fined and pilloried in the press, and
the state SC ordered that he be publicly censured for trying to mislead the court. Said he
violated MR 8.4 for intentionally misleading the court and evading the judge’s question. 7
years later, he was disbarred for other misconduct.

151
11-5 The Body Double
*634. Would you get a messenger at your law firm who looks like a defendant to sit beside
you to confuse a police officer who has to ID the defendant?
No
– it’s ‘dishonesty, fraud, deceit, or misrepresentation’ under MR 8.4c
o Also, could hurt other clients in the future, due to your reputation suffering if
this came out. (But funnily enough, there’s not a single MR about your
responsibility to your future clients.)
IN REAL LIFE
The lawyer got the messenger to sit beside him. The police officer misidentified him. The
messenger testified telling him his alibi. The judge found the lawyer had “misrepresented by
inference”. The state SC voted 4-3 upholding the judge’s finding that the lawyer was in
criminal contempt by defeating the judge.

11-6 Refreshing Recollection


638. There’s no rule on coaching except MR 3.4b, where you can’t counsel or assist a witness
to testify falsely. There’s a Restatement and some case law on witness coaching, but it’s
ambiguous.

11-7 Child Pornography


*649. You’re lawyer for a church, and find out the choir director’s laptop is filled with child
porn after he loaned it to someone else. The church wants to cover it up.
 NOTE: the choir director isn’t your client, the church is.
What will you do?
1. Delete the files 0%
2. Keep the computer as is in your office 8%
 At least the evidence is still there
 And it’s a pretty safe place, because usually prosecutors don’t search lawyers’ offices
 And it’s not likely to be found
 Does 3.4a apply? Arguable whether or not a lawsuit can be foreseen (Comment 2)
3. Return the laptop to the choir director 4%
4. Tell the rector to keep it in his office 8%
 Client sets the goals
 Church doesn’t have a legal duty to disclose the material on the laptop
 Keeping the laptop would implicate yourself or the rector
 So don’t ask don’t tell; give it back and tell him to get a lawyer (under Rule 4.3)
5. Deliver the computer to the FBI 46% (switched to 75% after more info got)
 protects yourself at the expense of your client (“don’t forget, when push comes to
shove, make sure it’s your client who goes to jail.”)
6. Destroy the hard drive or advise the rector to do so. 0%
7. Other (be prepared to specify). 33%
 Consult with the state bar’s ethics committee
 Hold the computer and get more info (do you have duty to report? Did he take the
photos himself?), try to get the rector to go to the cops

ANALYSIS

152
Rule 3.4(a): you shall not “unlawfully obstruct another party’s access to evidence or
unlawfully alter, destroy, or conceal a document or other material having potential
evidentiary value. A lawyer shall not counsel or assist another person to do any such act.”
 (but here, unlawfully refers to an extrinsic requirement: the destruction must be
unlawful.)
o The Sarbanes-Oxley law *646-7: covering up things is not okay (like in Yates
v. US the destroying fish as evidence case we read in Writing! – but is this
more like the fish, or more like Enron’s files?)
 Comment 2: “applicable law may require the lawyer to turn the evidence over
to the police or other prosecuting authority, depending on the circumstances”;
also “Applicable law in many jurisdictions makes it an offense to destroy material for
the purpose of impairing its availability in a pending proceeding or one whose
commencement can be foreseen.”
Rule 1.6(b)(1) can break confidentiality for reasonably certain substantial bodily harm
Rule 1.13(c)(2) if you “reasonably believe that the violation is reasonably certain to result in
substantial injury to the organization, then the lawyer may reveal info.. whether or not Rule
1.6 permits disclosure, but only if and to the extent the lawyer reasonably believes necessary
to prevent substantial injury to the organization”
 I.e. go to the church council before the FBI and let them know, and counsel them to
take it to the FBI.

Case law: State v. Olwell and Morrell v. State both say that if a client delivers physical
evidence to you, you may examine it but then must turn it over to the law enforcement
authorites within a reasonable period of time

Does it matter that the client is a church?


Maybe they should be held to a higher moral standard. Rule about advising clients on moral,
economic, social, and political things = okay. (MR 2.1)
 1.13(c)(1)&(2) – a hidden exception to MR 1.6!

IN REAL LIFE
Russell was a criminal defense lawyer; Tate was the choir director at George HW Bush’s
boyhood church. Russell was present when Tate resigned; he smashed the computer. He was
indicted under Sarbanes-Oxley. The government took the position that actual knowledge
of an ongoing investigation was not a necessary element. Russell challenged the indictment
in court, but ultimately pled guilty to misprision of a felony – an old, rarely-prosecuted
common-law crime of covering up a felony. Tate admitted to a 40-year history of sexually
abusing boys in other countries, but because it was abroad, couldn’t be charged on that. He
was charged for 5 years, $50,000, and had to go into a sex offenders education program.

11-8 A Letter to the Editor


672. It’s a state court judge trial, not a jury trial

MR 3.6b2 – you’re fine to reveal info contained in a public record


 Comment 5 lists subjects that are likely to have a material prejudicial effect on a
proceeding
Should you send the letter?

153
YES:
 Prosecutors are publically elected, so could get public pressure
 “failure to protect laws” are bullshit and unjust
 MR3.6 Comment 7
 Under Gentile case, maybe you have a constitutional right to send the letter?
REVISE:
 “To reprosecute her is abominable” – an inadvisable personal attack on the
prosecutor

Harry Subin’s Case


*676. Interesting because it was a favorite hypo of professors.
Is it okay to question a witness to cast doubt on her credibility, when you know she’s telling
the truth?
 MR 8.4c – can’t engage in conduct involving dishonesty, fraud, deceit or
misrepresentation – N/A (but could argue that this is misrepresentation)
 MR 3.4b – can’t counsel or assist a witness to testify falsely, or offer an incudement
to a witness – N/A
 MR 1.1 Competence – shall provide competent representation
 Restatement § 106: professional standards permit such cross-examination *675: you
can “cross-examine a witness with respect to testimony that the lawyer knows to be
truthful, including harsh implied criticism of the witness’s testimony, character or
capacity for truth-telling”
 And this is victim-shaming, and punishing her for reporting a crime – thus freeing
the guilty, harming the innocent, and deterring people from reporting crimes. A
horrible misuse of the legal system! I.e. MR 8.4c should be read in the social context.
 On the other hand, there are social costs to having defendants not trust their lawyers.

IN REAL LIFE
Prosecutor offered a deal, so Subin never did the cross-examination. In hindsight, Subin was
horrified that he had considered the strategy, and suggested changing the rules.

12-1 Emergency Food Stamps


*689. Lawyer Bill Simon told his paralegal to tell a welfare office director she was actually his
boss’s secretary. This maneuver worked, and the client got his food stamps.
Did he violate MR 4.1?
 truthfulness in statements to others – can’t “make a false statement of MATERIAL
fact or law to a third person” but this is less than material.
Did he violate MR 8.4c?
 can’t engage in conduct involving dishonesty, fraud, deceit or misrepresentation.
o Comment 1: can’t knowingly assist or induce another to do so, or do so
through the acts of another.
Did Simon do the right thing, even if it violated a Rule?
YES
 no victim. Welfare office director (and staffer shielding him) was doing the wrong
thing (i.e. a utilitarian argument – but does that mean a lie deserves a lie?) and needed
154
to be called out. This got the client the food stamps. You’re being an effective
advocate.
 Juggling JUSTICE and TRUTHFULNESS – two values underlying the rules
NO
 he didn’t just lie; he got someone else to lie.
 Furthermore, there were other ways to get the guy food. Or could have picked up
the phone and called himself, or could have gone in in person himself – i.e. he was
nowhere close to exhausting other remedies.
 Could this impact future clients? (i.e. maybe the director might never trust him
again).
 Lawyers shouldn’t just be able to make up their own justifications to justify breaking
the rules.
 Plus public thinks lawyers are all liars and manipulators, so lying reflects poorly on
the legal profession as well.

12-3 The Break-In


*718. A mother breaks in and finds evidence her ex is molesting their five and seven year old
daughters. She copies it and brings it to you. She wants sole custody. She thinks he’s about
to kidnap them and bring them to Kuwait. He recently got passports for the girls.
 MR 1.1: competent representation
 MR 1.2d: can’t counsel a client to engage a client or assist a client in conduct you know
to be criminal or fraudulent, but can counsel them about legal consequences
 MR 3.3b:. if client has engaged in criminal or fraudulent conduct, you have to take
reasonable remedial measures, including if necessary, disclosure to the tribunal
 MR 3.4a: can’t alter, destroy or conceal a document or other material having potential
evidentiary value. Here, (Comment 2), a proceeding’s commencement can be foreseen
 MR 4.2 – can’t talk to people directly if they have a lawyer
 MR 4.4: doesn’t apply here (see underline in Comment 2)
 MR 4.4a: can’t use means to obtain evidence that violate the legal rights of a third person
 MR 8.4b (misconduct): you’ve received stolen property, which is a crime. Are you thus
putting yourself in criminal liability?
o Maybe technically not stolen, just copied
 MR 1.5: don’t have to report client’s past crime in which your services were not used
 Some courts let lawyers bring forward stolen evidence on the grounds that the material
would be obtained through discovery anyway *719
 In most states, lawyers are excempt from child abuse reporting laws.
What to do?
- Turn the photos over to the police
- Go to Child Protective Services
- Do a discovery request
- If you use these at all (i.e. discovery request, cross-examination), are you risking your
client being criminally charged? He goes to jail for kiddie porn, and she goes to jail
for breaking into the house.
- Get a prosecutor to use their discretion to do justice here?

155
IN REAL LIFE
This case was the first case that ever came to Prof Lerman’s clinic. The dad could visit with
the kids while the case was pending. The law students observed the visits. The kids never
talked about the abuse. But on a trip with another family, the kids told the other kids about
sexual abuse, who told their parents, who told the mother. That’s why she broke into Ron’s
house. Lerman met with Ron’s attorney and showed him the documents, using them to
negotiate for the mom’s custody (but not saying where she had got the docs). The lawyer
was shocked and chagrined to learn of his client’s abuse. On the morning of the trial, the dad
agreed to give sole custody to the mom. Lerman was primarily concerned with danger to the
kids, but also didn’t think the mom would have been prosecuted for theft, had she reported
it.

12-4 The Prosecutor’s Masquerade


*725. Tempted to misrepresent yourself in negotiation with free ax murderer who wants a
lawyer named Biaggi, cigarettes, and to be separated from other prisoners
Will you:
1. Say you’re a PD, try to work out the terms of his surrender, and also try to get him
to confess?
2. Tell him you’re a PD and try to work out the terms of his surrender, but don’t
seek a confession?
3. Don’t lie to him; wait until he gets a lawyer, even if he might kill more people
before then?
Relevant Rules
 MR 3.8 (prosecutors)
 MR 4.3 (dealing with unrepresented persons): (“in dealing on behalf of a client with a
person who is not represented by counsel,” can’t state or imply you’re disinterested;
must correct any misunderstandings
 MR 8.4c dishonesty, fraud, deceit or misrepresentation
IN REAL LIFE
Decided to impersonate a public defender, and was disciplined for misconduct; suspended
for violating 4.3, 8.4c. Had to retake the MPRE!

Legal Services for the Poor (pp 773-774)


Hard to take in all the stats at the beginning of chapter 13 about the unmet need for legal
services
Class action – because it restricts strategic litigation that changes unjust laws
When Phil was a kid lawyer, there were backup centers for every civil legal subject, which
brought a series of class actions that brought about huge changes in the law – and so they
were shut down! Too successful!
Government actors who fight against low-income clients do so even when there’s not much
money at stake
Bleak House 1968 about Phil’s first year as a lawyer – how fiercely big corps change against
any change to the status quo, even when it’s just a change that would help low-income
people.

6-5 The Prisoner’s Dilemma


*365. You’re a legal services lawyer, working on a class-action suit on behalf of women
prisoners, trying to force the state to build a womens’ prison. A colleague at a different

156
office of your org is handling another class-action suit on behalf of people with disabilities
housed in a dilapidated facility called the Piper State School, who seek to require the state to
build a new home for intellectually disabled people. The state proposes moving half of the
patients at Piper to group homes, and renovate Piper to turn it into the womens’ prison.
What to do?
o The two classes have conflicting interests
o Two lawyers work for same org – but is an org the same as a firm? Yes, MR
1.0c says yes. But its Comment 4 leaves it ambiguous. For now, we’ll assume
they’re members of the same firm.
o Both bring it to your clients to see if they agree! MR 1.4 and MR 1.2. Have to
convey a settlement offer to your client, and have to keep your clients
informed
 in which case it might be consentable? Yes, MR 1.7b makes it potentially
consentable.
 MR 1.8g re lawyers representing two or more clients – shall not participate in making
an aggregate settlement of the claims
o And plus, the proposal divides the class of Piper residents (but the FCRP has
rules on how a judge can divide a class into two or more sub-classes).

You arrange for outside counsel to meet separately with them and advise them about
potential conflicts that you have. The prisoners would be willing to move to Piper if (a) the
people at Piper don’t object and (p) your org keeps representing them to enforce the consent
judgment. The Piper residents oppose moving the prisoners to Piper, but don’t mind 100
people being moved to group homes. In light of this, the prisoners don’t want the settlement
offer, because they want to act in solidarity with the Piper residents.
Can your org keep representing the plaintiffs in both classes without a written conflict
waiver?
1. YES, no need for consent, by rules or otherwise [OUR ANSWER, because it’s
only the settlement offer that created the conflict]
2. YES, no need for consent, but might as well get waivers
3. NO, the rules require consent, so you have to get waivers [OUR FIRST
ANSWER]
4. NO, there’s a non-consentable conflict and must withdraw in at least one of the
cases even if both groups consent
Do you need consent at all?
Yes, because there’s a foreseeable conflict in the future if circumstances change. One class
could change their mind, and the judge might want to see consent to totally confirm that this
is okay with everybody.
Yes, there’s a conflict – or at least today there’s no conflict, but tomorrow there might be.
The state might make another settlement offer that reignites the conflict.

Once you have the waivers, what do you need to do to make sure you can represent both
classes in the litigation that follows?
 Bring them to the court – ask the judge to hold a hearing to determine there is no
continuing conflict of interest and you can continue to represent more. At the
hearing, it would be great if both classes could testify that they still want you to
represent them – supplementing the signed waivers with oral testimony. (Though on

157
the other hand, this raises the issue for the judge, and they might rule the wrong
way.)

The state brought up this offer two weeks before your trial – seems suspicious. How would
you find out if they were deliberately trying to sabotage your case? Who in the state would
know? The bureaucrats who had worked on this idea. Can you interview those people?
 Yes: there’s a government exception to MR 4.2 (which otherwise bars you from
talking to represented people.).

If you claimed bad motives, would the state have violated a rule?
YES
 MR 4.4
 MR 8.4c or d

IN REAL LIFE
The judge held a 2-day hearing, in which all named parties testified against the settlement.
The director of the org wrote a letter too. The judge agreed there was no conflict. The
prisoners won a new women’s prison. The state appealed. The appeals judge vacated the
judgment on the grounds that there was a 1.7 conflict!

TAKEAWAYS
 Opponents can raise a disqualification issue, even if your side thinks everything is fine.
 It’s a big disputed question whether a legal services org is a firm under MR 1.0 Comment
4 or not.

13-2 Mandatory Pro Bono Services


You’re one of the ABA’s House of Delegates
Are you FOR or AGAINST a choice between mandatory pro bono 50 hours, or $1250
release fee?
REASONS FOR:
 Benefits to the lawyer – experience, direct work, more interesting
o And many lawyers go through their whole career without meeting a poor person.
Doing so makes us deeper, more broadly informed people, and exposes us to
more career choices.
 Such a massive gap in civil legal services that can only be met by strongly incentivizing or
requiring lawyers to fill it
 Lawyers are especially privileged people and so have duty
o Good PR for law as a profession/group
REASONS AGAINST:
 Prof. Maute *790: could lead to incompetence/substandard work, especially for
lawyers who are super-specialized (but still have to follow rules about competence)
 Hard on small firms (& thus their clients)
 If you’re already doing public interest work, it’s silly (& buyout hurts you more too)
 Freedom: it should be a personal choice
 No other profession requires it
 Might make lawyers just help people they already know
ALTERNATIVES:

158
 Only make big law lawyers do it
 Make it be a firm-by-firm requirement (assigning 10 hours per lawyer), not lawyer-
by-lawyer requirement
 Take away the $1250 buyout
 Direct the buyout to legal services organizations
 Make the buyout tied to the lawyers’ income

13-3 Special Education


Delaware state law Rule 4ciii and 4cv bars nonlawyers from “representing a person in a
governmental agency” (which includes public school systems) or “legal tribunals”, and from
doing things traditionally performed exclusively by lawyers. There’s a Resource Center that
represents disabled kids in accommodations requests. The Board has ordered them to stop.
Using legal arguments and policy concerns, would you recommend:
1. Affirming the Board’s order?
 Legal:
o The Center would be representing a person in a governmental agency Rule
4ciii – so it breaks the rule!
o And the Center would be engaging in activities traditionally performed
exclusively by lawyers.
o Federal IDEA Act doesn’t apply because “accompanied and advised” isn’t
the same as representing them.
 Same problem in asylum credible fear interviews: lawyers and
students can’t speak and ask questions!
 Whereas lawyers get the right to ask questions and speak in/to the
court from the Bar, and from our licenses!
 Policy:
o Nonlawyers aren’t subject to discipline or the rules, so can get away with
being crappy, or breaking confidentiality, or attorney-client privilege

2. Reversing the Boards order?


 Policy:
o The Center appears competent
o Saves a lot of money, so makes it more competent
o People who can’t afford lawyer will just be pro se, which puts them even
worse off, since school boards usually resist the accommodations that the
parents seek. I.e. it’s very unfair to poor people.
o Should be a relatively straight-forward factual inquiry based on expert
medical opinions and evidence (vs. say a custody battle)
 Legal:
o Under Federal IDEA Act (Individuals with Disabilities Education Act),
Center has “special knowledge and training” (20 USC § 1415h1)
o Constitutional argument: due process argument; equal protection clause
argument; maybe the Rule is unconstitutionally vague (since the ABA hasn’t
defined vagueness in decades of trying)
o Also, cross-examination, other hearings stuff aren’t exclusively performed by
lawyers: pro se people, prisoners (jail house lawyers), law students, social
workers do it all the time.
IN REAL LIFE
159
Delaware in SC held that because Congress had only let laypeople advise, not represent, the
federal statute didn’t apply. Rejected due process argument. The SC denied cert. The
Resource Center left the state.

14-1 Do You Need a Lawyer?


FL rule barred writing to a personal injury victim to offer legal services until 30 days after the
injury occurs. But insurances adjusters often settle claims with victims well before the 30
days elapse
Arguments FOR the rule:
- Discourages frivolous lawsuits
- Discourages ambulance-chasing (& thus increases dignity of the profession, and thereby
of the justice system)
- Doesn’t bar people from getting services; just bars lawyers from reaching out to people
- Shapero: lets lawyers send letters, but only if they have a label IDing them as ads
- Survey showed that most Floridians thought that lawyer letters to accident victims were
invasions of privacy
Arguments AGAINST the rule:
- Leaves people unrepresented at a vulnerable time
- Mail is a light burden, especially when it’s labeled as an ad.

IN REAL LIFE
The court upheld the rule, to protect the flagging reputations of Florida lawyers, because
mailing the letters is viewed as abhorrent! (“Went for it” law firm)

160

Das könnte Ihnen auch gefallen